Sei sulla pagina 1di 135

1.

AIEEECBSEENG03
A function f from the set of natural numbers to integers defined by

n 1
2 , whenis odd
f (n) =
is
n
, whenn is even
2
(A) oneone but not onto
(C) oneone and onto both

(B) onto but not oneone


(D) neither oneone nor onto

2.

Let z1 and z2 be two roots of the equation z2 + az + b = 0, z being complex.


Further, assume that the origin, z1 and z2 form an equilateral triangle, then
(A) a2 = b
(B) a2 = 2b
2
(C) a = 3b
(D) a2 = 4b

3.

If z and are two nonzero complex numbers such that |z| = 1, and Arg (z)

Arg () =
, then z is equal to
2
(A) 1
(B) 1
(C) i
(D) i
x

4.

5.

1+ i
If
= 1, then
1 i
(A) x = 4n, where n is any positive integer
(B) x = 2n, where n is any positive integer
(C) x = 4n + 1, where n is any positive integer
(D) x = 2n + 1, where n is any positive integer

a
If b
c

a2
b2
c2

1+a3
1+b3
1+c3

= 0 and vectors (1, a, a2) (1, b, b2) and (1, c, c2) are non

coplanar, then the product abc equals


(A) 2
(B) 1
(C) 1
(D) 0
6.

7.

If the system of linear equations


x + 2ay + az = 0
x + 3by + bz = 0
x + 4cy + cz = 0
has a nonzero solution, then a, b, c
(A) are in A. P.
(C) are in H.P.

(B) are in G.P.


(D) satisfy a + 2b + 3c = 0

If the sum of the roots of the quadratic equation ax 2 + bx + c = 0 is equal to the


a b
c
,
sum of the squares of their reciprocals, then
and
are in
c a
b
(A) arithmetic progression
(B) geometric progression
(C) harmonic progression
(D) arithmeticgeometricprogression

8.

The number of real solutions of the equation x2 3 |x| + 2 = 0 is


(A) 2
(B) 4
(C) 1
(D) 3

9.

The value of a for which one root of the quadratic equation


(a2 5a + 3) x2 + (3a 1) x + 2 = 0 is twice as large as the other, is
2
2
(A)
(B)
3
3
1
1
(C)
(D)
3
3

I10.

a
If A =
b
(A) = a2
(C) = a2

b

and A2 =

, then
a

+ b2, = ab
(B) = a2 + b2, = 2ab
2
2
2
+b,=a b
(D) = 2ab, = a2 + b2

11.

A student is to answer 10 out of 13 questions in an examination such that he


must choose at least 4 from the first five questions. The number of choices
available to him is
(A) 140
(B) 196
(C) 280
(D) 346

12.

The number of ways in which 6 men and 5 women can dine at a round table if no
two women are to sit together is given by
(A) 6! 5!
(B) 30
(C) 5! 4!
(D) 7! 5!

13.

If 1, , 2 are the cube roots of unity, then


n
1

2n
n
2n
1 is equal to
=
2n
n

(A) 0
(C)

(B) 1
(D) 2

14.

If nCr denotes the number of combinations of n things taken r at a time, then the
expression nCr+1 + nCr1 + 2 nCr equals
(A) n+2Cr
(B) n+2Cr+1
n+1
(C)
Cr
(D) n+1Cr+1

15.

The number of integral terms in the expansion of ( 3 + 8 5 )256 is


(A) 32
(B) 33
(C) 34
(D) 35

16.

If x is positive, the first negative term in the expansion of (1 + x)27/5 is


(A) 7th term
(B) 5th term
(C) 8th term
(D) 6th term

17.

The sum of the series


(A) 2 loge 2

1
1
1

+
upto is equal to
1 2 2 3 3 4
(B) log2 2 1

(C) loge 2

4
(D) loge
e

18.

Let f (x) be a polynomial function of second degree. If f (1) = f ( 1) and a, b, c


are in A. P., then f (a), f (b) and f (c) are in
(A) A.P.
(B) G.P.
(C) H. P.
(D) arithmeticgeometric progression

19.

If x1, x2, x3 and y1, y2, y3 are both in G.P. with the same common ratio, then the
points (x1, y1) (x2, y2) and (x3, y3)
(A) lie on a straight line
(B) lie on an ellipse
(C) lie on a circle
(D) are vertices of a triangle

20.

The sum of the radii of inscribed and circumscribed circles for an n sided regular
polygon of side a, is


a

(A) a cot
(B)
cot
n
2

2n


a

(C) a cot
(D)
cot
4
2n
2n

21.

22.

C
A
3b
If in a triangle ABC a cos2 + c cos2 =
, then the sides a, b and c
2
2
2
(A) are in A.P.
(B) are in G.P.
(C) are in H.P.
(D) satisfy a + b = c

In a triangle ABC, medians AD and BE are drawn. If AD = 4, DAB =


ABE =

, then the area of the ABC is


3

8
3
32
(C)
3
(A)

and
6

16
3
64
(D)
3
(B)

23.

The trigonometric equation sin1 x = 2 sin1 a, has a solution for


1
1
(A)
< |a| <
(B) all real values of a
2
2
1
1
(C) |a| <
(D) |a|
2
2

24.

The upper

25.

The real number x when added to its inverse gives the minimum value of the
sum at x equal to
(A) 2
(B) 1
(C) 1
(D) 2

3
3
th portion of a vertical pole subtends an angle tan1
at point in
4
5
the horizontal plane through its foot and at a distance 40 m from the foot. A
possible height of the vertical pole is
(A) 20 m
(B) 40 m
(C) 60 m
(D) 80 m

26.

If f : R R satisfies f (x + y) = f (x) + f (y), for all x, y R and f (1) = 7, then


n

f(r) is
r =1

(A)

7(n +1)
2
7n(n +1)
(D)
2

7n
2

(B)

(C) 7n (n + 1)

27.

If f (x) = xn, then the value of f (1)


(A) 2n
(C) 0

28.

(B) 2n1
(D) 1

3
+ log10 (x3 x), is
4 x2
(B) ( 1, 0) (1, 2)
(D) ( 1, 0) (1, 2) (2, )

Domain of definition of the function f (x) =


(A) (1, 2)
(C) (1, 2) (2, )

29.

f(1)
f(1)
f(1)
(1)n f n (1)
+

+... +
is
1!
2!
3!
n!

x
1 tan [1 sinx]
2
lim
is
x / 2
x
3
1+ tan [ 2x]
2

1
8
1
(C)
32
(A)

(B) 0
(D)

log(3 + x) log(3 x)
= k, the value of k is
x0
x
1
(A) 0
(B)
3
2
2
(C)
(D)
3
3

30.

If lim

31.

Let f (a) = g (a) = k and their n th derivatives fn (a), gn (a) exist and are not equal
f(a)g(x) f(a) g(a)f(x) + g(a)
for some n. Further if xlim
= 4, then the value
a
g(x) f(x)
of k is
(A) 4
(B) 2
(C) 1
(D) 0

32.

The function f (x) = log (x +


(A) an even function
(C) a periodic function

x2 +1 ), is

(B) an odd function


(D) neither an even nor an odd function

33.

1 + 1
|x| x
,
If f (x) = xe
0
,

x 0

then f (x) is

x= 0

(A) continuous as well as differentiable for all x


(B) continuous for all x but not differentiable at x = 0
(C) neither differentiable nor continuous at x = 0
(D) discontinuous everywhere
34.

If the function f (x) = 2x3 9ax2 + 12a2 x + 1, where a > 0, attains its maximum
and minimum at p and q respectively such that p2 = q, then a equals
(A) 3
(B) 1
1
(C) 2
(D)
2

35.

If f (y) = e , g (y) = y; y > 0 and F (t) =

f (t y) g (y) dy, then


0

(A) F (t) = 1 et (1 + t)
(C) F (t) = t et

(B) F (t) = et (1 + t)
(D) F (t) = t et
b

36.

If f (a + b x) = f (x), then

f (x) dx is equal to

(A)

a +b
f(b x)dx
2 a

(B)

a +b
f(x)dx
2 a

b a
f(x)dx
(C)
2 a

a +b
f(a + b x)dx
(D)
2 a

x2

sec

37.

The value of

lim

x0

t dt

is

x sin x

(A) 3
(C) 1

(B) 2
(D) 0
1

38.

The value of the integral I =

x (1 x)n dx is

1
n +1
1
1
(C)

n +1
n +2
(A)

39.

1
n+2
1
1
(D)
+
n +1
n+2
(B)

1+ 24 + 34 + ......+ n4
1+ 23 + 33 + ......+ n3
is

lim
n
n
n5
n5
1
(A)
(B) zero
30
lim

(C)

40.

1
4

(D)

esin x
d

Let
F (x) =
x , x > 0. If
dx

possible values of k, is
(A) 15
(C) 63

1
5

x e

sin x3

dx = F (k) F (1), then one of the

(B) 16
(D) 64

41.

The area of the region bounded by the curves y = |x 1| and y = 3 |x| is


(A) 2 sq units
(B) 3 sq units
(C) 4 sq units
(D) 6 sq units

42.

Let f (x) be a function satisfying f (x) = f (x) with f (0) = 1 and g (x) be a function
1

that satisfies f (x) + g (x) = x . Then the value of the integral


2

f (x) g (x) dx, is

e2 5

2 2
e2 3
(C) e

2
2

e2 3

2
2
2
e
5
(D) e +
+
2
2

(A) e

(B) e +

43.

The degree and order of the differential equation of the family of all parabolas
whose axis is xaxis, are respectively
(A) 2, 1
(B) 1, 2
(C) 3, 2
(D) 2, 3

44.

The solution of the differential equation (1 + y2) + (x etan

(A) (x 2) = k
1

(C) x etan

e tan

= tan1 y + k

(B) 2x e2 tan

(D) x e2 tan

dy
= 0, is
dx

+k
1

= etan

+k

45.

If the equation of the locus of a point equidistant from the points (a 1, b1) and (a2,
b2) is (a1 a2) x + (b1 b2) y + c = 0, then the value of c is
1
(A) (a22 + b22 a12 b12 )
(B) a12 + a22 + b12 b22
2
1
(C) (a12 + a22 b12 b22 )
(D) a12 + b12 a22 b22
2

46.

Locus of centroid of the triangle whose vertices are (a cos t, a sin t), (b sin t, b
cos t) and (1, 0), where t is a parameter, is
(A) (3x 1)2 + (3y)2 = a2 b2
(B) (3x 1)2 + (3y)2 = a2 + b2
2
2
2
2
(C) (3x + 1) + (3y) = a + b
(D) (3x + 1)2 + (3y)2 = a2 b2

47.

If the pair of straight lines x2 2pxy y2 = 0 and x2 2qxy y2 = 0 be such that


each pair bisects the angle between the other pair, then
(A) p = q
(B) p = q
(C) pq = 1
(D) pq = 1

48.

a square of side a lies above the xaxis and has one vertex at the origin. The

side passing through the origin makes an angle (0 < <


) with the positive
4
direction of xaxis. The equation of its diagonal not passing through the origin is
(A) y (cos sin ) x (sin cos ) = a
(B) y (cos + sin ) + x (sin cos ) = a
(C) y (cos + sin ) + x (sin + cos ) = a
(D) y (cos + sin ) + x (cos sin ) = a

49.

If the two circles (x 1)2 + (y 3)2 = r2 and x2 + y2 8x + 2y + 8 = 0 intersect in


two distinct points, then
(A) 2 < r < 8
(B) r < 2
(C) r = 2
(D) r > 2

50.

The lines 2x 3y = 5 and 3x 4y = 7 are diameters of a circle having area as


154 sq units. Then the equation of the circle is
(A) x2 + y2 + 2x 2y = 62
(B) x2 + y2 + 2x 2y = 47
2
2
(C) x + y 2x + 2y = 47
(D) x2 + y2 2x + 2y = 62

51.

The normal at the point (bt12, 2bt1) on a parabola meets the parabola again in the
point (bt22, 2bt2), then
2
2
(A) t2 = t1
(B) t2 = t1 +
t1
t1
2
2
(D) t2 = t1
(D) t2 = t1 +
t1
t1

52.

The foci of the ellipse


Then the value of b2 is
(A) 1
(C) 7

x2 y2
x2
y2
1
coincide.
+ 2 = 1 and the hyperbola

=
16 b
144 81 25
(B) 5
(D) 9

53.

A tetrahedron has vertices at O (0, 0, 0), A (1, 2, 1), B (2, 1, 3) and C ( 1, 1, 2).
Then the angle between the faces OAB and ABC will be
19
17

(A) cos1
(B) cos1
35
31
(C) 300
(D) 900

54.

The radius of the circle in which the sphere x2 + y2 + z2 + 2x 2y 4z 19 = 0 is


cut by the plane x + 2y + 2z + 7 = 0 is
(A) 1
(B) 2
(C) 3
(D) 4

55.

The lines

56.

The two lines x = ay + b, z = cy + d and x = ay + b, z = cy + d will be


perpendicular, if and only if
(A) aa + bb + cc + 1 = 0
(B) aa + bb + cc = 0

x2 y3 z 4
x 1 y 4 z 5
=
=
=
=
and
are coplanar if
1
1
k
k
2
1
(A) k = 0 or 1
(B) k = 1 or 1
(C) k = 0 or 3
(D) k = 3 or 3

(C) (a + a) (b + b) + (c + c) = 0

(D) aa + cc + 1 = 0

57.

The shortest distance from the plane 12x + 4y + 3z = 327 to the sphere x 2 + y2
+ z2 + 4x 2y 6z = 155 is
4
(A) 26
(B) 11
13
(C) 13
(D) 39

58.

Two systems of rectangular axes have the same origin. If a plane cuts them at
distances a, b, c and a, b, c from the origin, then

(A)

(C)

59.

111111
+2 2 + 2 + 2 + 2 + 2
a b c a b c
111111
2 2 2 + 2 2 2
a b c a b c

(B)

=0

(D)

=0

=0



a, b, c are 3 vectors, such that a + b + c = 0 , | a |= 1, | b |= 2, | c | = 3, then


a b + b c + c a is equal to
(A) 0
(C) 7

60.

=0

111111
+2 2 2 + 2 + 2 2
a b c a b c
111111
+2 2 + 2 2 2 2
a b c a b c

(B) 7
(D) 1

u, v
and
are
three
w



u + v w) (u v) (v w) equals

If

(A) 0

(C) u w v

noncoplanar

vectors,

then


(B) u v w

(D) 3 u v w

61.

i 4
j +7k
i 6
j +10k
Consider points A, B, C and D with position vectors 7
and 5
respectively. Then ABCD is a
i 3
j +4k
i
j +5k
,
(A) square
(B) rhombus
(C) rectangle
(D) parallelogram but not a rhombus

62.

are the sides of a triangle


, and AC =5
The vectors AB = 3
i 2
j +4k
i + 4k
ABC. The length of the median through A is
(A) 18
(B) 72

(C)

(D)

33

288

63.

and 3
is displaced
i +
j 3k
i +
j k
A particle acted on by constant forces 4
to the point 5
. The total work done by the
i +2
j +3k
i +4
j +k
from the point
forces is
(A) 20 units
(B) 30 units
(C) 40 units
(D) 50 units

64.

. If n
=0
i +
j, v =
i
j and w =
i + 2
j +3k
Let u =
is unit vector such that u n

| is equal to
= 0, then | w n
and v n
(A) 0
(B) 1
(C) 2
(D) 3

65.

The median of a set of 9 distinct observations is 20.5. If each of the largest 4


observations of the set is increased by 2, then the median of the new set
(A) is increased by 2
(B) is decreased by 2
(C) is two times the original median
(D) remains the same as that of the
original set

66.

In an experiment with 15 observations on x, then following results were


available:
x2 = 2830, x = 170
One observation that was 20 was found to be wrong and was replaced by the
correct value 30. Then the corrected variance is
(A) 78.00
(B) 188.66
(C) 177.33
(D) 8.33

67.

Five horses are in a race. Mr. A selects two of the horses at random and bets on
them. The probability that Mr. A selected the winning horse is
4
3
(A)
(B)
5
5
1
2
(C)
(D)
5
5

68.

Events A, B, C are mutually exclusive events such that P (A) =

1 x
and
4
values of x are in the interval
1 1
(A) ,

3 2
1 13
(C) ,
3
3

69.

P (C) =

3x +1
, P (B) =
3

1 2x
. The set of possible
2

1 2
(B) ,

3 3

(D) [0, 1]

The mean and variance of a random variable having a binomial distribution are 4
and 2 respectively, then P (X = 1) is
1
1
(A)
(B)
32
16
1
1
(C)
(D)
8
4

70.

71.

72.

The resultant of forces P and Q is R . If Q is doubled then R is doubled. If

the direction of Q is reversed, then R is again doubled. Then P2 : Q2 : R2 is


(A) 3 : 1 : 1
(B) 2 : 3 : 2
(C) 1 : 2 : 3
(D) 2 : 3 : 1
Let R1 and R2 respectively be the maximum ranges up and down an inclined
plane and R be the maximum range on the horizontal plane. Then R 1, R, R2 are
in
(A) arithmeticgeometric progression (B) A.P.
(C) G.P.
(D) H.P.

A couple is of moment G and the force forming the couple is P . If P is turned

through a right angle, the moment of the couple thus formed is H . If instead,

the forces P are turned through an angle , then the moment of couple
becomes

(A) G sin H cos


(B) H cos + G sin

(C) G cos H sin


(D) H sin G cos

73.

Two particles start simultaneously from the same point and move along two
straight lines, one with uniform velocity u and the other from rest with uniform

acceleration f . Let be the angle between their directions of motion. The


relative velocity of the second particle with respect to the first is least after a
time
u sin
f cos
(A)
(B)
f
u
ucos
(C) u sin
(D)
f

74.

Two stones are projected from the top of a cliff h meters high, with the same
speed u so as to hit the ground at the same spot. If one of the stones is projected
horizontally and the other is projected at an angle to the horizontal then tan
equals
2u
u
(A)
(B) 2g
gh
h
(C) 2h

75.

u
g

(D) u

2
gh

A body travels a distances s in t seconds. It starts from rest and ends at rest. In
the first part of the journey, it moves with constant acceleration f and in the
second part with constant retardation r. The value of t is given by
2s
1 1
(A) 2s +
(B) 1 1
+
f r
f
r
(C)

2s(f +r)

(D)

1 1
2s +
f r

1.

2.

5.

Solutions
Clearly both one one and onto
Because if n is odd, values are set of all nonnegative integers and if n is an
even, values are set of all negative integers.
Hence, (C) is the correct answer.
z12 + z22 z1z2 = 0
(z1 + z2)2 3z1z2 = 0
a2 = 3b.
Hence, (C) is the correct answer.
a
b
c

a2
b2
c2

a2
b2 = 0
c2

1 1 a
1 +1 b
1 1 c

a
(1 + abc) b
c

a2
b2
c2

1
1 =0
1

abc = 1.
Hence, (B) is the correct answer
4.

1+ i (1+ i)2
=i
=
1 i
2
x

1+ i

= ix
1 i
x = 4n.
Hence, (A) is the correct answer.

6.

2a

Coefficient determinant = 1
1

3b
4c

b =0
c

2ac
.
a +c
Hence, (C) is the correct answer
b=

8.

x2 3 |x| + 2 = 0
(|x| 1) (|x| 2) = 0
x = 1, 2.
Hence, (B) is the correct answer

7.

Let , be the roots


1
1
+= 2 + 2

+=

2 + 2 2
( + )

b b2 2ac
=
c2
a
2a2c = b (a2 + bc)
a b c
,
,

are in H.P.
c a b

Hence, (C) is the correct answer


10.

9.

a b
A=

b a
a b a b
A2 =

b a b a
a2 + b2
2ab
=

2
a + b2
2ab
= a2 + b2, = 2ab.
Hence, (B) is the correct answer.

= 2

3a 1
a 5a + 3
2
22 = 2
a 5a + 6
3 =

(3a 1)2
1
= 2
2
2
a(a 5a + 3)
a + 5a + 6
2
a=
.
3
Hence, (A) is the correct answer

12.

Clearly 5! 6!
(A) is the correct answer

11.

Number of choices = 5C4 8C6 + 5C5 8C5


= 140 + 56.
Hence, (B) is the correct answer

13.

1+n +2n
n
2n
= 1+ +
1+n +2n

n
2n
1

2n
1
n

=0
Since, 1 + n + 2n = 0, if n is not a multiple of 3
Therefore, the roots are identical.
Hence, (A) is the correct answer
14.

17.

Cr+1 + nCr1 + nCr + nCr


= n+1Cr+1 + n+1Cr
= n+2Cr+1.
Hence, (B) is the correct answer
n

1
1
1

+

1 2 2 3 3 4
1 1 1 1 1
+ +
=1

2 2 3 3 4
1
1 1

=12
2 3 + 4 .........

1 1 1

= 2 1 + + ......... 1
2 3 4

= 2 log 2 log e
4
= log .
e
Hence, (D) is the correct answer.

15.

General term = 256Cr ( 3 )256r [(5)1/8]r


From integral terms, or should be 8k
k = 0 to 32.
Hence, (B) is the correct answer.

18.

f (x) = ax2 + bx + c
f (1) = a + b + c
f ( 1) = a b + c
a + b + c = a b + c also 2b = a + c
f (x) = 2ax + b = 2ax
f (a) = 2a2
f (b) = 2ab
f (c) = 2ac
AP.
Hence, (A) is the correct answer.

19.

Result (A) is correct answer.

20.

(B)

21.

26.

1+ cosC
1+ cosA 3b
+ c
=
a
2
2
2

a + c + b = 3b
a + c = 2b.
Hence, (A) is the correct answer

f (1) = 7
f (1 + 1) = f (1) + f (1)
f (2) = 2 7
only f (3) = 3 7
n

f(r) = 7 (1 + 2 + + n)
r =1

=7
25.

(B)

23.

n(n +1)
.
2

sin2 x

4
2
4

sin1 (a)
4
4
1
1
|a|
.
2
2
Hence, (D) is the correct answer

n n(n 1) n(n 1)(n 2)


+

+
1!
2!
3!
= 1 nC1 + nC2
= 0.
Hence, (C) is the correct answer

27.

LHS = 1

30.

1
1
+
2.
lim 3 + x 3 x =
x 0
1
3

Hence, (C) is the correct answer.


28.

29.

4 x2 0
x2
x3 x > 0
x (x + 1) (x 1) > 0.
Hence (D) is the correct answer.

x
tan (1 sinx)
4 2
lim
x / 2
x
4 ( 2x)2
4 2
1
=
.
32
Hence, (C) is the correct answer.

32.

f ( x) = f (x)
Hence, (B) is the correct answer.

1.

sin ( + ) =

x
40

x
sin a =
140
x = 40.
Hence, (B) is the correct answer

3x/4
1

tan (3/4)
x/4

40

34.

f (x) = 0 at x = p, q
6p2 + 18ap + 12a2 = 0
6q2 + 18aq + 12a2 = 0
f (x) < 0 at x = p
and f (x) > 0 at x = q.

30.

Applying L. Hospitals Rule


f(a)g(a) g(a)f(a)
lim
=4
x2a
g(a) f(a)

k(g(a) ff (a))
=4
(g(a) f (a))
k = 4.

Hence, (A) is the correct answer.


b

36.

x f (x) dx
a

(a + b x) f (a + b x) dx.

Hence, (B) is the correct answer.


33.

37.

f (0)
f (0 h) = 1
f (0 + h) = 0
LHD RHD.
Hence, (B) is the correct answer.

tan(x2 )
x0
x sinx
tan(x2 )
lim
= x0
sinx
x2

x
= 1.
Hence (C) is the correct answer.
lim

38.

x (1 x)n dx =

(1 x)

1
1

.
n +1 n + 2
0
Hence, (C) is the correct answer.
=

(x

xn+1) =

35.

F (t) =

f (t y) f (y) dy
0

f (y) f (t y) dy
0
t

(t y) dy

= xt (1 + t).
Hence, (B) is the correct answer.
34.

Clearly f (x) > 0 for x = 2a q = 2a < 0 for x = a p = a


or p2 = q a = 2.
Hence, (C) is the correct answer.

40.

F (x) =
=

esin x
3x

x e

sin x

dx = F (k) F (1)

64

esin x
dx = F (k) F (1)
x
1

64

F(x) dx = F (k) F (1)


1

F (64) F (1) = F (k) F (1)


k = 64.
Hence, (D) is the correct answer.
41.

Clearly area = 2
= sq units

2
(1,2)

(2,1)
(1,0)

45.

Let p (x, y)
(x a1)2 + (y b1)2 = (x a2)2 + (y b2)2
1 2
(a1 a2) x + (b1 b2) y +
(b b12 + a22 a12 ) = 0.
2 2
Hence, (A) is the correct answer.

46.

x=

a cost + b sint + 1
a sint bcost +1
,y=
3
3
2

1
a2 + b2

.
x + y2 =
3
9

Hence, (B) is the correct answer.

43.

Equation y2 = 4a 9x h)
2yy1 = 4a yy1 = 2a
yy2 = y12 = 0.
Hence (B) is the correct answer.
1

42.

f(x) [x

f (x)] dx

solving this by putting f (x) = f (x).


Hence, (B) is the correct answer.
50.

47.
49.

Intersection of diameter is the point (1, 1)


s2 = 154
s2 = 49
(x 1)2 + (y + 1)2 = 49
Hence, (C) is the correct answer.
(D)
dx
1
(1 + y2) = (esin y x)
dy
1

x
dx
esub y
+
=
dy
1+ y
1+ y2

x2
52.

12

5

y2
2

9

5
5
e1 =
4
ae2 =

=1

b2
4=3
16

b2 = 7.
Hence, (C) is the correct answer.
54.

(C)

4
3

69.

np = 4
npq = 2
1
1
q=
,p=
2
2
n=8
8

1
p (x = 1) = 8C1
2
1
=
.
32
Hence, (A) is the correct answer.

49.

(x 1)2 + (y 3)2 = r2
(x 4)2 + (y + 2)2 16 4 + 8 = 0
(x 4)2 + (y + 2)2 = 12.

67.

Select 2 out of 5
2
=
.
5
Hence, (D) is the correct answer.

65.

3x + 1 1 x 1 2x
+
+
1
3
4
2
12x + 4 + 3 3x + 6 12x 1
0 13 3x 12
3x 13
1
x
3
13
x
.
3
Hence, (C) is the correct answer.

3.

z
Arg =
2
|z| = 1
z = i or + i.

FIITJEE

AIEEE 2004 (MATHEMATICS)

Important Instructions:
1
2

i)

The test is of 1 hours duration.

ii)

The test consists of 75 questions.

iii)

The maximum marks are 225.

iv)

For each correct answer you will get 3 marks and for a wrong answer you will
get -1 mark.

1.

Let R = {(1, 3), (4, 2), (2, 4), (2, 3), (3, 1)} be a relation on the set A = {1, 2, 3,
4}. The relation R is
(1) a function
(2) reflexive
(3) not symmetric
(4) transitive

2.

The range of the function f(x) = 7- xPx- 3 is


(1) {1, 2, 3}
(2) {1, 2, 3, 4, 5}
(3) {1, 2, 3, 4}
(4) {1, 2, 3, 4, 5, 6}

3.

Let z, w be complex numbers such that z +iw = 0 and arg zw = . Then arg z
equals
p
5p
(1)
(2)
4
4
3p
p
(3)
(4)
4
2

4.

If z = x i y and

1
z3

x y
+
p q is equal to
,
then
=p +iq
p2 +q2

(1) 1
(3) 2
5.

(2) -2
(4) -1
2

If z2 - 1 = z +1, then z lies on


(1) the real axis
(3) a circle

6.

(2) an ellipse
(4) the imaginary axis.

0 0 - 1

Let A = 0 - 1 0 . The only correct statement about the matrix A is


- 1 0 0

AIEEE-PAPERS--2

(1) A is a zero matrix


(3) A - 1 does not exist

7.

8.

(2) A 2 =I
(4) A =( - 1) I , where I is a unit matrix

1 - 1 1
4 2 2

Let A =2 1 - 3( 10) B =- 5 0 a . If B is the inverse of matrix A, then is


1 1 1
1 - 2 3

(1) -2
(2) 5
(3) 2
(4) -1

If a1, a2 , a3 , ....,an , .... are in G.P., then the value of the determinant
logan logan+1 logan+2
logan+3 logan+4 logan+5 , is
logan+6 logan+7 logan+8
(1) 0
(3) 2

(2) -2
(4) 1

9.

Let two numbers have arithmetic mean 9 and geometric mean 4. Then these
numbers are the roots of the quadratic equation
(1) x2 +18x +16 =0
(2) x2 - 18x - 16 =0
(3) x2 +18x - 16 =0
(4) x2 - 18x +16 =0

10.

If (1 p) is a root of quadratic equation x2 +px +( 1- p) =0 , then its roots are


(1) 0, 1
(2) -1, 2
(3) 0, -1
(4) -1, 1

11.

Let S(K) =1+3 +5 +... +( 2K - 1) =3 +K 2 . Then which of the following is true?


(1) S(1) is correct
(2) Principle of mathematical induction can be used to prove the formula
(3) S(K) S(K +1)
(4) S(K) S(K +1)

12.

How many ways are there to arrange the letters in the word GARDEN with the
vowels in alphabetical order?
(1) 120
(2) 480
(3) 360
(4) 240

13.

The number of ways of distributing 8 identical balls in 3 distinct boxes so that


none of the boxes is empty is
(1) 5
(2) 8 C 3
(3) 38

14.

(4) 21

If one root of the equation x2 +px +12 =0 is 4, while the equation x2 +px +q =0
has equal roots, then the value of q is
49
(1)
(2) 4
4
(3) 3
(4) 12
FIITJEE Ltd. ICES House, Sarvapriya Vihar (Near Hauz Khas Bus Term.), New Delhi - 16, Ph : 2686 5182, 26965626, 2685 4102, 26515949
Fax : 2651394

AIEEE-PAPERS--3

15.

The coefficient of the middle term in the binomial expansion in powers of x of


4

( 1+ax)

and of ( 1- ax)

is the same if equals

5
3
-3
(3)
10

3
5
10
(4)
3

(1) -

16.

(2)

The coefficient of xn in expansion of ( 1+x) ( 1- x) is


(1) (n 1)
(3) ( - 1)

17.

n- 1

( n - 1)

If S n =

r=0

Cr

and tn =

r =0

r
n

Cr

, then

1
(1) n
2

(2) ( - 1)

(4) ( - 1)

n- 1

( 1- n)
n

tn
is equal to
Sn
1
n- 1
2
2n - 1
(4)
2

(2)

(3) n 1
18.

Let Tr be the rth term of an A.P. whose first term is a and common difference is d.
1
1
If for some positive integers m, n, m n, Tm = and Tn = , then a d equals
n
m
(1) 0
(2) 1
1
1 1
+
(3)
(4)
mn
m n

19.

The sum of the first n terms of the series 12 +2 22 +32 +2 42 +52 +2 62 +... is
n( n +1)

when n is even. When n is odd the sum is


2
3n( n +1)
n2 ( n +1)
(1)
(2)
2
2
(3)

20.

n( n +1)
4

The sum of series

(e
(1)

(3)

n( n +1)
(4)

-1

2
2
e -1

2e

1 1 1
+ + +... is
2! 4! 6!

(2)

( e - 1)

(4)

(e

2e

-2
e

FIITJEE Ltd. ICES House, Sarvapriya Vihar (Near Hauz Khas Bus Term.), New Delhi - 16, Ph : 2686 5182, 26965626, 2685 4102, 26515949
Fax : 2651394

AIEEE-PAPERS--4

21.

Let , be such that < - < 3. If sin + sin = then the value of cos

(3)
22.

(2)

130

130
6
(4) 65

6
65

If u = a2 cos2 q+b2 sin2 q + a2 sin2 q+b2 cos2 q , then the difference between the
maximum and minimum values of u2 is given by

2
2
(1) 2 a +b

(3) ( a +b)
23.

a- b
is
2

(1) -

21
27
and cos + cos = ,
65
65

(2) 2 a2 +b2

(4) ( a - b)

The sides of a triangle are sin, cos and

1+sin a cos a for some 0 < <

Then the greatest angle of the triangle is


(1) 60o
(2) 90o
(3) 120o
(4) 150o

p
.
2

24.

A person standing on the bank of a river observes that the angle of elevation of
the top of a tree on the opposite bank of the river is 60o and when he retires 40
meter away from the tree the angle of elevation becomes 30o . The breadth of the
river is
(1) 20 m
(2) 30 m
(3) 40 m
(4) 60 m

25.

If f : R S, defined by f(x) =sinx (1) [0, 3]


(3) [0, 1]

26.

The graph of the function y = f(x) is symmetrical about the line x = 2, then
(1) f(x + 2)= f(x 2)
(2) f(2 + x) = f(2 x)
(3) f(x) = f(-x)
(4) f(x) = - f(-x)

27.

The domain of the function f(x) =


(1) [2, 3]
(3) [1, 2]

3 cos x +1, is onto, then the interval of S is


(2) [-1, 1]
(4) [-1, 3]

sin- 1 ( x - 3)

is
9 - x2
(2) [2, 3)
(4) [1, 2)

2x

28.

a b
If lim 1+ + 2 =e2 , then the values of a and b, are
x
x x
(1) a R , b R
(2) a = 1, b R
(3) a R, b =2
(4) a = 1 and b = 2
FIITJEE Ltd. ICES House, Sarvapriya Vihar (Near Hauz Khas Bus Term.), New Delhi - 16, Ph : 2686 5182, 26965626, 2685 4102, 26515949
Fax : 2651394

AIEEE-PAPERS--5

29.

30.

1- tanx
p
p
p
p
, x , x 0, . If f(x) is continuous in 0, , then f is
Let f(x) =
4x - p
4
2
2
4
1
(1) 1
(2)
2
1
(3) (4) -1
2
y +...to

If x =ey+e

, x > 0, then

x
1+x
1- x
(3)
x

dy
is
dx
1
x
1+x
(4)
x

(1)

(2)

31.

A point on the parabola y2 =18x at which the ordinate increases at twice the rate
of the abscissa is
(1) (2, 4)
(2) (2, -4)
- 9 9
9 9
(3) ,
(4) ,
8 2
8 2

32.

A function y = f(x) has a second order derivative f(x) = 6(x 1). If its graph
passes through the point (2, 1) and at that point the tangent to the graph is y =
3x 5, then the function is
(1) ( x - 1)

(2) ( x - 1)

(3) ( x +1)

(4) ( x +1)

33.

The normal to the curve x = a(1 + cos), y = asin at always passes through
the fixed point
(1) (a, 0)
(2) (0, a)
(3) (0, 0)
(4) (a, a)

34.

If 2a + 3b + 6c =0, then at least one root of the equation ax2 +bx +c =0 lies in
the interval
(1) (0, 1)
(2) (1, 2)
(3) (2, 3)
(4) (1, 3)
n

35.

36.

1 n
lim
e is
n
r =1 n
(1) e
(3) 1 e

If

(2) e 1
(4) e + 1

sinx

sin(x - a ) dx =Ax +Blogsin(x - a ) +C , then value of (A, B) is

(1) (sin, cos)


(3) (- sin, cos)

(2) (cos, sin)


(4) (- cos, sin)

FIITJEE Ltd. ICES House, Sarvapriya Vihar (Near Hauz Khas Bus Term.), New Delhi - 16, Ph : 2686 5182, 26965626, 2685 4102, 26515949
Fax : 2651394

AIEEE-PAPERS--6

37.

dx

cos x - sinx is equal to


(1)
(3)

x p
log tan - +C
2
2 8

(2)

x 3p
log tan +C
2
2 8

(4)

x
log cot +C
2
2

x 3p
log tan + +C
2
2 8

38.

The value of

|1- x

|dx is

-2

28
(1)
3
7
(3)
3

14
3
1
(4)
3

(2)

p/ 2

39.

The value of I =

(sinx +cos x)2

1+sin2x

dx is

(1) 0
(3) 2

(2) 1
(4) 3

40.

If

p/ 2

xf(sinx)dx =A

f(sinx) dx, then A is


0

(2)

(1) 0
p
(3)
4
41.

If f(x) =

(4) 2
ex
, I1 =
1+ex

f(a)

f(a)

xg{x(1- x)}dx and I2 =

f(- a)

g{x(1- x)}dx

then the value of

f(- a)

I2
I1

is
(1) 2
(3) 1

(2) 3
(4) 1

42.
The area of the region bounded by the curves y = |x 2|, x = 1, x = 3 and the xaxis is
(1) 1
(2) 2
(3) 3
(4) 4
43.

The differential equation for the family of curves x2 +y2 - 2ay =0 , where a is an
arbitrary constant is
(1) 2(x2 - y2 )y=xy
(2) 2(x2 +y2 )y=xy
(3) (x2 - y2 )y=2xy

44.

(4) (x2 +y2 )y=2xy

The solution of the differential equation y dx + (x + x2y) dy = 0 is


1
1
=C
+logy =C
(1) (2) xy
xy
FIITJEE Ltd. ICES House, Sarvapriya Vihar (Near Hauz Khas Bus Term.), New Delhi - 16, Ph : 2686 5182, 26965626, 2685 4102, 26515949
Fax : 2651394

AIEEE-PAPERS--7

(3)

1
+logy =C
xy

(4) log y = Cx

45.

Let A (2, 3) and B(2, 1) be vertices of a triangle ABC. If the centroid of this
triangle moves on the line 2x + 3y = 1, then the locus of the vertex C is the line
(1) 2x + 3y = 9
(2) 2x 3y = 7
(3) 3x + 2y = 5
(4) 3x 2y = 3

46.

The equation of the straight line passing through the point (4, 3) and making
intercepts on the co-ordinate axes whose sum is 1 is
x y
x y
x y
x y
+ =- 1
+ =- 1
(1) + =- 1and
(2) - =- 1and
2 3
-2 1
2 3
-2 1
x y
x y
x y
x y
+ =1
(3) + =1and + =1
(4) - =1and
2 3
2 1
2 3
-2 1

47.

If the sum of the slopes of the lines given by x2 - 2cxy - 7y2 =0 is four times their
product, then c has the value
(1) 1
(2) 1
(3) 2
(4) 2

48.

If one of the lines given by 6x2 - xy +4cy2 =0 is 3x + 4y = 0, then c equals


(1) 1
(2) 1
(3) 3
(4) 3

49.

If a circle passes through the point (a, b) and cuts the circle x2 +y2 =4
orthogonally, then the locus of its centre is
(1) 2ax +2by +(a2 +b2 +4) =0
(2) 2ax +2by - (a2 +b2 +4) =0
(3) 2ax - 2by +(a2 +b2 +4) =0

50.

A variable circle passes through the fixed point A (p, q) and touches x-axis. The
locus of the other end of the diameter through A is
(1) (x - p)2 =4qy
(2) (x - q)2 =4py
(3) (y - p)2 =4qx

51.

(4) x2 +y2 +2x - 2y - 23 =0

The intercept on the line y = x by the circle x2 +y2 - 2x =0 is AB. Equation of the
circle on AB as a diameter is
(1) x2 +y2 - x - y =0
(2) x2 +y2 - x +y =0
(3) x2 +y2 +x +y =0

53.

(4) (y - q)2 =4px

If the lines 2x + 3y + 1 = 0 and 3x y 4 = 0 lie along diameters of a circle of


circumference 10, then the equation of the circle is
(1) x2 +y2 - 2x +2y - 23 =0
(2) x2 +y2 - 2x - 2y - 23 =0
(3) x2 +y2 +2x +2y - 23 =0

52.

(4) 2ax - 2by - (a2 +b2 +4) =0

(4) x2 +y2 +x - y =0

If a 0 and the line 2bx + 3cy + 4d = 0 passes through the points of intersection
of the parabolas y2 =4ax and x2 =4ay , then
FIITJEE Ltd. ICES House, Sarvapriya Vihar (Near Hauz Khas Bus Term.), New Delhi - 16, Ph : 2686 5182, 26965626, 2685 4102, 26515949
Fax : 2651394

AIEEE-PAPERS--8

54.

(1) d2 +(2b +3c)2 =0

(2) d2 +(3b +2c)2 =0

(3) d2 +(2b - 3c)2 =0

(4) d2 +(3b - 2c)2 =0

The eccentricity of an ellipse, with its centre at the origin, is


directrices is x = 4, then the equation of the ellipse is
(1) 3x2 +4y2 =1
(2) 3x2 +4y2 =12
(3) 4x2 +3y2 =12

1
. If one of the
2

(4) 4x2 +3y2 =1

55.

A line makes the same angle , with each of the x and z axis. If the angle ,
which it makes with y-axis, is such that sin2 b =3sin2 q, then cos2 q equals
2
1
(1)
(2)
3
5
3
2
(3)
(4)
5
5

56.

Distance between two parallel planes 2x + y + 2z = 8 and 4x + 2y + 4z + 5 = 0


is
3
5
(1)
(2)
2
2
7
9
(3)
(4)
2
2

57.

A line with direction cosines proportional to 2, 1, 2 meets each of the lines x = y


+
a
=
z
and
x + a = 2y = 2z. The co-ordinates of each of the point of intersection are given
by
(1) (3a, 3a, 3a), (a, a, a)
(2) (3a, 2a, 3a), (a, a, a)
(3) (3a, 2a, 3a), (a, a, 2a)
(4) (2a, 3a, 3a), (2a, a, a)

58.

If the straight lines x = 1 + s, y = 3 s, z = 1 + s and x =

59.

The

t
, y = 1 + t, z = 2
2
t with parameters s and t respectively, are co-planar then equals
(1) 2
(2) 1
1
(3)
(4) 0
2

intersection
2

of

the

spheres

x2 +y2 +z2 +7x - 2y - z =13

and

x +y +z - 3x +3y +4z =8 is the same as the intersection of one of the sphere


and the plane
(1) x y z = 1
(2) x 2y z = 1
(3) x y 2z = 1
(4) 2x y z = 1

60.

r r
r
Let a, b and c be three non-zero vectors such that no two of these are collinear.
r
r
r
r
r
r
If the vector a +2b is collinear with c and b +3c is collinear with a ( being some
r
r
r
non-zero scalar) then a +2b +6c equals
FIITJEE Ltd. ICES House, Sarvapriya Vihar (Near Hauz Khas Bus Term.), New Delhi - 16, Ph : 2686 5182, 26965626, 2685 4102, 26515949
Fax : 2651394

AIEEE-PAPERS--9

r
(1) l a
r
(3) l c

r
(2) l b
(4) 0

61.

A particle is acted upon by constant forces 4i +j - 3k and 3i +j - k which displace


it from a point i +2j +3k to the point 5i +4j +k . The work done in standard units
by the forces is given by
(1) 40
(2) 30
(3) 25
(4) 15

62.

If a, b, c are non-coplanar vectors and is a real number, then the vectors


a +2b +3c, l b +4c and (2l - 1)c are non-coplanar for
(1) all values of
(2) all except one value of
(3) all except two values of
(4) no value of

63.

Let u, v, w be such that u =1, v =2, w =3 . If the projection v along u is equal


to that of w along u and v, w are perpendicular to each other then u - v +w
equals
(1) 2
(2) 7
(3) 14
(4) 14

64.

Let a, b and c be non-zero vectors such that (a b) c =

1
b c a . If is the acute
3

angle between the vectors b and c , then sin equals


1
2
(1)
(2)
3
3
2
2 2
(3)
(4)
3
3
65.

Consider the following statements:


(a) Mode can be computed from histogram
(b) Median is not independent of change of scale
(c) Variance is independent of change of origin and scale.
Which of these is/are correct?
(1) only (a)
(2) only (b)
(3) only (a) and (b)
(4) (a), (b) and (c)

66.

In a series of 2n observations, half of them equal a and remaining half equal a.


If the standard deviation of the observations is 2, then |a| equals
1
(1)
(2) 2
n
2
(3) 2
(4)
n

67.

The probability that A speaks truth is

4
3
, while this probability for B is . The
5
4
probability that they contradict each other when asked to speak on a fact is
FIITJEE Ltd. ICES House, Sarvapriya Vihar (Near Hauz Khas Bus Term.), New Delhi - 16, Ph : 2686 5182, 26965626, 2685 4102, 26515949
Fax : 2651394

AIEEE-PAPERS--10

3
20
7
(3)
20

(1)

1
5
4
(4)
5

(2)

68.

A random variable X has the probability distribution:


X:
1
2
3
4
5
6
7
8
p(X) 0.15 0.23 0.12 0.10 0.20 0.08 0.07 0.05
:
For the events E = {X is a prime number} and F = {X < 4}, the probability P (E
F) is
(1) 0.87
(2) 0.77
(3) 0.35
(4) 0.50

69.

The mean and the variance of a binomial distribution are 4 and 2 respectively.
Then the probability of 2 successes is
37
219
(1)
(2)
256
256
128
28
(3)
(4)
256
256

70.

With two forces acting at a point, the maximum effect is obtained when their
resultant is 4N. If they act at right angles, then their resultant is 3N. Then the
forces are
(1) (2 + 2)N and (2 - 2)N
(2) (2 + 3)N and (2 - 3)N
1

2 N
(3) 2 + 2 N and 2 2
2

71.

72.

73.

10

3 N
(4) 2 + 3 N and 2 2
2

In a right angle ABC, A = 90 and sides a, b, c are respectively, 5 cm, 4 cm


r
and 3 cm. If a force F has moments 0, 9 and 16 in N cm. units respectively
r
about vertices A, B and C, then magnitude of F is
(1) 3
(2) 4
(3) 5
(4) 9
r r
r
Three forces P, Q and R acting along IA, IB and IC, where I is the incentre of a
r r r
ABC, are in equilibrium. Then P : Q : R is
A
B
C
A
B
C
(1) cos : cos : cos
(2) sin : sin : sin
2
2
2
2
2
2
A
B
C
A
B
C
(3) sec : sec : sec
(4) cosec : cosec : cosec
2
2
2
2
2
2

A particle moves towards east from a point A to a point B at the rate of 4 km/h
and then towards north from B to C at the rate of 5 km/h. If AB = 12 km and BC
= 5 km, then its average speed for its journey from A to C and resultant average
velocity direct from A to C are respectively
17
13
13
17
(1)
km/h and
km/h
(2)
km/h and
km/h
4
4
4
4
FIITJEE Ltd. ICES House, Sarvapriya Vihar (Near Hauz Khas Bus Term.), New Delhi - 16, Ph : 2686 5182, 26965626, 2685 4102, 26515949
Fax : 2651394

AIEEE-PAPERS--11

17
13
13
17
km/h and
km/h
(4)
km/h and
km/h
9
9
9
9
1
A velocity
m/s is resolved into two components along OA and OB making
4
angles 30 and 45 respectively with the given velocity. Then the component
along OB is
1
1
(1) m/s
(2) ( 3 - 1) m/s
8
4
1
1
(3) m/s
(4) ( 6 - 2) m/s
4
8

(3)
74.

75.

If t1 and t2 are the times of flight of two particles having the same initial velocity
u and range R on the horizontal, then t12 +t22 is equal to
u2
(1)
g

(3)

u2
2g

4u2
(2) 2
g

(4) 1

FIITJEE Ltd. ICES House, Sarvapriya Vihar (Near Hauz Khas Bus Term.), New Delhi - 16, Ph : 2686 5182, 26965626, 2685 4102, 26515949
Fax : 2651394

11

AIEEE-PAPERS--12

FIITJEE

AIEEE 2004 (MATHEMATICS)


ANSWERS

1.
2.
3.
4.
5.
6.
7.
8.
9.
10.
11.
12.
13.
14.
15.

12

3
1
3
2
4
2
2
1
4
3
4
3
4
1
3

16.
17.
18.
19.
20.
21.
22.
23.
24.
25.
26.
27.
28.
29.
30.

2
1
1
2
2
1
4
3
1
4
2
2
2
3
3

31. 4
32. 2
33. 1
34. 1
35. 2
36. 2
37. 4
38. 1
39. 3
40. 2
41. 1
42. 1
43. 3
44. 2
45. 1

46.
47.
48.
49.
50.
51.
52.
53.
54.
55.
56.
57.
58.
59.
60.

4
3
4
2
1
1
1
1
2
3
3
2
1
4
4

61.
62.
63.
64.
65.
66.
67.
68.
69.
70.
71.
72.
73.
74.
75.

1
3
3
4
3
3
3
2
4
3
3
1
1
4
2

FIITJEE Ltd. ICES House, Sarvapriya Vihar (Near Hauz Khas Bus Term.), New Delhi - 16, Ph : 2686 5182, 26965626, 2685 4102, 26515949
Fax : 2651394

AIEEE-PAPERS--13

FIITJEE

AIEEE 2004 (MATHEMATICS)


SOLUTIONS

1.

(2, 3) R but (3, 2) R.


Hence R is not symmetric.

2.

f(x) =7- xPx- 3


7- x 0 x 7
x - 3 0 x 3,
x5
and 7 - x x - 3
3 x 5 x = 3, 4, 5 Range is {1, 2, 3}.

3.

Here =

4.

z =( p +iq) =p p2 - 3q2 - iq q2 - 3p2

5.

z
3p
z
arg z. =p 2 arg(z) arg(i) = arg(z) =
.
i
4
i

y
x
=p2 - 3q2 &
=q2 - 3p2
p
q
2

x y
+
p q
p2 +q2

) ( z - 1) ( z - 1) = z
2

z2 - 1 = z +1

=- 2 .

+2 z +1

z2 +z 2 +2zz =0 z +z =0
R (z) = 0 z lies on the imaginary axis.

6.

7.

1 0 0

A.A = 0 1 0 =I .

0 0 1

AB = I
1

2

1
logan

8.

A(10 B) = 10
1 4 2

1 - 3
- 5 0
1 1

1 - 2

-1

logan+1 logan+2

logan+3 logan+4 logan+5


logan+6

logan+7

C3 C 3
logan
= logan+3
logan+6
9.

I
2 10 0 5 - a
1 0 0

a = 0 10 a - 5 =10
0 1 0if a =5 .

3

0 0 5 +a

0 0 1

C2, C2
logr
logr
logr

logan+8

C3 C 1
logr
logr = 0 (where r is a common ratio).
logr

Let numbers be a, b a +b =18,


equation

ab =4

ab =16 , a and b are roots of the

FIITJEE Ltd. ICES House, Sarvapriya Vihar (Near Hauz Khas Bus Term.), New Delhi - 16, Ph : 2686 5182, 26965626, 2685 4102, 26515949
Fax : 2651394

13

AIEEE-PAPERS--14

x2 - 18x +16 =0 .

10.

(3)
2

( 1- p)

+p ( 1- p) +( 1- p) =0
p) = 0)
( 1- p) ( 1- p +p +1) =0

(since (1 p) is a root of the equation x2 + px + (1

2( 1- p) =0 (1 p) = 0 p = 1
sum of root is a +b =- p and product ab =1- p =0
a +0 =- 1 a =- 1 Roots are 0, 1

11.

(where = 1 p = 0)

S ( k) =1+3 +5 +........ +( 2k - 1) =3 +k2


S(k + 1)=1 + 3 + 5 +............. + (2k 1) + (2k + 1)

2
2
= 3 +k +2k +1 =k +2k +4 [from S(k) = 3 +k2 ]
= 3 + (k2 + 2k + 1) = 3 + (k + 1)2 = S (k + 1).
Although S (k) in itself is not true but it considered true will always imply towards
S (k + 1).

12.

Since in half the arrangement A will be before E and other half E will be before A.
6!
Hence total number of ways =
= 360.
2

13.

Number of balls = 8
number of boxes = 3
Hence number of ways = 7C2 = 21.

14.

Since 4 is one of the root of x2 + px + 12 = 0 16 + 4p + 12 = 0 p = 7


and equation x2 + px + q = 0 has equal roots
49
D = 49 4q = 0 q =
.
4

15.

Coefficient of Middle term in ( 1+ax) =t3 =4C 2 a 2

Coefficient of Middle term in ( 1- ax) =t4 =6C3 ( - a )

-3
10
Coefficient of xn in (1 + x)(1 x)n = (1 + x)(nC0 nC1x + .. + (1)n 1 nCn 1 xn 1
+ (1)n nCn xn)
4

16.

C 2a 2 =- 6 C3 .a 3

- 6 =20a

= (1)n nCn + (1)n 1 nCn 1


n

17.

t=

r =0

14

n
r +n - r
n
=

n
n
Cr
r=0
r =0 Cr

=( - 1)

n
n
r
n- r
n- r
=
=
n
n
Cr r=0 Cn- r r=0 n Cr

2tn =

18.

( 1- n) .

(Q

tn =

a=

Cr =n Cn- r

n n 1
n
= Sn
n
2 r=0 Cr 2

1
Tm = =a +( m - 1) d
n

tn
n
=
Sn 2

.....(1)

FIITJEE Ltd. ICES House, Sarvapriya Vihar (Near Hauz Khas Bus Term.), New Delhi - 16, Ph : 2686 5182, 26965626, 2685 4102, 26515949
Fax : 2651394

AIEEE-PAPERS--15

and Tn =

1
=a +( n - 1) d
m

.....(2)

from (1) and (2) we get a =


Hence a d = 0

1
1
, d=
mn
mn

If n is odd then (n 1) is even sum of odd terms =

19.

( n - 1) n2
2

+n =

n2 ( n +1)
2

ea +e- a
a2 a 4 a6
=1+ +
+ +..
2
2! 4! 6!
ea +e- a
a2 a 4 a6
- 1=
+
+ +.......
2
2! 4! 6!
put = 1, we get

20.

( e - 1)
2e

1 1 1 ..
+ + +
2! 4! 6!

21
27
and cos + cos = .
65
65
Squaring and adding, we get
1170
2 + 2 cos ( ) =
(65)2

sin + sin = -

21.

-3
a - b 9
a - b
2
=
=
cos
cos

2
130
2 130

22.

p a - b 3p
Q 2 < 2 < 2 .

u = a2 cos2 q+b2 sin2 q + a2 sin2 q+b2 cos2 q

a2 +b2 a2 - b2
a2 +b2 b2 - a2
+
cos 2q +
+
cos 2q
2
2
2
2
2

a2 +b2 a2 - b2
2
u =a +b +2
-
cos 2q
2
2

2
2
2
min value of u =a +b +2ab
2

2
2
2
max value of u =2 a +b

2
2
umax
- umin
=( a - b) .

23.

Greatest side is 1+sin a cos a , by applying cos rule we get greatest angle = 120.

24.

tan30 =

h
40 +b
3 h =40 +b

..(1)

30

40

60

tan60 = h/b h = 3 b .(2)


b = 20 m
FIITJEE Ltd. ICES House, Sarvapriya Vihar (Near Hauz Khas Bus Term.), New Delhi - 16, Ph : 2686 5182, 26965626, 2685 4102, 26515949
Fax : 2651394

15

AIEEE-PAPERS--16

25.

- 2 sinx - 3 cos x 2 - 1 sinx range of f(x) is [1, 3].


Hence S is [1, 3].

26.

If y = f (x) is symmetric about the line x = 2 then f(2 + x) = f(2 x).

27.

9 - x2 >0 and - 1 x - 3 1 x [2, 3)

28.

b a + b
x x2
2

2x

a b
a
lim 1+ + 2 = lim 1+ +
x

x x
x x

29.

3 cos x +1 3

a b
2x
+ 2

x x

=e2a

a =1, b R

1- tanx
1- tanx
1
f(x) =
lim
=p
4x - p
4x - p
2
x
4

30.

31.

y+ey+.....

x = ey+x
dy 1
1- x
= - 1=
lnx x = y
.
dx x
x
x =ey+e

9 2

Any point be t , 9t; differentiating y2 = 18x


2

dy 9 1
1
= = =2 (given) t = .

dx y t
2
9 9
Point is ,
8 2

32.

f (x) = 6(x 1) f (x) = 3(x 1)2 + c


and f (2) = 3 c = 0
f (x) = (x 1)3 + k and f (2) = 1 k = 0
f (x) = (x 1)3.

33.

Eliminating , we get (x a)2 + y2 = a2.


Hence normal always pass through (a, 0).

34.

Let f(x) = ax2 +bx +c f(x) =


f(x) =

ax3 bx2
+
+cx +d
3
2

1
2ax3 +3bx2 +6cx +6d , Now f(1) = f(0) = d, then according to Rolles
6

theorem
f(x) = ax2 +bx +c =0 has at least one root in (0, 1)
n

35.
36.
16

lim

1 n
e =
r =1 n

e dx =(e - 1)
x

Put x = t
FIITJEE Ltd. ICES House, Sarvapriya Vihar (Near Hauz Khas Bus Term.), New Delhi - 16, Ph : 2686 5182, 26965626, 2685 4102, 26515949
Fax : 2651394

AIEEE-PAPERS--17

sin(a +t)
dt =sin a cottdt +cos a dt
sint
= cos a ( x - a ) +sin a ln sint +c
A = cos a, B = sin a

37.

1
1
=
dx
1
1
dx
p
x 3p
=
sec x + dx =
log tan + +C
2 cos x + p
4
cos x - sinx
2

2
2 8

-1

38.

( x

- 1 dx +

-2

-1

x3
x3
1- x dx + x - 1 dx =
- x +x 3
3
1
-2

-1

p
2

x3
28
+ - x =
.
3
3
-1
1

39.

2
( sinx +cos x)
dx = ( sinx +cos x) dx

0 ( sinx +cos x)
0

40.

Let I =

= - cos x +sinx 2 = 2.
0

xf(sinx)dx = (p - x)f(sinx)dx =p f(sinx)dx - I (since f (2a x) = f (x))

0
p/ 2

I=

f(sinx)dx A = .
0

41.

f(-a) + f(a) = 1
f(a)

I1 =

f(a)

xg{x(1- x)}dx =

f( - a)

( 1- x) g{x(1-

x)}dx

f( - a)

b
b

Q f ( x) dx = f ( a +b - x) dx

a
a

f(a)

2I1 =

g{x(1- x)}dx = I

I2 / I1 = 2.

f( - a)

42.

Area =

y =x 2

(2 - x)dx + (x - 2)dx = 1.

y=2 x

43.

2x + 2yy - 2ay = 0
x +yy
a=
(eliminating a)
y
(x2 y2)y = 2xy.

45.

y dx + x dy + x2y dy = 0.
FIITJEE Ltd. ICES House, Sarvapriya Vihar (Near Hauz Khas Bus Term.), New Delhi - 16, Ph : 2686 5182, 26965626, 2685 4102, 26515949
Fax : 2651394

17

AIEEE-PAPERS--18

d(xy)

1
1
+ dy =0 +logy =C .
y
xy
x y
2 2

45.

46.

If C be (h, k) then centroid is (h/3, (k 2)/3) it lies on 2x + 3y = 1.


locus is 2x + 3y = 9.
x y
4 3
+ =1where a + b = -1 and + =1
a b
a b
a = 2, b = -3 or a = -2, b = 1.
x y
x y
+ =1.
Hence - =1 and
2 3
-2 1
2c
1
and m1 m2 = 7
7
m1 + m2 = 4m1m2 (given)
c = 2.

47.

m1 + m2 = -

48.

m1 + m2 =

49.

Let the circle be x2 + y2 + 2gx + 2fy + c = 0 c = 4 and it passes through (a, b)


a2 + b2 + 2ga + 2fb + 4 = 0.
Hence locus of the centre is 2ax + 2by (a2 + b2 + 4) = 0.

50.

Let the other end of diameter is (h, k) then equation of circle is


(x h)(x p) + (y k)(y q) = 0
Put y = 0, since x-axis touches the circle
x2 (h + p)x + (hp + kq) = 0 (h + p)2 = 4(hp + kq)
(x p)2 = 4qy.

1
6
3
, m1m2 =
and m1 = - .
4c
4c
4
Hence c = -3.

(D = 0)

51.

Intersection of given lines is the centre of the circle i.e. (1, 1)


Circumference = 10 radius r = 5
equation of circle is x2 + y2 2x + 2y 23 = 0.

52.

Points of intersection of line y = x with x2 + y2 2x = 0 are (0, 0) and (1, 1)


hence equation of circle having end points of diameter (0, 0) and (1, 1) is
x2 + y2 x y = 0.

53.

Points of intersection of given parabolas are (0, 0) and (4a, 4a)


equation of line passing through these points is y = x
On comparing this line with the given line 2bx + 3cy + 4d = 0, we get
d = 0 and 2b + 3c = 0
(2b + 3c)2 + d2 = 0.

54.

Equation of directrix is x = a/e = 4 a = 2


b2 = a2 (1 e2) b2 = 3
Hence equation of ellipse is 3x2 + 4y2 = 12.

18

FIITJEE Ltd. ICES House, Sarvapriya Vihar (Near Hauz Khas Bus Term.), New Delhi - 16, Ph : 2686 5182, 26965626, 2685 4102, 26515949
Fax : 2651394

AIEEE-PAPERS--19

55.

l = cos , m = cos , n = cos


cos2 + cos2 + cos2 = 1 2 cos2 = sin2 = 3 sin2
cos2 = 3/5.

(given)

56.

Given planes are


2x + y + 2z 8 = 0, 4x + 2y + 4z + 5 = 0 2x + y + 2z + 5/2 = 0
| d1 - d2 |
| - 8 - 5/ 2 |
7
= .
Distance between planes =
=
22 +12 +22 2
a2 +b2 +c2

57.

Any point on the line

x y +a z
=
= =t1 (say) is (t1, t1 a, t1) and any point on
1
1
1

x +a y z
= = =t2 ( say) is (2t2 a, t2, t2).
2
1 1
Now direction cosine of the lines intersecting the above lines is proportional to
(2t2 a t1, t2 t1 + a, t2 t1).
Hence 2t2 a t1 = 2k , t2 t1 + a = k and t2 t1 = 2k
On solving these, we get t1 = 3a , t2 = a.
Hence points are (3a, 2a, 3a) and (a, a, a).

the line

y- 1 z - 2
x - 1 y +3 z - 1
x
=
=
=s and
=
=
=t are coplanar then plan
1
-l
l
1/ 2
1
-1
passing through these lines has normal perpendicular to these lines
a
+b - c =0 (where a, b, c are direction ratios of the
a - b + c = 0 and
2
normal to the plan)
On solving, we get = -2.

58.

Given lines

59.

Required plane is S1 S2 = 0
where S1 = x2 + y2 + z2 + 7x 2y z 13 = 0 and
S2 = x2 + y2 + z2 3x + 3y + 4z 8 = 0
2x y z = 1.

60.

61.

( a +2br ) =t c

.(1)
r
r
and b +3c =t2a
.(2)
r
r
(1) 2(2) a ( 1+2t2 ) +c ( - t1 - 6) =0 1+ 2t2 = 0 t2 = -1/2 & t1 = -6.
r
r
Since a and c are non-collinear.
r
r
r r
Putting the value of t1 and t2 in (1) and (2), we get a +2b +6c =0 .
1

r
r
r r r
r
Work done by the forces F1 and F2 is (F1 +F2 ) d , where d is displacement
r r
+(3i +j - k)
=7i +2j - 4k
According to question F1 +F2 = (4i +j - 3k)
r
r r r
- (i +2j +3k)
=4i +2j - 2k . Hence (F +F ) d is 40.
and d =(5i +4j +k)
1

1 2

63.

4 = 0 = 0, 1/2.
Condition for given three vectors to be coplanar is 0 l
0 0 2l - 1
FIITJEE Ltd. ICES House, Sarvapriya Vihar (Near Hauz Khas Bus Term.), New Delhi - 16, Ph : 2686 5182, 26965626, 2685 4102, 26515949
Fax : 2651394

19

AIEEE-PAPERS--20

Hence given vectors will be non coplanar for all real values of except 0, 1/2.
Projection of v along u and w along u is

63.

According to question

v u
w u
and
respectively
|u|
|u|

v u w u
=
v u =w u . and v w =0
|u|
|u|

| u - v +w |2 =| u |2 +| v |2 +| w |2 - 2u v +2u w - 2v w = 14 | u - v +w |= 14 .

r r

r r r

r r r

( a b) c = 31 b c a ( a c) b - ( b c) a =31 b c a

64.

r r r 1
r r
1
r
a c b = b c + b c a a c =0 and b c + b c =0
3
3

2 2
b c +cos q=0 cos = 1/3 sin =
.
3

65.

Mode can be computed from histogram and median is dependent on the scale.
Hence statement (a) and (b) are correct.

66.

xi =a for i =1, 2, .... ,n and xi =- a for i =n, ...., 2n

1
2n

S.D. =

2n

( xi - x)
i=1

2=

1
2n

2n

xi2

i=1

Since

2n

i=1

1
=0 2 =
2na2
2n

a =2

67.

E1 : event denoting that A speaks truth


E 2 : event denoting that B speaks truth

Probability that both contradicts each other = P E1 E 2 +P E1 E 2

4 1 1 3 7
+ =
5 4 5 4 20

68.

P(E F) =P(E) +P(F) - P ( E F ) = 0.62 + 0.50 0.35 = 0.77

69.

Given that n p = 4, n p q = 2 q = 1/2 p = 1/2 , n = 8 p(x = 2) =


2

70.

20

28
1 1
C2 =
2 2 256

1
1

2 N .
P + Q = 4, P2 + Q2 = 9 P = 2 + 2 N and Q =2 2

FIITJEE Ltd. ICES House, Sarvapriya Vihar (Near Hauz Khas Bus Term.), New Delhi - 16, Ph : 2686 5182, 26965626, 2685 4102, 26515949
Fax : 2651394

AIEEE-PAPERS--21

71.

F . 3 sin = 9
F . 4 cos = 16
F = 5.

C
4cos

3sin

72.

By Lamis theorem
r r r
P :Q :R =
A
B
C

sin90 + : sin 90 + : sin90 +


2
2
2

A
B
C
cos : cos : cos .
2
2
2

73.

Time T1 from A to B =

90+C/2

90+A/2

12
= 3 hrs.
4

C
13

5
= 1 hrs.
5
Total time = 4 hrs.
17
Average speed =
km/ hr.
4

T2 from B to C =

Resultant average velocity =

74.

Component along OB =

75.

t1 =

90+B/2

12

13
km/hr.
4

1
sin30
1
4
=
sin(45 +30) 8

6-

2 m/s.

2usin a
2usinb
, t2 =
where + = 900
g
g

2
2
t1 +t2 =

4u2
.
g2

FIITJEE Ltd. ICES House, Sarvapriya Vihar (Near Hauz Khas Bus Term.), New Delhi - 16, Ph : 2686 5182, 26965626, 2685 4102, 26515949
Fax : 2651394

21

FIITJEE
SOLUTION TO AIEEE-2005
MATHEMATICS
1.
1.

2.

2.

3.

3.

4.

If A2 A + I = 0, then the inverse of A is


(1) A + I
(2) A
(3) A I
(4) I A
(4)
Given A2 A + I = 0
A1A2 A1A + A1 I = A10 (Multiplying A1 on both sides)
A - I + A-1 = 0 or A1 = I A.
If the cube roots of unity are 1, , 2 then the roots of the equation
(x 1)3 + 8 = 0, are
(1) -1 , - 1 + 2, - 1 - 22
(2) -1 , -1, - 1
(3) -1 , 1 - 2, 1 - 22
(4) -1 , 1 + 2, 1 + 22
(3)
(x 1)3 + 8 = 0 (x 1) = (-2) (1)1/3
x 1 = -2 or -2 or -22
or n = -1 or 1 2 or 1 22.
Let R = {(3, 3), (6, 6), (9, 9), (12, 12), (6, 12), (3, 9), (3, 12), (3, 6)} be a relation on
the set A = {3, 6, 9, 12} be a relation on the set A = {3, 6, 9, 12}. The relation is
(1) reflexive and transitive only
(2) reflexive only
(3) an equivalence relation
(4) reflexive and symmetric only
(1)
Reflexive and transitive only.
e.g.
(3, 3), (6, 6), (9, 9), (12, 12) [Reflexive]
(3, 6), (6, 12), (3, 12)
[Transitive].

(1) 2ab
(3) ab
4.

x2 y2
+
= 1 is
a 2 b2

Area of the greatest rectangle that can be inscribed in the ellipse

(1)
Area of rectangle ABCD = (2acos)
(2bsin) = 2absin2
Area of greatest rectangle is equal to
2ab
when sin2 = 1.

(2) ab
a
(4)
b
Y
(-acos, bsin)
B

A(acos, bsin)
X

(-acos, -bsin)C

5.

D(acos, -bsin)

The differential equation representing the family of curves y2 = 2c x + c , where c


> 0, is a parameter, is of order and degree as follows:
(1) order 1, degree 2
(2) order 1, degree 1

FIITJEE

Ltd. ICES House, 29-A, Kalu Sarai, Sarvapriya Vihar, New Delhi - 110016, Ph : 26515949, 26569493, Fax : 26513942

5.

(3) order 1, degree 3


(3)
y2 = 2c(x + c)
(i)
2yy = 2c1 or yy = c (ii)
y2 = 2yy (x + yy )

(4) order 2, degree 2

[on putting value of c from (ii) in (i)]

On simplifying, we get
(iii)
(y 2xy)2 = 4yy3
Hence equation (iii) is of order 1 and degree 3.
6.

6.

1
2
4
1
1

lim 2 sec 2 2 + 2 sec 2 2 + .... + 2 sec 2 1 equals


n
n
n
n
n

1
1
(1)
(2) cos ec1
sec1
2
2
1
(3) tan1
(4)
tan1
2
(4)
1
2
4
3
9
1
1

lim
sec 2 2 + 2 sec 2 2 + 2 sec 2 2 + .... + sec 2 1 is equal to
n n2
n
n
n
n
n
n

2
2
r
1 r
2 r
2 r
=

sec
lim
sec
n n2
n2 n n n
n2

lim

Given limit is equal to value of integral

x sec

x 2 dx

or
=
7.

7.

1
1
2x sec x 2 dx = sec 2 tdt

20
20

[put x2 = t]

1
1
1
( tan t )0 = tan1 .
2
2

ABC is a triangle. Forces P, Q, R acting along IA, IB and IC respectively are in


equilibrium, where I is the incentre of ABC. Then P : Q : R is
A
B
C
(1) sinA : sin B : sinC
(2) sin : sin : sin
2
2
2
A
B
C
(3) cos : cos : cos
(4) cosA : cosB : cosC
2
2
2
(3)
A
Using Lamis Theorem
A
B
C
P : Q : R = cos : cos : cos .
P
2
2
2
Q
B

8.

8.

If in a frequently distribution, the mean and median are 21 and 22 respectively, then
its mode is approximately
(1) 22.0
(2) 20.5
(3) 25.5
(4) 24.0
(4)
Mode + 2Mean = 3 Median
Mode = 3 22 2 21= 66 42= 24.

FIITJEE

Ltd. ICES House, 29-A, Kalu Sarai, Sarvapriya Vihar, New Delhi - 110016, Ph : 26515949, 26569493, Fax : 26513942

3
9.

9.

10.

10.

Let P be the point (1, 0) and Q a point on the locus y2 = 8x. The locus of mid point of
PQ is
(1) y2 4x + 2 = 0
(2) y2 + 4x + 2 = 0
2
(4) x2 4y + 2 = 0
(3) x + 4y + 2 = 0
(1)
P = (1, 0)
Q = (h, k) such that k2 = 8h
Let (, ) be the midpoint of PQ
h +1
k+0
=
=
,
2
2
2 - 1 = h
2 = k.
(2)2 = 8 (2 - 1) 2 = 4 - 2
y2 4x + 2 = 0.
If C is the mid point of AB and P is any point outside AB, then
(1) PA + PB = 2PC
(2) PA + PB = PC
(3) PA + PB + 2PC = 0
(4) PA + PB + PC = 0
(1)
PA + AC + CP = 0
PB + BC + CP = 0
Adding, we get
PA + PB + AC + BC + 2CP = 0
Since AC = BC

& CP = PC
PA + PB 2PC = 0 .
11.

11.

If the coefficients of rth, (r+ 1)th and (r + 2)th terms in the binomial expansion of (1 +
y)m are in A.P., then m and r satisfy the equation
(2) m2 m(4r+1) + 4r2 + 2 = 0
(1) m2 m(4r 1) + 4r2 2 = 0
2
2
(4) m2 m(4r 1) + 4r2 + 2 = 0
(3) m m(4r + 1) + 4r 2 = 0
(3)
Given m Cr 1, mCr , mCr +1 are in A.P.

2 mCr = mCr 1 + mCr +1


2=

Cr 1 m Cr +1
+ m
Cr
Cr

r
mr
+
m r +1 r +1
m2 m (4r + 1) + 4r2 2 = 0.

P
Q
In a triangle PQR, R = . If tan and tan are the roots of
2
2
2
ax2 + bx + c = 0, a 0 then
(1) a = b + c
(2) c = a + b
(3) b = c
(4) b = a + c
(2)
P
Q
tan , tan are the roots of ax2 + bx + c = 0
2

2
b
P
Q
tan + tan =
a
2
2
=

12.

12.

FIITJEE

Ltd. ICES House, 29-A, Kalu Sarai, Sarvapriya Vihar, New Delhi - 110016, Ph : 26515949, 26569493, Fax : 26513942

4
P
Q c
tan tan =
2

2 a
P
Q
tan + tan
2

2 = tan P + Q = 1
2 2
P
Q

1 tan tan
2
2
b

a = 1 b = a c b = a c

c
a a a
1
a
c = a + b.
13.

13.

The system of equations


x + y + z = - 1,
x + y + z = - 1,
x + y + z = - 1
has no solution, if is
(1) -2
(3) not -2
(1)
x + y + z = - 1
x + y + z = - 1
x + y + z = - 1
1 1
= 1 1
1 1

(2) either 2 or 1
(4) 1

= (2 1) 1( - 1) + 1(1 - )
= ( - 1) ( + 1) 1( - 1) 1( - 1)
( - 1)[2 + - 1 1] = 0
( - 1)[2 + - 2] = 0
[2 + 2 - - 2] = 0
( - 1) [( + 2) 1( + 2)] = 0
( - 1) = 0, + 2 = 0 = 2, 1; but 1.
14.

14.

The value of for which the sum of the squares of the roots of the equation
x2 (a 2)x a 1 = 0 assume the least value is
(1) 1
(2) 0
(3) 3
(4) 2
(1)
x2 (a 2)x a 1 = 0
+=a2
= (a + 1)
2 + 2 = ( + )2 - 2
= a2 2a + 6 = (a 1)2 + 5
a = 1.

15.

If roots of the equation x2 bx + c = 0 be two consectutive integers, then b2 4c


equals
(1) 2
(2) 3

FIITJEE

Ltd. ICES House, 29-A, Kalu Sarai, Sarvapriya Vihar, New Delhi - 110016, Ph : 26515949, 26569493, Fax : 26513942

15.

16.

16.

17.

(3) 2
(4)
Let , + 1 be roots
++1=b
( + 1) = c
b2 4c = (2 + 1)2 - 4( + 1) = 1.

If the letters of word SACHIN are arranged in all possible ways and these words are
written out as in dictionary, then the word SACHIN appears at serial number
(1) 601
(2) 600
(3) 603
(4) 602
(1)
Alphabetical order is
A, C, H, I, N, S
No. of words starting with A 5!
No. of words starting with C 5!
No. of words starting with H 5!
No. of words starting with I 5!
No. of words starting with N 5!
SACHIN 1
601.
The value of

50

C4 +

r =1

17.

(4) 1

56 r

C3 is

(1) 55C4
(3) 56C3
(4)
50

C4 +

(2) 55C3
(4) 56C4
6

56 r

r =1

50
50

C3

C4 + 55 C3 + 54 C3 + 53 C3 + 52C3 + 51C3 + 50 C3

)
C )+

C4 + 50 C3 + 51C3 + 52C3 + 53 C3 + 54 C3 + 55 C3

51

C4 + 51

52

C3 + 53 C3 + 54 C3 + 55 C3

55C4 + 55C3 = 56C4.


18.

18.

19.

19.

1 0
1 0
and I =
If A =

, then which one of the following holds for all n 1, by


1 1
0 1
the principle of mathematical indunction
(2) An = 2n-1A (n 1)I
(1) An = nA (n 1)I
n
(4) An = 2n-1A + (n 1)I
(3) A = nA + (n 1)I
(1)
By the principle of mathematical induction (1) is true.

1
If the coefficient of x in ax 2 +
bx

then a and b satisfy the relation


(1) a b = 1
a
(3) = 1
b
(4)
7

FIITJEE

11

11

1
equals the coefficient of x in ax 2 ,
bx

-7

(2) a + b = 1
(4) ab = 1

Ltd. ICES House, 29-A, Kalu Sarai, Sarvapriya Vihar, New Delhi - 110016, Ph : 26515949, 26569493, Fax : 26513942

6
11

11r 1
1

Tr + 1 in the expansion ax 2 + = 11Cr ax 2



bx

bx
11
11 r
-r
22 2r r
(b) (x)
= Cr (a)
22 3r = 7 r = 5
coefficient of x7 = 11C5(a)6 (b)-5 (1)
11
r
1
1
11r

Again Tr + 1 in the expansion ax 2 = 11Cr ( ax ) 2


bx

bx
11
11 r
r
-r
-2r
11 - r
(-1) (b) (x) (x)
= Cr a
Now 11 3r = -7 3r = 18 r = 6
coefficient of x-7 = 11C6 a5 1 (b)-6

11

C5 ( a ) ( b )
6

= 11C6a5 ( b )

ab = 1.
20.

20.

Let f : (-1, 1) B, be a function defined by f(x) = tan1


and onto when B is the interval

(1) 0,
2

(3) ,
2 2
(4)
2x
Given f(x) = tan1
for x(-1, 1)
2
1 x

clearly range of f(x) = ,
2 2

2x
, then f is both one-one
1 x2


(2) 0,
2

(4) ,
2 2


co-domain of function = B = , .
2 2
21.

21.

22.

22.

If z1 and z2 are two non-zero complex numbers such that |z1 + z2| = |z1| + |z2| then
argz1 argz2 is equal to

(2) -
(1)
2

(3) 0
(4) 2
(3)
|z1 + z2| = |z1| + |z2| z1 and z2 are collinear and are to the same side of origin;
hence arg z1 arg z2 = 0.
If =

and || = 1, then z lies on


1
z i
3
(1) an ellipse
(3) a straight line
(3)

FIITJEE

(2) a circle
(4) a parabola.

Ltd. ICES House, 29-A, Kalu Sarai, Sarvapriya Vihar, New Delhi - 110016, Ph : 26515949, 26569493, Fax : 26513942

7
z

As given w =

1
i
3

|w| =

|z|
= 1 distance of z from origin and point
1
| z i|
3

1
0, 3 is same hence z lies on bisector of the line joining points (0, 0) and (0, 1/3).

Hence z lies on a straight line.

(1 + b ) x (1 + c ) x
) x 1 + b x (1 + c ) x
) x (1 + b ) x 1 + c x

1 + a2 x
23.

23.

(
(1 + a

If a2 + b2 + c2 = -2 and f(x) = 1 + a2
2

polynomial of degree
(1) 1
(3) 3
(4)

then f(x) is a

(2) 0
(4) 2

(
) (1 + b ) x (1 + c ) x
f ( x ) = 1 + ( a + b + c + 2 ) x 1 + b x (1 + c ) x , Applying C
1 + ( a + b + c + 2 ) x (1 + b ) x 1 + c x
1 (1 + b ) x (1 + c ) x
= 1 1 + b x (1 + c ) x a + b + c + 2 = 0
1 (1 + b ) x 1 + c x
1 + a2 + b 2 + c 2 + 2 x

x 1

f(x) = 0

1 x

(1 + b ) x
2

C1 + C2 + C3

0
x 1 ; Applying R1 R1 R2 , R2 R2 R3
1 + c2 x

f(x) = (x 1)2
Hence degree = 2.
24.

24.

The normal to the curve x = a(cos + sin), y = a( sin - cos) at any point is
such that
(1) it passes through the origin

(2) it makes angle


+ with the x-axis
2

(3) it passes through a , a


2

(4) it is at a constant distance from the origin


(4)
dy
Clearly
= tan slope of normal = - cot
dx
Equation of normal at is
y a(sin - cos ) = - cot (x a(cos + sin )
y sin - a sin2 + a cos sin = -x cos + a cos2 + a sin cos
x cos + y sin = a
Clearly this is an equation of straight line which is at a constant distance a from
origin.

FIITJEE

Ltd. ICES House, 29-A, Kalu Sarai, Sarvapriya Vihar, New Delhi - 110016, Ph : 26515949, 26569493, Fax : 26513942

8
25.

25.

A function is matched below against an interval where it is supposed to be


increasing. Which of the following pairs is incorrectly matched?
Interval
Function
(1) (-, )
x3 3x2 + 3x + 3
(2) [2, )
2x3 3x2 12x + 6
1

3x2 2x + 1
(3) ,
3

(4) (- , -4]
x3 + 6x2 + 6
(3)
Clearly function f(x) = 3x2 2x + 1 is increasing when
f(x) = 6x 2 0 x[1/3, )
Hence (3) is incorrect.
2

26.

Let and be the distinct roots of ax + bx + c = 0, then lim

26.

equal to
a2
2
(1)
( )
2
a2
2
(3) ( )
2
(1)

27.

28.

( x )

is

(2) 0
(4)

1
2
( )
2

( x )( x )
2 sin2 a

2
1 cos a ( x )( x )

Given limit = lim


= lim
2
2
x
x

x
x
(
)
(
)

= lim

27.

1 cos ax 2 + bx + c

( x )( x )
sin2 a

2
2
2
a2 ( x ) ( x )

2
2
4
a2 ( x ) ( x )
4

(x )

a2 ( )
2

1
Suppose f(x) is differentiable x = 1 and lim f (1 + h ) = 5 , then f(1) equals
h 0 h
(1) 3
(2) 4
(3) 5
(4) 6
(3)
f (1 + h ) f (1)
; As function is differentiable so it is continuous as it is given
f (1) = lim
h0
h
f (1 + h )
that lim
= 5 and hence f(1) = 0
h 0
h
f (1 + h )
Hence f(1) = lim
=5
h 0
h
Hence (3) is the correct answer.
Let f be differentiable for all x. If f(1) = - 2 and f(x) 2 for x [1, 6] , then
(1) f(6) 8
(2) f(6) < 8
(3) f(6) < 5
(4) f(6) = 5

FIITJEE

Ltd. ICES House, 29-A, Kalu Sarai, Sarvapriya Vihar, New Delhi - 110016, Ph : 26515949, 26569493, Fax : 26513942

9
28.

(1)
As f(1) = - 2 & f(x) 2 x [1, 6]
Applying Lagranges mean value theorem
f ( 6 ) f (1)
= f (c ) 2
5
f(6) 10 + f(1)
f(6) 10 2
f(6) 8.

29.

If f is a real-valued differentiable function satisfying |f(x) f(y)| (x y)2, x, y R and


f(0) = 0, then f(1) equals
(1) -1
(2) 0
(3) 2
(4) 1
(2)
f ( x + h) f ( x )
f(x) = lim
h0
h

29.

| f ( x ) |= lim

f ( x + h) f ( x )
h

h0

(h)
lim
h0

|f(x)| 0 f(x) = 0 f(x) = constant


As f(0) = 0 f(1) = 0.
30.

If x is so small that x3 and higher powers of x may be neglected, then


1

1 + x
2

1/ 2
(1 x )

(1 + x )
(1) 1
(3)
30.

3/2

may be approximated as

3 2
x
8

3 2
x
8
x 3
(4) x 2
2 8

(2) 3x +

3 2
x
8

(3)
2
3
33 2
1
1
(1 x) 1 + x + 1 x 1 3 x 3 ( 2 ) x
22
2
2
2
3
3
= (1 x)1/2 x 2 = - x 2 .
8
8

1/2

31.

If x =

a ,
n =0

31.

n=0

n =0

y = bn , z = c n where a, b, c are in A.P. and |a| < 1, |b|<1, |c|< 1,

then x, y, z are in
(1) G.P.
(3) Arithmetic Geometric Progression
(4)

1
1
x = an =
a = 1
x
1 a
n =0

y = bn =
n =0

FIITJEE

1
1 b

b = 1

(2) A.P.
(4) H.P.

1
y

Ltd. ICES House, 29-A, Kalu Sarai, Sarvapriya Vihar, New Delhi - 110016, Ph : 26515949, 26569493, Fax : 26513942

10

1
1
c = 1

z
1
c
n =0
a, b, c are in A.P.
2b = a + c

1
1
1
2 1 = 1 + 1
y
x
y

2 1 1
= +
y x z
x, y, z are in H.P.
z = cn =

32.

32.

. If r is the inradius and R is the circumradius of the the


2
triangle ABC, then 2 (r + R) equals
(1) b + c
(2) a + b
(3) a + b + c
(4) c + a
(2)
2
(a + b) + c (a + b) = a + b
2ab
2r + 2R = c +
( since c2 = a2 + b2).
=
a
b
c
a
b
c
+
+
+
+
(
)
(
)

In a triangle ABC, let C =

33.

If cos1 x cos1

33.

(1) 2 sin 2
(3) 4 sin2
(3)

y
= , then 4x2 4xy cos + y2 is equal to
2
(2) 4
(4) 4 sin2

y
cos-1x cos-1 2 =

xy

y2
cos1
+ 1 x2 1 =
2
4

xy + 4 y 2 4x 2 + x 2 y 2
=
cos1

2
2
2 2
2
4 y 4x + x y = 4 cos + x2y2 4xy cos
4x2 + y2 4xy cos = 4 sin2.

34.

34.

If in a triangle ABC, the altitudes from the vertices A, B, C on opposite sides are in
H.P., then sin A, sin B, sin C are in
(1) G.P.
(2) A.P.
(3) Arithmetic Geometric Progression
(4) H.P.
(2)
1
1
1
= p1a = p2b = p3b
2
2
2
p1, p2, p3 are in H.P.
2 2 2
,
,

are in H.P.
a b c
1 1 1
, , are in H.P
a b c
a, b, c are in A.P.
sinA, sinB, sinC are in A.P.

FIITJEE

Ltd. ICES House, 29-A, Kalu Sarai, Sarvapriya Vihar, New Delhi - 110016, Ph : 26515949, 26569493, Fax : 26513942

11

35.

If I1 = 2x dx, I2 = 2x dx , I3 =

35.

(1) I2 > I1
(3) I3 = I4
(2)

x
x
2 dx and I4 = 2 dx then
2

(2) I1 > I2
(4) I3 > I4
1

I1 = 2 dx , I2 = 2 dx , I3 = 2 dx , I4 =
x2

x3

x2

x3

dx

0 < x < 1, x2 > x3


1

x
2 dx >
2

x3

dx

I1 > I2.
36.
36.

The area enclosed between the curve y = loge (x + e) and the coordinate axes is
(1) 1
(2) 2
(3) 3
(4) 4
(1)
0

Required area (OAB) =

ln ( x + e )dx

1 e

x dx = 1.
= x ln ( x + e )
x+e

0
37.

37.

The parabolas y2 = 4x and x2 = 4y divide the square region bounded by the lines x =
4, y = 4 and the coordinate axes. If S1, S2, S3 are respectively the areas of these
parts numbered from top to bottom; then S1 : S2 : S3 is
(1) 1 : 2 : 1
(2) 1 : 2 : 3
(3) 2 : 1 : 2
(4) 1 : 1 : 1
(4)
y2 = 4x and x2 = 4y are symmetric about line y = x
4
8
2
area bounded between y = 4x and y = x is 2 x x dx =
3
0
16
16
and A s1 = A s3 =
A s2 =
3
3
A s1 : A s2 : A s3 :: 1 : 1 : 1.

38.

38.

dy
= y (log y log x + 1), then the solution of the equation is
dx
x
y
(1) y log = cx
(2) x log = cy
y
x

If x

y
(3) log = cx
x
(3)
x dy
= y (log y log x + 1)
dx
dy y
y
= log + 1
dx x
x
Put y = vx

FIITJEE

x
(4) log = cy
y

Ltd. ICES House, 29-A, Kalu Sarai, Sarvapriya Vihar, New Delhi - 110016, Ph : 26515949, 26569493, Fax : 26513942

12
dy
x dv
=v+
dx
dx
xdv
v+
= v ( log v + 1)
dx
xdv
= v log v
dx
dv
dx

=
v log v
x
put log v = z
1
dv = dz
v
dz dx

=
z
x
ln z = ln x + ln c
z = cx
log v = cx
y
log = cx .
x
39.

39.

The line parallel to the xaxis and passing through the intersection of the lines ax +
2by + 3b = 0 and bx 2ay 3a = 0, where (a, b) (0, 0) is
3
from it
(1) below the xaxis at a distance of
2
2
(2) below the xaxis at a distance of
from it
3
3
(3) above the xaxis at a distance of
from it
2
2
from it
(4) above the xaxis at a distance of
3
(1)
ax + 2by + 3b + (bx 2ay 3a) = 0
(a + b)x + (2b 2a)y + 3b - 3a = 0
a + b = 0 = -a/b
a
(bx 2ay 3a) = 0
ax + 2by + 3b b
2a2
3a2
ax + 2by + 3b ax +
=0
y+
b
b

2a2
3a2
y 2b +
=0
+ 3b +
b
b

2b2 + 2a2
3b2 + 3a2
y
=

b
b

y=

(
2 (b

) = 3
) 2

3 a 2 + b 2

y=

FIITJEE

+a

3
so it is 3/2 units below x-axis.
2

Ltd. ICES House, 29-A, Kalu Sarai, Sarvapriya Vihar, New Delhi - 110016, Ph : 26515949, 26569493, Fax : 26513942

13
40.

40.

A spherical iron ball 10 cm in radius is coated with a layer of ice of uniform thickness
than melts at a rate of 50 cm3/min. When the thickness of ice is 5 cm, then the rate at
which the thickness of ice decreases, is
1
1
cm/min
(2)
cm/min
(1)
36
18
1
5
cm/min
(3)
cm/min
(4)
54
6
(2)
dv
= 50
dt
dr
= 50
4r2
dt
dr
50
=

where r = 15
dt 4 (15 )2
=

1
.
16
2

41.

(log x 1)
(1 + (log x)2 dx is equal to
log x
(1)
+C
(log x)2 + 1
(3)

41.

xe x
+C
1 + x2

(2)

x
+C
x +1

(4)

x
+C
(log x)2 + 1

(4)

(log x 1) dx

2 2
1
+
log
x
(
)
(
)
2

1
2log x

dx
2
2
2
1 + ( log x )
1 + ( log x )

e
2t et

dt put logx = t dx = et dt
=
2
2
1 + t 2

1+ t

1
2t
t

e
1 + t 2 1 + t 2 2 dt

42.

) (

x
et
+c
+c =
2
2
1+ t
1 + ( log x )

1
Let f : R R be a differentiable function having f (2) = 6, f (2) =
. Then
48
f ( x)
4t 3
lim
dt equals
x 2
x2
6
(1) 24
(2) 36
(3) 12
(4) 18

FIITJEE

Ltd. ICES House, 29-A, Kalu Sarai, Sarvapriya Vihar, New Delhi - 110016, Ph : 26515949, 26569493, Fax : 26513942

14
42.

(4)

f ( x)

4t 3
dt
x2 x 2
0
Applying L Hospital rule
2
lim 4f ( x ) f ( x ) = 4f(2)3 f(2)

x 2
1
= 18.
= 4 63
48
lim

43.

43.

44.

44.

Let f (x) be a nonnegative continuous function such that the area bounded by the

and x = >
curve y = f (x), xaxis and the ordinates x =
4
4

is sin + cos + 2 . Then f is


4
2

(1) + 2 1
(2) 2 + 1
4

(3) 1 2
(4) 1 + 2
4
4

(4)

Given that f ( x ) dx = sin + cos + 2


4
/4
Differentiating w. r. t

f() = cos + sin sin + 2


4


f = 1 sin + 2 = 1 + 2 .
4
2
4
2
The locus of a point P (, ) moving under the condition that the line y = x + is a
x2 y2
tangent to the hyperbola 2 2 = 1 is
a
b
(1) an ellipse
(2) a circle
(3) a parabola
(4) a hyperbola
(4)
x2 y2
Tangent to the hyperbola 2 2 = 1 is
a
b
y = mx a2m2 b 2
Given that y = x + is the tangent of hyperbola
m = and a2m2 b2 = 2
a22 b2 = 2
Locus is a2x2 y2= b2 which is hyperbola.

45.

If the angle between the line


0 is such that sin =
(1)

5
3

FIITJEE

x +1 y 1 z 2
=
=
and the plane 2x y +
1
2
2

z+4=

1
the value of is
3
(2)

3
5

Ltd. ICES House, 29-A, Kalu Sarai, Sarvapriya Vihar, New Delhi - 110016, Ph : 26515949, 26569493, Fax : 26513942

15

(3)
45.

3
4

4
3

(4)

(1)
Angle between line and normal to plane is

22+2
where is angle between line & plane
cos =
2
3 5 +

sin =

2
3 5+

1
3

5
= .
3
46.

The angle between the lines 2x = 3y = z and 6x = y = 4z is


(2) 900
(1) 00
0
(4) 300
(3) 45

46.

(2)
Angle between the lines 2x = 3y = - z & 6x = -y = -4z is 90
Since a1a2 + b1b2 + c1c2 = 0.

47.

If the plane 2ax 3ay + 4az + 6 = 0 passes through the midpoint of the line joining
the centres of the spheres
x2 + y2 + z2 + 6x 8y 2z = 13 and
x2 + y2 + z2 10x + 4y 2z = 8, then a equals
(1) 1
(2) 1
(4) 2
(3) 2
(3)
Plane
2ax 3ay + 4az + 6 = 0 passes through the mid point of the centre of spheres
x2 + y2 + z2 + 6x 8y 2z = 13 and x2 + y2 + z2 10x + 4y 2z = 8 respectively
centre of spheres are (-3, 4, 1) & (5, - 2, 1)
Mid point of centre is (1, 1, 1)
Satisfying this in the equation of plane, we get
2a 3a + 4a + 6 = 0 a = -2.

47.

48.

The distance between the line


= 5 is
r (i + 5j + k)

r = 2i 2j + 3k + (i j + 4k)

and the plane

10
10
(2)
9
3 3
3
10
(4)
(3)
10
3
(2)
Distance between the line
r = 2i 2j + 3k + i j + 4k and the plane r i + 5j + k = 5 is
(1)

48.

equation of plane is x + 5y + z = 5
Distance of line from this plane
= perpendicular distance of point (2, -2, 3) from the plane
i.e.

FIITJEE

2 10 + 3 5
1+ 5 + 1
2

10
3 3

Ltd. ICES House, 29-A, Kalu Sarai, Sarvapriya Vihar, New Delhi - 110016, Ph : 26515949, 26569493, Fax : 26513942

16
49.

2 is equal to
For any vector a , the value of (a i)2 + (a j)2 + (a k)

49.

(1) 3a2
(3) 2a2
(3)
Let a = xi + yj + zk
a i = zj yk

a i

(2) a2
(4) 4a2

= y 2 + z2

similarly a j

= x 2 + z2

(a k ) = x + y (a i ) = y + z
similarly ( a j ) = x + z
and ( a k ) = x + y
( a i ) + ( a j ) + ( a k ) = 2 ( x + y + z ) = 2 a
2

and

50.

If non-zero numbers a, b, c are in H.P., then the straight line


passes through a fixed point. That point is
(1) (-1, 2)
(3) (1, -2)

x y 1
+ + = 0 always
a b c

(2) (-1, -2)


1

(4) 1,
2

50.

(3)
a, b, c are in H.P.
2 1 1
=0

b a c
x y 1
+ + =0
a b c
x y 1

= =
x = 1, y = -2
1 2 1

51.

If a vertex of a triangle is (1, 1) and the mid-points of two sides through this vertex
are (-1, 2) and (3, 2), then the centroid of the triangle is
7

1 7
(2) ,
(1) 1,
3

3 3
7
1 7
(4) ,
(3) 1,
3
3 3
(3)
A(1, 1)
Vertex of triangle is (1, 1) and midpoint of sides
through this vertex is (-1, 2) and (3, 2)
vertex B and C come out to be
(-3, 3) and (5, 3)
(-1, 2)
1 3 + 5 1+ 3 + 3
(3, 2)
,
centroid is
3
3
(1, 7/3)

51.

FIITJEE

Ltd. ICES House, 29-A, Kalu Sarai, Sarvapriya Vihar, New Delhi - 110016, Ph : 26515949, 26569493, Fax : 26513942

17

52.

52.

53.

53.

If the circles x2 + y2 + 2ax + cy + a = 0 and x2 + y2 3ax + dy 1 = 0 intersect in two


distinct points P and Q then the line 5x + by a = 0 passes through P and Q for
(1) exactly one value of a
(2) no value of a
(3) infinitely many values of a
(4) exactly two values of a
(2)
S1 = x2 + y2 + 2ax + cy + a = 0
S2 = x2 + y2 3ax + dy 1 = 0
Equation of radical axis of S1 and S2
S1 S2 = 0
5ax + (c d)y + a + 1 = 0
Given that 5x + by a = 0 passes through P and Q
a c d a +1
=
=
1
b
a
2
a + 1 = -a
a2 + a + 1 = 0
No real value of a.
A circle touches the x-axis and also touches the circle with centre at (0, 3) and radius
2. The locus of the centre of the circle is
(1) an ellipse
(2) a circle
(3) a hyperbola
(4) a parabola
(4)
Equation of circle with centre (0, 3) and radius 2 is
x2 + (y 3)2 = 4.
Let locus of the variable circle is (, )
It touches x-axis.
It equation (x - )2 + (y - )2 = 2
Circles touch externally

2 + ( 3 ) = 2 +
2

2 + ( - 3)2 = 2 + 4 + 4
2 = 10( - 1/2)
Locus is x2 = 10(y 1/2) which is parabola.

54.

54.

55.

(, )

If a circle passes through the point (a, b) and cuts the circle x2 + y2 = p2 orthogonally,
then the equation of the locus of its centre is
(1) x2 + y2 3ax 4by + (a2 + b2 p2) = 0 (2) 2ax + 2by (a2 b2 + p2) = 0
(3) x2 + y2 2ax 3by + (a2 b2 p2) = 0 (4) 2ax + 2by (a2 + b2 + p2) = 0
(4)
Let the centre be (, )
It cut the circle x2 + y2 = p2 orthogonally
2(-) 0 + 2(-) 0 = c1 p2
c1 = p2
Let equation of circle is x2 + y2 - 2x - 2y + p2 = 0
It pass through (a, b) a2 + b2 - 2a - 2b + p2 = 0
Locus 2ax + 2by (a2 + b2 + p2) = 0.
An ellipse has OB as semi minor axis, F and F its focii and the angle FBF is a right
angle. Then the eccentricity of the ellipse is
1
1
(2)
(1)
2
2

FIITJEE

Ltd. ICES House, 29-A, Kalu Sarai, Sarvapriya Vihar, New Delhi - 110016, Ph : 26515949, 26569493, Fax : 26513942

18

(3)
55.

1
4

(4)

(1)
FBF = 90o

a2 e2 + b2

a2 e2 + b2

= (2ae)2

2(a2 e2 + b2) = 4a2e2


e2 = b2/a2
Also e2 = 1- b2/a2 = 1 e2

56.

56.

3
B(0, b)

) +(

2e2 = 1, e =

1
2

F(-ae, 0)

F(ae, 0)

i + k and
Let a, b and c be distinct non-negative numbers. If the vectors ai + aj + ck,
ci + cj + bk lie in a plane, then c is
(1) the Geometric Mean of a and b
(2) the Arithmetic Mean of a and b
(3) equal to zero
(4) the Harmonic Mean of a and b
(1)
Vector a i + aj + ck , i + k and ci + cj + bk are coplanar
a a c
1 0 1 = 0 c2 = ab
c c b
a, b, c are in G.P.

57.

If a, b, c are non-coplanar vectors and is a real number then

57.

a + b 2b c = a b + c b for


(1) exactly one value of
(3) exactly three values of
(2)
a + b 2b c = a b + c b


0 1 0 0

(2) no value of
(4) exactly two values of

0 2
0 0

0 =0 1 1
0 1 0

4 = -1
Hence no real value of .

58.

58.

b = xi + j + (1 x ) k and c = yi + xj + (1 + x y ) k . Then a, b, c
Let a = i k,

depends on
(1) only y
(2) only x
(3) both x and y
(4) neither x nor y
(4)
a = i k , b = xi + j + (1 x ) k and c = yi + xj + (1 + x y ) k

a b c = a b c

FIITJEE

Ltd. ICES House, 29-A, Kalu Sarai, Sarvapriya Vihar, New Delhi - 110016, Ph : 26515949, 26569493, Fax : 26513942

19
i

bc = x 1
1 x = i (1 + x x x2) - j (x + x2- xy y + xy) + k (x2 y)
y x 1+ x y

a. b c = 1

which does not depend on x and y.


59.

59.

60.

60.

Three houses are available in a locality. Three persons apply for the houses. Each
applies for one house without consulting others. The probability that all the three
apply for the same house is
2
1
(1)
(2)
9
9
8
7
(3)
(4)
9
9
(2)
For a particular house being selected
1
Probability =
3
1
1 1 1
Prob(all the persons apply for the same house) = 3 = .
9
3 3 3
A random variable X has Poisson distribution with mean 2. Then P(X > 1.5) equals
2
(1) 2
(2) 0
e
3
3
(3) 1 2
(4) 2
e
e
(3)
k
P(x = k) = e
k!
P(x 2) = 1 P(x = 0) P(x = 1)

= 1 e- e- 1!

=1-

61.

61.

3
.
e2

1
1
1
, P ( A B ) = and P A = ,
6
4
4
where A stands for complement of event A. Then events A and B are
(1) equally likely and mutually exclusive
(2) equally likely but not independent
(3) independent but not equally likely
(4) mutually exclusive and independent
(3)
1
1
1
P A B = , P(A B) =
and P A =
6
4
4
P(A B) = 5/6 P(A) = 3/4
Also P(A B) = P(A) + P(B) P(A B)
P(B) = 5/6 3/4 + 1/4 = 1/3
P(A) P(B) = 3/4 1/3 = 1/4 = P(A B)
Let A and B be two events such that P A B =

FIITJEE

( )

( )

Ltd. ICES House, 29-A, Kalu Sarai, Sarvapriya Vihar, New Delhi - 110016, Ph : 26515949, 26569493, Fax : 26513942

20
Hence A and B are independent but not equally likely.
62.

62.

63.

63.

A lizard, at an initial distance of 21 cm behind an insect, moves from rest with an


acceleration of 2 cm/s2 and pursues the insect which is crawling uniformly along a
straight line at a speed of 20 cm/s. Then the lizard will catch the insect after
(1) 20 s
(2) 1 s
(3) 21 s
(4) 24 s
(3)
1 2
2t = 21 + 20t
2
t = 21.
Two points A and B move from rest along a straight line with constant acceleration f
and f respectively. If A takes m sec. more than B and describes n units more than B
in acquiring the same speed then
(1) (f - f)m2 = ffn
(2) (f + f)m2 = ffn
1
1
(3) ( f + f ) m = ff n2
(4) ( f f ) n = ff m2
2
2
(4)
v2 = 2f(d + n) = 2fd
v = f(t) = (m + t)f
eliminate d and m we get
1
(f - f)n = ff m2 .
2

64.

A and B are two like parallel forces. A couple of moment H lies in the plane of A and
B and is contained with them. The resultant of A and B after combining is displaced
through a distance
2H
H
(2)
(1)
A B
A +B
H
H
(3)
(4)
A B
2 ( A + B)

64.

(2)
(A + B) = d = H
H
d=
.
A +B

65.

The resultant R of two forces acting on a particle is at right angles to one of them and
its magnitude is one third of the other force. The ratio of larger force to smaller one is
(1) 2 : 1
(2) 3 : 2

65.

(3) 3 : 2
(4)
F = 3F cos
F = 3F sin
F = 2 2 F

(4) 3 : 2 2
F
3F

F : F : : 3 : 2 2 .
F

FIITJEE

Ltd. ICES House, 29-A, Kalu Sarai, Sarvapriya Vihar, New Delhi - 110016, Ph : 26515949, 26569493, Fax : 26513942

21

66.

The sum of the series 1 +


(1)
(3)

e 1
e
e 1
2 e

1
1
1
+
+
+ ......... ad inf. is
4.2! 16.4! 64.6!
e +1
(2)
e
e +1
(4)
2 e

66.

(4)
e x + e x
x2 x4 x6
= 1+
+
+
+ .......
2
2! 4! 6!
putting x = 1/2 we get
e +1
.
2 e

67.

The value of

cos2 x
dx, a > 0, is
x

1 + a

(1) a
(3)

(2)

(4) 2

67.

(2)

cos2 x

2
=
dx
1 + ax
0 cos x dx = 2 .

68.

The plane x + 2y z = 4 cuts the sphere x2 + y2 + z2 x + z 2 = 0 in a circle of


radius
(1) 3
(2) 1
(3) 2
(4) 2
(2)
1
1
Perpendicular distance of centre , 0, from x + 2y 2 = 4
2
2

68.

1 1
+ 4
3
2 2
=
2
6
5 3
= 1.
radius =
2 2
69.

69.

If the pair of lines ax2 + 2(a + b)xy + by2 = 0 lie along diameters of a circle and divide
the circle into four sectors such that the area of one of the sectors is thrice the area
of another sector then
(1) 3a2 10ab + 3b2 = 0
(2) 3a2 2ab + 3b2 = 0
2
2
(3) 3a + 10ab + 3b = 0
(4) 3a2 + 2ab + 3b2 = 0
(4)
2

(a + b)

a+b

ab

=1

(a + b)2 = 4(a2 + b2 + ab)


3a2 + 3b2 + 2ab = 0.

FIITJEE

Ltd. ICES House, 29-A, Kalu Sarai, Sarvapriya Vihar, New Delhi - 110016, Ph : 26515949, 26569493, Fax : 26513942

22
70.

Let x1, x2, ,xn be n observations such that

70.

possible value of n among the following is


(1) 15
(3) 9
(2)

xi

n
n
n 16.

71.

71.

2
i

(2) 18
(4) 12

A particle is projected from a point O with velocity u at an angle of 60o with the
horizontal. When it is moving in a direction at right angles to its direction at O, its
velocity then is given by
u
u
(1)
(2)
3
2
2u
u
(4)
(3)
3
3
(4)
u cos 60o = v cos 30o
4
.
v=
30o
3
o

60

72.

72.

73.

73.

74.

= 400 and xi = 80 . Then a

30o

If both the roots of the quadratic equation x2 2kx + k2 + k 5 = 0 are less than 5,
then k lies in the interval
(1) (5, 6]
(2) (6, )
(3) (-, 4)
(4) [4, 5]
(3)
b
<5
2a
f(5) > 0
k(-, 4).
If a1, a2, a3,, an, are in G.P., then the determinant
logan logan+1 logan+ 2
= logan+ 3 logan+ 4 logan+ 5 is equal to
logan + 6 logan + 7 logan+ 8
(1) 1
(3) 4
(2)
C1 C2, C2 C3
two rows becomes identical
Answer: 0.

(2) 0
(4) 2

A real valued function f(x) satisfies the functional equation f(x y) = f(x) f(y) f(a x)
f(a + y) where a is a given constant and f(0) = 1, f(2a x) is equal to
(1) f(x)
(2) f(x)
(3) f(a) + f(a x)
(4) f(-x)

FIITJEE

Ltd. ICES House, 29-A, Kalu Sarai, Sarvapriya Vihar, New Delhi - 110016, Ph : 26515949, 26569493, Fax : 26513942

23
74.

(1)
f(a (x a)) = f(a) f(x a) f(0) f(x)
= -f(x) x = 0, y = 0, f ( 0 ) = f 2 ( 0 ) f 2 ( a ) f 2 ( a ) = 0 f ( a ) = 0 .

75.

If the equation
an xn + an1x n1 + ...... + a1x = 0 , a1 0, n 2, has a positive root x = , then the
equation nan x n1 + (n 1) an1x n 2 + ..... + a1 = 0 has a positive root, which is

75.

(1) greater than


(3) greater than or equal to
(2)
f(0) = 0, f() = 0
f(k) = 0 for some k(0, ).

FIITJEE

(2) smaller than


(4) equal to

Ltd. ICES House, 29-A, Kalu Sarai, Sarvapriya Vihar, New Delhi - 110016, Ph : 26515949, 26569493, Fax : 26513942

FIITJEE Solutions to AIEEE2006

MATHEMATICS
PART A

Ans.

ABC is a triangle, right angled at A. The resultant of the forces acting along AB, AC
1
1
with magnitudes
and
respectively is the force along AD , where D is the
AB
AC
foot of the perpendicular from A onto BC. The magnitude of the resultant is
AB2 + AC2
(AB)(AC)
(2)
(1)
2
2
AB + AC
(AB) (AC)
1
1
1
+
(4)
(3)
AB AC
AD
(4)

Sol:

Magnitude of resultant

1.

AB2 + AC2
1 1
+
=
AB AC
AB AC

BC
BC
1
=
=
=
AB AC AD BC AD

2.

Suppose a population A has 100 observations 101, 102, , 200, and another
population B has 100 observations 151, 152, , 250. If VA and VB represent the
V
variances of the two populations, respectively, then A is
VB
(1) 1
(2) 9/4
(3) 4/9
(4) 2/3

Ans.

(1)

Sol:

2x =

2
i

. (Here deviations are taken from the mean)

Since A and B both has 100 consecutive integers, therefore both have same
standard deviation and hence the variance.

( d

VA
= 1 As
VB

2
i

is same in both the cases .

3.

If the roots of the quadratic equation x2 + px + q = 0 are tan30 and tan15,


respectively then the value of 2 + q p is
(3) 2
(2) 3
(3) 0
(4) 1

Ans.
Sol:

(2)
x2 + px + q = 0
tan 30 + tan 15 = p
tan 30 tan 15 = q

FIITJEE Ltd., ICES House, 29 A, Kalu Sarai, Sarvapriya Vihar, New Delhi -110016, Ph 26515949, 26569493, Fax: 011-26513942.

FIITJEE Solutions to AIEEE2006


tan 45 =

p
tan 30 + tan15
=
=1
1 tan 30 tan15 1 q

p=1q
q p = 1 2 + q p = 3.
6

4.

The value of the integral,

x
9x + x

dx is

(1) 1/2
(3) 2
Ans.

(2)

Sol:

I=

(2) 3/2
(4) 1

9x + x

9x + x

3
6

I=

9x

dx

dx

2I =

dx = 3 I =
3

3
.
2

5.

The number of values of x in the interval [0, 3] satisfying the equation


2sin2x + 5sinx 3 = 0 is
(1) 4
(2) 6
(3) 1
(4) 2

Ans.
Sol:

(1)
2 sin2 x + 5 sin x 3 = 0
(sin x + 3) (2 sin x 1) = 0
sin x =

6.

1
2

In (0, 3), x has 4 values

If a b c = a b c , where a, b and c are any three vectors such that a b 0 ,

b c 0 , then a and c are


(1) inclined at an angle of /3 between them
(2) inclined at an angle of /6 between them
(3) perpendicular
(4) parallel
Ans.
Sol:

(4)

( a b ) c = a ( b c ) , a b 0, b c 0
(a c ) b (b c ) a = (a c ) b (a b ) c
(a b) c = (b c ) a
a c

7.

Let W denote the words in the English dictionary. Define the relation R by :

FIITJEE Ltd., ICES House, 29 A, Kalu Sarai, Sarvapriya Vihar, New Delhi -110016, Ph 26515949, 26569493, Fax: 011-26513942.

FIITJEE Solutions to AIEEE2006


R = {(x, y) W W | the words x and y have at least one letter in common}. Then R
is
(1) not reflexive, symmetric and transitive
(2) reflexive, symmetric and not transitive
(3) reflexive, symmetric and transitive
(4) reflexive, not symmetric and transitive
Ans.
Sol:

(2)
Clearly (x, x) R x W. So, R is reflexive.
Let (x, y) R, then (y, x) R as x and y have at least one letter in common. So, R is
symmetric.
But R is not transitive for example
Let x = DELHI, y = DWARKA and z = PARK
then (x, y) R and (y, z) R but (x, z) R.

8.

If A and B are square matrices of size n n such that A2 B2 = (A B) (A + B), then


which of the following will be always true ?
(1) A = B
(2) AB = BA
(3) either of A or B is a zero matrix
(4) either of A or B is an identity matrix

Ans.
Sol:

(2)
A2 B2 = (A B) (A + B)
A2 B2 = A2 + AB BA B2
AB = BA.

9.

The value of

10

sin
k =1

2k
2k
is
+ icos
11
11

(1) i
(3) 1
Ans.

(4)
10

Sol:

(2) 1
(4) i

2k
2k

sin 11 + i cos 11 =

k =1

10

k =1

10

sin

2k
2k
+ i cos
11
11
k =1

= 0 + i ( 1) = i.
10.

All the values of m for which both roots of the equations x2 2mx + m2 1 = 0 are
greater than 2 but less than 4, lie in the interval
(2) m > 3
(1) 2 < m < 0
(3) 1 < m < 3
(4) 1 < m < 4

Ans.
Sol:

(3)
Equation x2 2mx + m2 1 = 0
(x m)2 1 = 0
(x m + 1) (x m 1) = 0
x = m 1, m + 1
2 < m 1 and m + 1 < 4

FIITJEE Ltd., ICES House, 29 A, Kalu Sarai, Sarvapriya Vihar, New Delhi -110016, Ph 26515949, 26569493, Fax: 011-26513942.

FIITJEE Solutions to AIEEE2006


m > 1 and m < 3
1 < m < 3.
11.

A particle has two velocities of equal magnitude inclined to each other at an angle .
If one of them is halved, the angle between the other and the original resultant
velocity is bisected by the new resultant. Then is
(1) 90
(2) 120
(3) 45
(4) 60

Ans.

(2)

Sol:

u
sin

tan = 2
4 u + u cos
2
1

1
sin + sin cos = sin cos
4 2
4
2
4

3
3
2 sin = sin
= 3 sin 4 sin
4
4
4
4

sin2 =
= 30 or = 120.
4 4
4

R1

R2

u
/4 /4
/2
u/2

12.

At a telephone enquiry system the number of phone cells regarding relevant enquiry
follow Poisson distribution with an average of 5 phone calls during 10-minute time
intervals. The probability that there is at the most one phone call during a 10-minute
time period is
6
5
(2)
(1) e
6
5
6
6
(4) 5
(3)
55
e

Ans.

(4)

Sol:

P (X = r) =

e m mr
r!

P (X 1) = P (X = 0) + P (X = 1)
= e5 + 5 e5 =

6
e5

13.

A body falling from rest under gravity passes a certain point P. It was at a distance of
400 m from P, 4s prior to passing through P. If g = 10 m/s2, then the height above the
point P from where the body began to fall is
(1) 720 m
(2) 900 m
(3) 320 m
(4) 680 m

Ans.

(1)

FIITJEE Ltd., ICES House, 29 A, Kalu Sarai, Sarvapriya Vihar, New Delhi -110016, Ph 26515949, 26569493, Fax: 011-26513942.

FIITJEE Solutions to AIEEE2006


Sol:

1 2
1
gt and h + 400 = g(t + 4)2 .
2
2
Subtracting we get 400 = 8g + 4gt
t = 8 sec
1
h = 10 64 = 320m
2
Desired height = 320 + 400 = 720 m.

We have h =

h
Q(t)
400m
P(t+4)

14.

xf(sin x)dx is equal to


0

(1) f(cos x)dx

(2) f(sin x)dx

0
/2

(3)
Ans.
Sol:

0
/2

f(sin x)dx

(4) f(cos x)dx

(4)
I=

xf(sin x) dx = ( x) f(sin x) dx

= f(sin x) dx I
0

2I = f(sin x) dx
0

I=

f(sin x) dx =
2

0
/2

/2

f(sin x) dx
0

f(cos x)dx .
0

15.

A straight line through the point A(3, 4) is such that its intercept between the axes is
bisected at A. Its equation is
(1) x + y = 7
(2) 3x 4y + 7 = 0
(3) 4x + 3y = 24
(4) 3x + 4y = 25

Ans.
Sol:

(3)
The equation of axes is xy = 0
the equation of the line is
x4 + y3
= 12 4x + 3y = 24.
2

16.

The two lines x = ay + b, z = cy + d; and x = ay + b, z = cy + d are perpendicular to


each other if
(1) aa + cc = 1
(2) aa + cc = 1
a c
a c
(3)
+ =1
+ = 1
(4)
a c
a c

FIITJEE Ltd., ICES House, 29 A, Kalu Sarai, Sarvapriya Vihar, New Delhi -110016, Ph 26515949, 26569493, Fax: 011-26513942.

FIITJEE Solutions to AIEEE2006


Ans.

(1)

Sol:

Equation of lines

x b
zd
=y=
a
c

x b
z d
=y=
a
c

Lines are perpendicular aa + 1 + cc = 0.


17.

The locus of the vertices of the family of parabolas y =


105
64
35
(3) xy =
16

3
4
64
(4) xy =
105

(!) xy =

Ans.

(1)

Sol:

Parabola: y =

a3 x 2 a 2 x
+
2a is
3
2

(2) xy =

a3 x 2 a 2 x
+
2a
3
2

Vertex: (, )
a4

a3
1 8

+ 4
2a
+ a4
4

2
3
a / 2
3

4 3
=
=
,=
=
3
3
4 3
4a
2a / 3
a
a
4
3
3
35 a
35
3 =
a
=
12 4
16
3 35
105

a=
=
.
4a 16
64

18.

The values of a, for which the points A, B, C with position vectors


i 3j 5k and ai 3j + k respectively are the vertices of a right-angled
2i j + k,

triangle with C = are


2
(1) 2 and 1
(2) 2 and 1
(3) 2 and 1
(4) 2 and 1

Ans.
Sol:

(1)
BA = i 2j + 6k
CA = (2 a)i + 2j
CB = (1 a)i 6k

CA CB = 0 (2 a) (1 a) = 0
a = 2, 1.

FIITJEE Ltd., ICES House, 29 A, Kalu Sarai, Sarvapriya Vihar, New Delhi -110016, Ph 26515949, 26569493, Fax: 011-26513942.

FIITJEE Solutions to AIEEE2006


/ 2

( x + )

19.

3 / 2

+ cos2 ( x + 3 ) dx is equal to

4
32

(3)
2

4
+
32 2

(4) 1
4

(2)

(1)

Ans.

(3)

Sol:

I=

/ 2

3 / 2

(x + )3 + cos2 (x + 3) dx

Put x + = t
/2

I=

/ 2
/2

/2

t 3 + cos2 t dt = 2

cos

t dt

(1 + cos 2t) dt = 2 + 0 .
0

20.

If x is real, the maximum value of


(1) 1/4
(3) 1

Ans.

(2)

Sol:

y=

3x 2 + 9x + 17
is
3x 2 + 9x + 7
(2) 41
(4) 17/7

3x 2 + 9x + 17
3x 2 + 9x + 7

3x2(y 1) + 9x(y 1) + 7y 17 = 0
D 0 x is real
81(y 1)2 4x3 ( y 1)( 7y 17 ) 0

(y 1) (y 41) 0 1 y 41.

21.

In an ellipse, the distance between its foci is 6 and minor axis is 8. Then its
eccentricity is
3
1
(B)
(1)
5
2
4
1
(C)
(D)
5
5

Ans.
Sol:

(1)
2ae = 6 ae = 3
2b = 8 b = 4
b2 = a2(1 e2)
16 = a2 a2e2
a2 = 16 + 9 = 25
a=5
e =

3 3
=
a 5

FIITJEE Ltd., ICES House, 29 A, Kalu Sarai, Sarvapriya Vihar, New Delhi -110016, Ph 26515949, 26569493, Fax: 011-26513942.

FIITJEE Solutions to AIEEE2006


22.

Ans.
Sol:

1 2
a 0
Let A =
and B =
, a , b N. Then
3 4
0 b
(1) there cannot exist any B such that AB = BA
(2) there exist more than one but finite number of Bs such that AB = BA
(3) there exists exactly one B such that AB = BA
(4) there exist infinitely many Bs such that AB = BA

(4)
1 2
A=
B=

3 4
a 2b
AB =

3a 4b

a 0
0 b

a 0 1 2 a 2a
BA =

0 b 3 4 3b 4b

AB = BA only when a = b
23.

The function f(x) =


(1) x = 2
(3) x = 0

x 2
+ has a local minimum at
2 x
(2) x = 2
(4) x = 1

Ans.

(1)

Sol:

x 2
1
+ is of the form x + 2 & equality holds for x = 1
2 x
x

24.

Angle between the tangents to the curve y = x2 5x + 6 at the points (2, 0) and (3, 0)
is

(1)
(2)
2
2

(4)
(3)
6
4

Ans.

(2)
dy
= 2x 5
dx
m1 = (2x 5)(2, 0) = 1, m2 = (2x 5)(3, 0) = 1
m1m2 = 1

Sol:

25.

Let a1, a2, a3, be terms of an A.P. If


41
11
2
(3)
7
(1)

Ans.

a1 + a2 + ap
a1 + a2 + + aq

a
p2
, p q , then 6 equals
2
a21
q

7
2
11
(4)
41

(2)

(4)

FIITJEE Ltd., ICES House, 29 A, Kalu Sarai, Sarvapriya Vihar, New Delhi -110016, Ph 26515949, 26569493, Fax: 011-26513942.

FIITJEE Solutions to AIEEE2006


Sol:

26.

p
2a + ( p 1) d
2a + ( p 1) d p
p2
2 1
= 2 1
=
q
2a1 + ( q 1) d q
2a1 + ( q 1) d q
2
p 1
a1 +
d
2 =p
q
q 1
a1 +
d
2

a6
a
11
For
, p = 11, q = 41 6 =
a21
a21
41

The set of points where f(x) =


(1) (, 0) (0, )
(3) (, )

Ans.

(3)

Sol:

x
1 x ,
f (x) =
x ,
1 + x

x<0
x0

x
is differentiable is
1+ | x |
(2) (, 1) (1, )
(4) (0, )

1
, x<0

2
(1 x)
f (x) =
1 , x0
2

(1 + x )

f(x) exist at everywhere.


27.

A triangular park is enclosed on two sides by a fence and on the third side by a
straight river bank. The two sides having fence are of same length x. The maximum
area enclosed by the park is
3 2
x
2
1
(3) x 2
2

(2)

(1)

Ans.

(3)

Sol:

Area =
A max

1 2
x sin
2
1
= x 2 at sin = 1,
2

x3
8

(4) x2

28.

At an election, a voter may vote for any number of candidates, not greater than the
number to be elected. There are 10 candidates and 4 are of be elected. If a voter
votes for at least one candidate, then the number of ways in which he can vote is
(1) 5040
(2) 6210
(3) 385
(4) 1110

Ans.
Sol:

(3)
10
C1 + 10C2 + 10C3 + 10C4
= 10 + 45 + 120 + 210 = 385

FIITJEE Ltd., ICES House, 29 A, Kalu Sarai, Sarvapriya Vihar, New Delhi -110016, Ph 26515949, 26569493, Fax: 011-26513942.

FIITJEE Solutions to AIEEE2006


29.

If the expansion in powers of x of the function


a0 + a1x + a2x2 + a3x3 + , then an is
bn an
(1)
ba
an+1 bn +1
(3)
ba

Ans.
Sol:

1
is
(1 ax)(1 bx)

an bn
ba
bn+1 an +1
(4)
ba
(2)

(4)

(1 ax )1 (1 bx )1 = (1 + ax + a2 x 2 + ......)(1 + bx + b2 x2 + ....)

coefficient of xn = bn + abn1 + a2bn2 + .... + an1b + an =


an =

bn+1 an +1
ba

bn+1 an+1
ba

30.

For natural numbers m, n if (1 y)m (1 + y)n = 1 + a1y + a2y2 + , and a1 = a2 = 10,


then (m, n) is
(1) (20, 45)
(2) (35, 20)
(3) (45, 35)
(4) (35, 45)

Ans.

(4)

Sol:

(1 y )m (1 + y )n = 1 m C1y +m C2 y 2 .... 1 +n C1y +n C2 y2 + ...


m ( m 1) n ( n 1)

+
mn y 2 + .....
= 1 + (n m) +
2

a1 = n m = 10 and a2 =

m + n m n 2mn
= 10
2

So, n m = 10 and (m n)2 (m + n) = 20


m = 35, n = 45

m + n = 80

31.

The value of [x] f (x) dx , a > 1, where [x] denotes the greatest integer not exceeding
1

x is
(1) af(a) {f(1) + f(2) + + f([a])}
(3) [a] f([a]) {f(1) + f(2) + + f(a)}
Ans.
Sol:

(2) [a] f(a) {f(1) + f(2) + + f([a])}


(4) af([a]) {f(1) + f(2) + + f(a)}

(2)
Let a = k + h, where [a] = k and 0 h < 1

k +h

k 1

[ x] f ' ( x ) dx = 1f ' ( x )dx + 2f ' ( x ) dx + ........ (k 1) dx +

kf ' ( x ) dx

{f(2) f(1)} + 2{f(3) f(2)} + 3{f(4) f(3)}+.+ (k1) {f(k) f(k 1)}
+ k{f(k + h) f(k)}
= f(1) f(2) f(3). f(k) + k f(k + h)
= [a] f(a) {f(1) + f(2) + f(3) + . + f([a])}

FIITJEE Ltd., ICES House, 29 A, Kalu Sarai, Sarvapriya Vihar, New Delhi -110016, Ph 26515949, 26569493, Fax: 011-26513942.

FIITJEE Solutions to AIEEE2006


32.

If the lines 3x 4y 7 = 0 and 2x 3y 5 = 0 are two diameters of a circle of area


49 square units, the equation of the circle is
(1) x2 + y2 + 2x 2y 47 = 0
(2) x2 + y2 + 2x 2y 62 = 0
2
2
(4) x2 + y2 2x + 2y 47 = 0
(3) x + y 2x + 2y 62 = 0

Ans.
Sol:

(4)
Point of intersection of 3x 4y 7 = 0 and 2x 3y 5 = 0 is (1 , 1), which is the
centre of the circle and radius = 7.
Equation is (x 1)2 + (y + 1)2 = 49 x2 + y2 2x + 2y 47 = 0.

33.

The differential equation whose solution is Ax2 + By2 = 1, where A and B are arbitrary
constants is of
(1) second order and second degree
(2) first order and second degree
(3) first order and first degree
(4) second order and first degree

Ans.
Sol:

(4)
Ax 2 + By 2 = 1
Ax + By
A + By

(1)

dy
=0
dx

(2)
2

d2 y

dy
+ B = 0
2
dx
dx

(3)

From (2) and (3)


2

d2 y
dy
dy
x By 2 B + By
=0
dx
dx
dx

xy

d2 y

dy
dy
+ x y
=0
dx
dx
dx 2

34.

Let C be the circle with centre (0, 0) and radius 3 units. The equation of the locus of
2
at its centre
the mid points of the chords of the circle C that subtend an angle of
3
is
3
(1) x 2 + y 2 =
(B) x2 + y2 = 1
2
27
9
(3) x 2 + y 2 =
(D) x 2 + y 2 =
4
4

Ans.

(4)

Sol:

cos

35.

If (a, a2) falls inside the angle made by the lines y =

=
3

h2 + k 2
3

belongs to
1
(1) 0,
2
1
(3) , 3
2

h2 + k 2 =

9
4

x
, x > 0 and y = 3x, x > 0, then a
2

(2) (3, )
1

(4) 3,
2

FIITJEE Ltd., ICES House, 29 A, Kalu Sarai, Sarvapriya Vihar, New Delhi -110016, Ph 26515949, 26569493, Fax: 011-26513942.

FIITJEE Solutions to AIEEE2006


Ans.

(3)

Sol:

a2 3a < 0 and a2

36.

The image of the point (1, 3, 4) in the plane x 2y = 0 is


17 19
(1) , , 4
(2) (15, 11, 4)
3
3

17 19
(3) , , 1
(4) (8, 4, 4)
3
3

Sol:

If (, , ) be the image then

a
1
>0 <a<3
2
2

1
+3
2
=0
2
2

1 2 6 2 = 7

(1)

+1 3 4
=
=
and
1
2
0

(2)

From (1) and (2)


=

9
13
, = , =4
5
5

No option matches.
37.

If z2 + z + 1 = 0, where z is a complex number, then the value of


2
2
2
2
1 2 1 3 1

6 1
z + z + z + z 2 + z + z3 + + z + z6 is

(1) 18
(2) 54
(3) 6
(4) 12

Ans.
Sol:

(4)
z2 + z + 1 = 0

z = or 2

1
1
1
= + 2 = 1, z2 + 2 = 2 + = 1, z3 + 3 = 3 + 3 = 2
z
z
z
1
1
1
z 4 + 4 = 1, z5 + 5 = 1 and z6 + 6 = 2
z
z
z

so, z +

The given sum = 1 + 1 + 4 + 1 + 1 + 4 = 12


38.

If 0 < x < and cosx + sinx =


(1 7 )
4
(4 + 7 )
(3)
3
(1)

Ans.
Sol:

1
, then tanx is
2
(4 7)
3
(1 + 7 )
(4)
4

(B)

(3)
1
1
3
1 + sin2x =
sin2x = , so x is obtuse
2
4
4
2 tan x
3
2
and
=
3 tan x + 8 tan x + 3 = 0
4
1 + tan2 x
cos x + sin x =

FIITJEE Ltd., ICES House, 29 A, Kalu Sarai, Sarvapriya Vihar, New Delhi -110016, Ph 26515949, 26569493, Fax: 011-26513942.

FIITJEE Solutions to AIEEE2006


tan x =

8 64 36 4 7
=
6
3

tan x < 0

tan x =

4 7
3

39.

If a1, a2, , an are in H.P., then the expression a1a2 + a2a3 + + an1an is equal to
(1) n(a1 an)
(2) (n 1) (a1 an)
(4) (n 1)a1an
(3) na1an

Ans.

(4)

Sol:

1
1
1
1
1
1

= ..... =

= d (say)
a2 a1 a3 a2
an an1

a a3
a1 a2
a an
, a 2 a3 = 2
,......., an1an = n1
d
d
d
a1 an
1
1
a1a2 + a2a3 + ....... + an1an =
Also,
=
+ ( n 1) d
d
an a1

Then a1a2 =

40.

a1 an
= ( n 1) a1an
d

If xm yn = (x + y)m +n , then
(1)

x+y
xy
x
(4)
y

y
x

(2)

(3) xy
Ans.
Sol:

dy
is
dx

(1)
m+n

xm .yn = ( x + y )

mln x + nln y = ( m + n ) ln ( x + y )

m m + n m + n n dy
m n dy m + n
dy
+
=
1+


=

x y dx x + y
dx
x x + y x + y y dx
my nx my nx dy
dy y

x ( x + y ) y ( x + y ) dx
dx x

FIITJEE Ltd., ICES House, 29 A, Kalu Sarai, Sarvapriya Vihar, New Delhi -110016, Ph 26515949, 26569493, Fax: 011-26513942.

Code : N
Mathematics
1.

If (2, 3, 5) is one end of a diameter of the sphere x 2 + y 2 + z2 6x 12y 2z + 20 = 0, then the


coordinates of the other end of the diameter are
(1) (4, 3, 3)
(2) (4, 9, 3)
(3) (4, 3, 3)
(4) (4, 3, 5)

Sol. (2)
Centre of sphere is (3, 6, 1)
Let other end is (x1, y1, z1)

x1 + 2
= 3 x1 = 4
2

y1 + 3
= 6 y1 = 9
2
z1 + 5
= 1 z1 = 3
2
Other end of diameter is (4, 9, 3)
2.

b = i j + 2k and c = xi + ( x 2 ) j k,
. If the vector c lies in the plane of a and
Let a = i + j + k,
b , then x equals
(1) 2

(2) 0

(3) 1

(4) 4

Sol. (1)

H H
H
3 c, a and b are coplanar
H
H
H
c = a + b

) (

xi + ( x 2 ) j k = i + j + k + i j + 2k

x = +

... (i)

x2 =

... (ii)

1 = + 2
... (iii)
From (i) and (ii)
= x 1, = 1
From (iii) 1 = x 1 + 2
x = 2

Career Launcher 8 12 / Solutions AIEEE Test Paper 2007

Page 1

Code : N
Method II

1
1
1
1 1
2 =0
x x 2 1

1(1 2x + 4 ) 1( 1 2x ) + 1( x 2 + x ) = 0
2x = 4
x = 2
3.

Let A(h, k), B(1, 1) and C(2, 1) be the vertices of a right angled triangle with AC as its hypotenuse.
If the area of the triangle is 1, then the set of values which k can take is given by
(1) {3, 2}
(2) {1, 3}
(3) {0, 2}
(4) {1, 3}

Sol. (4)
=1

h k 1
1
1 1 1 =1
2
2 1 1

h (1 1) k (1 2 ) + 1(1 2 ) = 2
k 1 = 2
k = 3 or 1
4.

Let P = (1, 0), Q = (0, 0) and R = 3, 3 3 be three points. The equation of the bisector of the
angle PQR is
(1) x + 3y = 0

(2)

3x + y = 0

(3) x +

3
y=0
2

(4)

3
x+y =0
2

Sol. (2)

Y
P (3, 3 3)

/ 3

P (1, 0) Q

/3
/3

Career Launcher 8 12 / Solutions AIEEE Test Paper 2007

Page 2

Code : N
Slope of QS, m = tan120 = 3

y = 3x
y + 3x = 0

5.

2
2
2
If one of the lines of my + 1 m xy mx = 0 is a bisector of the angle between the lines

xy = 0, then m is
2.

(1) 2

1
2

(3) 2

(4) 1

Sol. (4)
Joint equation of bisector of the lines xy = 0 is y 2 x 2 = 0

2
2
2
Since my + 1 m xy mx = 0

( y mx )(my + x ) = 0
One of the line is bisector of xy = 0
m=1
x

6.

logt
1
dt. Then F(e) equals
Let F(x) = f(x) + f , where f ( x ) =
1+ t
x
1
(1) 2

(2)

1
2

(3) 0

(4) 1

Sol. (2)

1
F(e) = f(e) + f
e
=

1/ e

logt
1 + t dt +

logt
dt
1+ t

= I1 + I2
1/ e

For I2 =

logt
dt
1+ t

1
1
dt = dz
z
z2
When t = 1 z = 1
Let t =

Career Launcher 8 12 / Solutions AIEEE Test Paper 2007

Page 3

Code : N
1
z=e
e

t=

logz 1
.
dz
1 z2
1 1+
z

I2 =

logz
log t
z (1 + z ) dz = t (1 + t ) dt

F(e) = I1 + I2
e
logt
log t
dt
=
+
1 + t t (1 + t )

log t
t dt = s ds

1
s = log t; ds = dt, when t = 1. s = 0 and t = e. s = 1
t

s2
1
= =
2 0 2
7.

Let f : R IR be a function defined by f(x) = Min{x + 1, |x| + 1}. Then which of the following is
true?
(1) f(x) is not differentiable at x = 0
(2) f(x) 1 for all x R
(3) f(x) is not differentiable at x = 1.
(4) f(x) is differentiable everywhere

Sol. (4)

f(x) = M in{x + 1, |x| + 1}


Y

+1
O

Career Launcher 8 12 / Solutions AIEEE Test Paper 2007

Page 4

Code : N
f(x) = x + 1
x0
x+1
x<0
f(x) is differentiable everywhere.
8.

The function f : R {0} R given by

1
2

2x
x e 1
can be made continuous at x = 0 by defining f(0) as
(1) 1
(2) 2
(3) 1
f (x ) =

(4) 0

Sol. (1)
1
2
lim

2x
x 0 x e
1

= lim

x 0

e2x 1 2x

x e

= lim

x 0 e2x

= lim

2x

0
0

2e2x 2
1 + x.2e

x 0 2e2x

0
0

2x

4e2x
+ 2e2x + 4xe2x

4
=1
4
f(0) = 1
=

9.

The solution for x of the equation

(1) 2 2
Sol.

(2) 2

dt
t t2 1

2 is
(3)

(4)

3
2

Wrong Question

Career Launcher 8 12 / Solutions AIEEE Test Paper 2007

Page 5

Code : N
10.

dx

cos x +

3 sin x

equals

x
(1) log tan + C
2 12
(3)

1
x
log tan + C
2
2 12

(2)

1
x
log tan + + C
2
2 12

x
(4) log tan + + C
2 12

Sol. (2)

I=

1
dx

2 1
3
cos x +
sin x
2
2

1
2

dx

sin x cos + cos x sin


6
6

1
2

dx

sin x +
6

cos ec x + dx

2
6

1
x
logtan + + C
2
2 12

Career Launcher 8 12 / Solutions AIEEE Test Paper 2007

Page 6

Code : N
11.

The area enclosed between the curves y 2 = x and y =| x | is


(1)

1
3

(2)

2
3

(3) 1

(4)

1
6

Sol. (4)

y=x

y = x

(1,1)

X'

O (0, 0)

y =x
Y'
1

Area =

x x dx

x3 / 2 x 2
=

2
3
2
0
=

12.

2 1 1
=
sq. units
3 2 6

If the difference between the roots of the equation x 2 + ax + 1 = 0 is less than


of possible values of a is
(1) ( , 3 )

(2) ( 3, 3 )

(3) ( 3, )

5 , then the set

(4) (3, )

Sol.

3 < 5
Again + = a, = 1

( ) < 5
2

Career Launcher 8 12 / Solutions AIEEE Test Paper 2007

Page 7

Code : N
( + ) 4 < 5
2

a2 4 < 5
a2 < 9

a ( 3, 3 )

... (i)

Also D 0
a2 4 0
a ( , 2 ) (2, )

... (ii)

From (i) and (ii)

a ( 3, 2 ) (2, 3 )
13.

In a geometric progression consisting of positive terms, each term equals the sum of the next two
terms. Then the common ratio of this progression equals
(1)

1
2

5 1

(2)

1
1 5
2

(3)

1
5
2

(4)

Sol. (1)
Let the GP be a, ar, ar2, ar3...
a = ar + ar2
1 = r + r2
r2 + r 1 = 0

r =
r =

14.

1 1 + 4
2
5 1
2

(3 GP has positive terms)

1 x
1 5
If sin + cos ec = then a value of x is
5

4 2
(1) 5
(2) 1
(3) 3

(4) 4

Sol. (3)

x
4
sin1 + sin1 =
5

5 2
x
3
sin1 + cos1 =
5
5 2

x 3
=
x=3
5 5

Career Launcher 8 12 / Solutions AIEEE Test Paper 2007

Page 8

Code : N
15.

In the binomial expansion of (a b )n , n 5, the sum of 5th and 6th term is zero, then
(1)

n4
5

(2)

5
n4

(3)

6
n5

(4)

a
equals
b

n5
6

Sol. (1)

T5 + T6 = 0
n

C4 an 4 .b4 nC5 an5 b5 = 0

16.

an 4 b4
n 4

C5

C4

4! (n 4 )! n 4
a
n!
=

=
b 5! (n 5 )!
n!
5

The set S : = {1, 2, 3..., 12} is to be partitioned into three sets A, B and C of equal size. Thus,

A B C = S, A C = B C = A C = . The numbers of ways to partition S is


12!

(1)

(3!)

12!

(2)

12!

3! ( 4!)

(3)

12!

3! (3!)

(4)

( 4!)3

Sol. (4)
12!

Total number of ways =

17.

( 4!)

3!

3! =

12!

( 4!)3


The largest interval lying in , for which the function
2 2

x
x2
+ cos1 1 + log (cos x )
f (x ) = 4
2

is defined, is


(1) 0,
2
Sol. (1)

(2) [0, ]


(3) ,
2 2


(4) ,
4 2

2
x
f(x) = 4 x + cos1 1 + log (cos x )
2

Career Launcher 8 12 / Solutions AIEEE Test Paper 2007

Page 9

Code : N
For 4 x

x R

... (i)

1 x
For cos 1
2
1

x
x
1 1 0 2
2
2

0x4
For log(cosx)

... (ii)

<x<
2
2
From (i), (ii) and (iii)
cosx > 0

... (iii)


x 0,
2
18.

A body weighing 13 kg is suspended by two strings 5 m and 12 m long, their other ends being
fastened to the extremities of a rod 13 m long. If the rod be so held that the body hands immediately
below the middle point. The tension in the strings are
(1) 5 kg and 13 kg
(2) 12 kg and 13 kg
(3) 5 kg and 5 kg
(4) 5 kg and 12 kg

Sol. (4)

13 m
G

A
90
T2

90
5m

T1

12m

13 kg

Career Launcher 8 12 / Solutions AIEEE Test Paper 2007

Page 10

Code : N
132 = 52 + 122 AB2 = AC2 + BC2 ACB = 90
3 G is mid-point of hypotenuse AB.
GA = GB = GC GC = 6.5m
Let GBC = , then, GCB =
By Lamis theorem
T1
T2
13
=
=
sin (180 ) sin (90 + ) sin90
T1
T
13
= 2 =
sin cos
1

T1 = 13 sin and T2 = 13cos


T1 = 13

5
12
and T2 = 13
13
13

as sin =

5
12
, cos =
13
13

T1 = 5 kg and T2 = 12 kg
19.

A pair of fair dice is thrown independently three times. The probability of getting a score of exactly
9 twice is
(1)

8
243

(2)

1
729

(3)

8
9

(4)

8
729

Sol. (1)
Probability of getting exactly 9 is

1
9

and probability of not getting 9 is 1

1 8
=
9 9

1 8
Required probability = C2
9 9
3

3! 1 8

2! 81 9

68
8
=
2 81 9 243

Career Launcher 8 12 / Solutions AIEEE Test Paper 2007

Page 11

Code : N
20.

Consider a family of circles which are passing through the point (1, 1) and are tangent to x-axis.
If (h, k) are the coordinates of the centre of the circles, then the set of values of k is given by the
interval
(1) k

1
2

(2) 0 < k <

1
2

(3) k

1
2

(4)

1
1
k
2
2

Sol. (3)

(1, 1 )

C (h, k)
k

(h + 1)2 + (k 1)2 = k 2
h2 + 2h + 1 + k 2 2k + 1 = k 2
h2 + 2h + (2 2k ) = 0

D0
4 4(2 2k) 0
4 8 + 8k 0
8k 4
k
21.

1
2

Let L be the line of intersection of the planes 2x + 3y + z = 1 and x + 3y + 2z = 2. If L makes an


angle with the positive x-axis, then cos equals.
(a)

1
2

(2)

1
3

(3)

1
2

(4) 1

Sol. (2)
Given planes are

2x + 3y + z 1 = 0

... (i)

... (ii)
x + 3y + 2x 2 = 0
Let l, m, n be the direction cosines of line of intersection of plane (i) and (ii).
... (iii)
2l + 3m + n = 0
l + 3m + 2n = 0

... (iv)

Career Launcher 8 12 / Solutions AIEEE Test Paper 2007

Page 12

Code : N
Solving (iii) and (iv), we get
m = l, n = l

l2 + m2 + n2 = 1

3l2 = 1 l =
22.

1
3

The differential equation of all circles passing through the origin and having their centres on the
x-axis is
(1) y 2 = x 2 2xy

dy
dx

(3) x 2 = y 2 + 3xy dy
dx
Sol. (4)
General equation of circle

(2) x 2 = y 2 + xy

dy
dx

(4) y 2 = x 2 + 2xy

dy
dx

x 2 + y 2 + 2gx + 2fy + c = 0
As centre is on x-axis, f = 0
As circle is passing through origin, c = 0
Equation of required circle will be
... (i)
x 2 + y 2 + 2gx = 0
Differentiating w.r.t. x, we get
dy
+ 2g = 0
... (ii)
dx
Eliminating g from (i) and (ii)
2x + 2y

dy

x 2 + y 2 + x 2x 2y = 0
dx

y 2 = x 2 + 2xy

dy
dx

Career Launcher 8 12 / Solutions AIEEE Test Paper 2007

Page 13

Code : N
23.

If p and q are positive real numbers such that p2 + q2 = 1, then the maximum value of (p + q) is
(1)

(2) 2

(3)

1
2

(4)

1
2

Sol. (1)
... (i)
Given p2 + q2 = 1
From (i), we can say 0 p 1 and 0 q 1
Put p = sin
q = cos
p + q = sin + cos
Maximum value of sin + cos =
Maximum value of p + q =
24.

A tower stands at the centre of a circular park. A and B are two points on the boundary of the park
such that AB (= a) subtends an angle of 60 at the foot of the tower, and the angle of elevation of
the top of the tower from A or B is 30. The height of the tower is
(2)

(1) a 3

2a
3

(3) 2a 3

(4)

a
3

Sol. (4)

P
h

30
60
30

A
a

B
OA = OB = radii
In OAB, OA = OB = AB = a
In POB
tan30 =

h
a

Career Launcher 8 12 / Solutions AIEEE Test Paper 2007

Page 14

Code : N

1
3

h=
25.

h
a

a
3

The sum of the series


20C 20C + 20C 20C + + 20C is
0
1
2
3
10
(1) 20C10

(2)

20C
10

(3)

1 20
C10
2

(4) 0

Sol. (3)
Given series is
x = 20C0 20C1 + 20C2 20C3 + + 20C10
2x = 2 20C0 2 20C1 + 2 20C2 + + 2 20C10

( 20 C0 +20 C20 ) ( 20 C1 +20 C19 ) + ( 20 C2 +20 C18 ) + ... + ( 20 C10 + 20C10 )


2x = ( 20 C0 20 C1 + 20 C2 + ... + 20 C10 + ... + 20 C18 20 C19 + 20C20 ) + 20C10
=

As

20

C0 20 C1 + 20 C2 + ... + 20 C20 = 0

2x =20 C10 x =
26.

1 20
C10
2

The normal to a curve at P(x, y) meets the x-axis at G. If the distance of G from the origin is twice
the abscissa of P, then the curve is a
(1) hyperbola
(2) ellipse
(3) parabola
(4) circle

Sol. (1, 2)
Let y = f(x) be a curve

dy
= slope of tangent
dx

dx
= slope of normal
dy

Equation of normal
Yy=

dx
(X x)
dx

dy

G x + y , 0
dx

Career Launcher 8 12 / Solutions AIEEE Test Paper 2007

Page 15

Code : N
Given x + y
y

dy
= 2x
dx

dy
dy
= 3x
= x ydy = xdx or y =
dx
dx

y2 x2
=
+ K or ydy = 3xdx
2
2

y2
x2
y2
3x 2
=
+ K or
+
=K
2
2
2/3 2
Curve is hyperbola or ellipse.
y 2 x 2 = K1 or

27.

If z + 4 3, then the maximum value of z + 1 is


(1) 0

(2) 4

(3) 10

(4) 6

Sol. (4)
Given z + 4 3
z + 1 = z + 4 + ( 3 ) z + 4 + 3

z +1 z + 4 + 3
z +1 3 + 3
z +1 6
Maximum value of z + 1 is 6.
28.

The resultant of two forces P N and 3 N is a force of 7 N. If the direction of the 3 N force were
reversed, the resultant would be
(1) 4 N

(2) 5 N

19 N . The value of P is
(3) 6 N

(4) 3 N

Sol. (2)

F2

F1
KKH
F1 = PN
Career Launcher 8 12 / Solutions AIEEE Test Paper 2007

Page 16

Code : N
KKH
F2 = 3N

R1 = 32 + P2 + 6Pcos = 7
cos =

40 P2
6P

... (i)

F2

F 2

F
1

R2 = 9 + P2 + 6P cos ( ) = 19
P2 6P cos = 10

P2 40 P2 = 10

[(from i)]

2P2 = 50
P = 5N

29.

Two aeroplanes I and II bomb a target in succession. The probabilities of I and II scoring a hit
correctly are 0.3 and 0.2, respectively. The second plane will bomb only if the first misses the
target. The probability that the target is hit by the second plane is
(1) 0.7
(2) 0.06
(3) 0.14
(4) 0.2

Sol. (3)
Let A is the event of the plane I hit the target correctly.
B is the event of the plane II hit the target correctly.
P(A) = .3
P(B) = .2

( )
P (Bc ) = .8

P A c = .7

( )

c
Probability that the target is hit by the second plane = P A .P (B ) = .7 .2 = .14

Assume second plane hit the target only one time.

Career Launcher 8 12 / Solutions AIEEE Test Paper 2007

Page 17

Code : N
30.

1
1
1
1 for x 0, y 0, then D is
If D = 1 1 + x
1
1
1+ y
(1) divisible by y but not x
(3) divisible by both x and y

(2) divisible by neither x nor y


(4) divisible by x but not y

Sol. (3)

1 1
1
D = 1 1+ x
1
1 1 1+ y
C1 C1 C2 , C2 C2 C3

0 0
1
D = x x
1 = xy
0 y 1 + y
D is divisible by both x and y.

31.

For the hyperbola

x2

cos2
(1) Abscissae of foci
(3) Directrix

y2
sin2

= 1, which of the following remains constant when varies?


(2) Eccentricity
(4) Abscissae of vertices

Sol. (1)

x2
cos2

y2
sin2

=1

sin2 = cos2 e2 1

cos2 e2 = 1 ecos = 1
Here, a = cos, b = sin
Abscissae of foci = ae = ecos = 1
e=

1
cos

(depends on )

a
= cos2
e
Abscissae of vertices = a = cos
x=

Career Launcher 8 12 / Solutions AIEEE Test Paper 2007

Page 18

Code : N
32.

with the positive directions of each of x-axis and y-axis, then the
4
angle that the line makes with the positive direction of z-axis is

If a line makes an angle of

(1)

(2)

(3)

(4)

Sol. (1)

l = cos

=
4
2

m = cos

=
4
2

l2 + m2 + n2 = 1

1 1
+ + n2 = 1
2 2

n2 = 0 n = 0
cos = 0 =
33.

A value of C for which the conclusion of Mean Value Theorem holds for the function f(x) = loge x on
the interval [1, 3] is
(1) loge 3

(2) 2 log3 e

(3)

1
loge 3
2

(4) log3 e

Sol. (2)
Given function f(x) = loge x
Mean Value Theorem for [1, 3]
f '(c) =

f(3) f(1)
3 1

1 loge 3 loge 1 1
=
= loge 3
c
2
2

c=

2
= 2log3 e
loge 3

Career Launcher 8 12 / Solutions AIEEE Test Paper 2007

Page 19

Code : N
34.

The function f(x) = tan1 (sin x + cos x ) is an increasing function in


(1) ,
2 2
Sol. (3)


(2) ,
4 2


(3) ,
2 4


(4) 0,
2

f(x) = tan1 (sin x + cos x )

Let Z = sinx + cosx


f(x) = tan1(Z)
f(x) is increasing only when Z increases

Z = 2 sin x +
4

From options Z increases only when

35.

<x<
2
4

5 5
Let A = 0 5

0 0
5
2
If A = 25, then || equals

(2) 52

(1) 5

(3) 1

(4)

1
5

Sol. (4)

5 5
A = 0 5
0 0
5

A 2 = 25 A

= 25

A = 5
5 5
A = 0 5 = 5 (5 0 ) = 25
0 0
5
25 = 5 =
=

1
5

1
5

Career Launcher 8 12 / Solutions AIEEE Test Paper 2007

Page 20

Code : N
36.

The sum of the series

(1) e

1 1 1
+ ... up to infinity is
2! 3! 4!

1
2

(2) e 2

(3) e 1

(4) e

1
2

Sol. (3)
1 1 1
+ ...
2! 3! 4!
ex = 1 +

x x 2 x3
+
+
+ ...
1! 2! 3!

e 1 = 1

1 1 1 1
+ + ...
1! 2! 3! 4!

1 1 1
+ + ... = e1
2! 3! 4!

37.

If u and v are unit vectors and is the acute angle between them, then 2u 3 v is a unit vector for
(1) Exactly one value of
(2) Exactly two values of
(3) More than two values of
(4) No value of

Sol. (1)

u
2u 3v

= 6 u v sin n
Where n unit vector perpendicular to u and v

= 2u 3v = 6 sin n
6 sin = 1 sin =

1
6

Here is one and only one value of between 0 and 90 for which sin =

Career Launcher 8 12 / Solutions AIEEE Test Paper 2007

1
6

Page 21

Code : N
38.

A particle just clears a wall of height b at a distance a and strikes the ground at a distance c from
the point projection. The angle of projection is
(1) tan1

bc
a

(2) tan1

b
ac

(3) 45

1
(4) tan

bc
a (c a )

Sol. (4)

S (a,b )

u
L

b
a

T
c

M (c, 0)

Let u = velocity of projection


= angle of projection

x2
2u cos2
As M is on the trajectory.
y = x tan

0 = c tan

tan =

g
2

2u cos
g
2

2u cos2

b = a tan

c ... (i)

g
2

2u cos
From (i) and (ii), we get
tan =

a2

... (ii)

bc
a (c a )

= tan1

39.

c2

bc
a (c a )

The average marks of boys in a class is 52 and that of girls is 42. The average marks of boys and
girls combined is 50. The percentage of boys in the class is
(1) 60
(2) 40
(3) 20
(4) 80

Sol. (4)
Let x = Number of boys in the class
y = Number of girls in the class
Sum of marks of all boys = 52x
Career Launcher 8 12 / Solutions AIEEE Test Paper 2007

Page 22

Code : N
Sum of marks of all girls = 42y
Average of boys and girls combined marks = 50

50 =

52x + 42y
x+y

x = 4y
Percentage of boys in the class =

40.

x
4y
100 =
100 = 80%
x+y
5y

The equation of a tangent to the parabola y 2 = 8x is y = x + 2. The point on this line from which
the other tangent to the parabola is perpendicular to the given tangent is
(1) (2, 0)
(2) (1, 1)
(3) (0, 2)
(4) (2, 4)

Sol. (1)
Given parabola is y2 = 8x

Given tangent y = x + 2 ...(i)


As second tangent is perpendicular to (i), so that pair is on the directrix as directrix is the
director circle.
Equation of direcrix x = 2 ...(ii)
Solving (i) and (ii), we get (2, 0)

Career Launcher 8 12 / Solutions AIEEE Test Paper 2007

Page 23

FIITJEE Solutions to AIEEE - 2008

AIEEE2008, PAPER(C5)
Note: (i) The test is of 3 hours duration.
(ii) The test consists of 105 questions of 3 marks each. The maximum marks are 315.
(iii) There are three parts in the question paper. The distribution of marks subjectwise in each part is as under for each correct
response.
Part A Mathematics (105 marks) 35 Questions
Part B Chemistry (105 marks) 35 Questions
Part C Physics (105 marks) 35 Questions
(iv) Candidates will be awarded three marks each for indicated correct response of each question. One mark will be deducted
for indicated incorrect response of each question. No deduction from the total score will be made if no response is indicated for
an item in the Answer Sheet.

Mathematics
PART A
1.

AB is a vertical pole with B at the ground level and A at the top. A man finds that the angle of
elevation of the point A from a certain point C on the ground is 60. He moves away from the pole
along the line BC to a point D such that CD = 7 m. From D the angle of elevation of the point A is 45.
Then the height of the pole is
7 3
1
7 3
m
(2)
( 3 + 1) m
(1)

2
2
3 1
(3)

Sol:

7 3
( 3 1) m
2

45
7

It is given that the events A and B are such that P (A) =


1
6
2
(3)
3

(1)

Sol:

7 3
1

2
3 +1

(2)
BD = AB = 7 + x
Also AB = x tan 60 = x 3
x 3 =7+x
7
x=
3 1

7 3
( 3 + 1) .
AB =
2
2.

(4)

60
C

1
B 2
A 1
, P = and P = . Then P (B) is
B 2
A 3
4

1
3
1
(4)
2

(2)

(2)
P ( A B) 1 P ( A B) 2
= ,
=
2
3
P (B )
P (A)
P (A) 3
Hence
(But P (A) = 1/4)
= .
P (B ) 4
1
P (B ) = .
3
FIITJEE Ltd., ICES House, 29 A, Kalu Sarai, Sarvapriya Vihar, New Delhi -110016, Ph 26515949, 26569493, Fax: 011-26513942.

FIITJEE Solutions to AIEEE - 2008

3.

A die is thrown. Let A be the event that the number obtained is greater than 3. Let B be the event that
the number obtained is less than 5. Then P (A B) is
3
(1)
(2) 0
5
2
(3) 1
(4)
5

Sol:

(3)
A = {4, 5, 6} , B = {1, 2, 3, 4} .
Obviously P (A B) = 1.

4.

A focus of an ellipse is at the origin. The directrix is the line x = 4 and the eccentricity is 1/2. Then the
length of the semimajor axis is
8
2
(1)
(2)
3
3
4
5
(3)
(4)
3
3

Sol:

(1)
Major axis is along x-axis.
a
ae = 4
e
1

a2 = 4

2
8
a= .
3

5.

A parabola has the origin as its focus and the line x = 2 as the directrix. Then the vertex of the
parabola is at
(1) (0, 2)
(2) (1, 0)
(3) (0, 1)
(4) (2, 0)

Sol:

(2)
Vertex is (1, 0)

X =2

(2, 0)

6.

The point diametrically opposite to the point P (1, 0) on the circle x2 + y2 + 2x + 4y 3 = 0 is


(1) (3, 4)
(2) ( 3, 4)
(3) ( 3, 4)
(4) (3, 4)

Sol:

(3)
Centre ( 1, 2)
Let (, ) is the required point
+1
+0
= 2 .
= 1 and
2
2

7.

Let f : N Y be a function defined as f (x) = 4x + 3, where Y = {y N : y = 4x + 3 for some x N}.


Show that f is invertible and its inverse is
3y + 4
y+3
(1) g (y) =
(2) g (y) = 4 +
4
3
y+3
y3
(4) g (y) =
(3) g (y) =
4
4
FIITJEE Ltd., ICES House, 29 A, Kalu Sarai, Sarvapriya Vihar, New Delhi -110016, Ph 26515949, 26569493, Fax: 011-26513942.

FIITJEE Solutions to AIEEE - 2008


Sol:

(4)
Function is increasing
y3
= g(y) .
x=
4

8.

The conjugate of a complex number is


1
i 1
1
(3)
i+1

(1)

1
. Then the complex number is
i 1
1
(2)
i+1
1
(4)
i 1

Sol:

(3)
Put i in place of i
1
.
Hence
i+1

9.

Let R be the real line. Consider the following subsets of the plane R R.
S = {(x, y) : y = x + 1 and 0 < x < 2}, T = {(x, y) : x y is an integer}. Which one of the following is true?
(1) neither S nor T is an equivalence relation on R
(2) both S and T are equivalence relations on R
(3) S is an equivalence relation on R but T is not
(4) T is an equivalence relation on R but S is not

Sol:

(4)
T = {(x, y) : xy I}
as 0 I T is a reflexive relation.
If x y I y x I
T is symmetrical also
If x y = I1 and y z = I2
Then x z = (x y) + (y z) = I1 + I2 I
T is also transitive.
Hence T is an equivalence relation.
Clearly x x + 1 (x, x) S
S is not reflexive.

10.

The perpendicular bisector of the line segment joining P (1, 4) and Q (k, 3) has yintercept 4. Then
a possible value of k is
(1) 1
(2) 2
(3) 2
(4) 4

Sol:

(4)
Slope of bisector = k 1
k +1 7
Middle point =
,
2
2
Equation of bisector is
( k + 1)

7
y
= (k 1) x

2
Put x = 0 and y = 4.
k = 4.

11.

The solution of the differential equation


(1) y = ln x + x
(3) y = xe(x1)

Sol:

dy x + y
=
satisfying the condition y (1) = 1 is
dx
x
(2) y = x ln x + x2
(4) y = x ln x + x

(4)
y = vx
FIITJEE Ltd., ICES House, 29 A, Kalu Sarai, Sarvapriya Vihar, New Delhi -110016, Ph 26515949, 26569493, Fax: 011-26513942.

FIITJEE Solutions to AIEEE - 2008

dy
dv
=v+x
dx
dx
dv
v+ x
= 1+ v
dx
dx
dv =
x
v = log x + c
y
= log x + c
x
Since, y (1) = 1, we have
y = x log x + x

12.

The mean of the numbers a, b, 8, 5, 10 is 6 and the variance is 6.80. Then which one of the following
gives possible values of a and b?
(1) a = 0, b = 7
(2) a = 5, b = 2
(3) a = 1, b = 6
(4) a = 3, b = 4

Sol:

(4)
Mean of a, b, 8, 5, 10 is 6
a + b + 8 + 5 + 10

=6
5
a+b=7
Given that Variance is 6.8

Variance =
2

(X A)

(1)

n
2

( a 6 ) + ( b 6 ) + 4 + 1 + 16
= 6.8
5
a2 + b2 = 25
a2 + (7 a)2 = 25
(Using (1))
a2 7a + 12 = 0
a = 4, 3 and b = 3, 4.
=

13.

Sol:

G
G
G
The vector a = i + 2j + k lies in the plane of the vectors b = i + j and c = j + k and bisects the
G
G
angle between b and c . Then which one of the following gives possible values of and ?
(1) = 2, = 2
(2) = 1, = 2
(4) = 1, = 1
(3) = 2, = 1

(4)
G
a = ( b + c )
i + 2j + k
i + 2j + k =

= 2 and = 2 and =
= 1 and = 1.

14.

Sol:

G
G
G G
G
G
G
G
The nonzero verctors a, b and c are related by a = 8b and c = 7b . Then the angle between a
G
and c is
(1) 0
(2) /4
(3) /2
(4)

(4)
G
G
Since a = 8b
G
G
c = 7b
G
G
G
G
a and b are like vectors and b and c are unlike.
G
G
a and c will be unlike
FIITJEE Ltd., ICES House, 29 A, Kalu Sarai, Sarvapriya Vihar, New Delhi -110016, Ph 26515949, 26569493, Fax: 011-26513942.

FIITJEE Solutions to AIEEE - 2008

G
G
Hence, angle between a and c = .

15.

The line passing through the points (5, 1, a) and (3, b, 1) crosses the yzplane at the point
17 13
,
0,
. Then
2
2

(1) a = 2, b = 8
(2) a = 4, b = 6
(3) a = 6, b = 4
(4) a = 8, b = 2

Sol:

(3)
Equation of line passing through (5, 1, a) and (3, b, 1) is
x 5 y 1 z a
=
=
=.
2
1 b a 1
If line crosses yzplane i.e., x = 0
x = 2 + 5 = 0
= 5/2,
17
Since, y = (1 b) + 1 =
2
5
17
(1 b ) + 1 =
2
2
b = 4.
13
Also, z = (a 1) + a =
2
5
13
( a 1) + a =
2
2
a = 6.

16.

If the straight lines

x 1 y 2 z 3
x 2 y 3 z 1
=
=
=
=
and
intersect at a point, then the
k
2
3
3
k
2

integer k is equal to
(1) 5
(3) 2
Sol:

(2) 5
(4) 2

(1)
x 1 y 2 z 3
x 2 y 3 z 1
and
=
=
=
=
k
2
3
3
k
2
Since lines intersect in a point
k 2 3
3 k 2 =0
1 1 2
2k2 + 5k 25 = 0
k = 5, 5/2.

Directions: Questions number 17 to 21 are AssertionReason type questions. Each of these questions
contains two statements : Statement 1 (Assertion) and Statement2 (Reason). Each of these questions also
has four alternative choices, only one of which is the correct answer. You have to select the correct choice.

17.

Statement 1: For every natural number n 2,

1
1

1
2

+ ... +

1
n

> n.

Statement 2: For every natural number n 2, n ( n + 1) < n + 1 .


(1) Statement 1 is false, Statement 2 is true
(2) Statement 1 is true, Statement 2 is true, Statement 2 is a correct explanation for Statement 1
(3) Statement 1 is true, Statement 2 is true; Statement 2 is not a correct explanation for
Statement 1.
(4) Statement 1 is true, Statement 2 is false.
Sol:

(3)
FIITJEE Ltd., ICES House, 29 A, Kalu Sarai, Sarvapriya Vihar, New Delhi -110016, Ph 26515949, 26569493, Fax: 011-26513942.

FIITJEE Solutions to AIEEE - 2008


1

P (n) =

1
1

P (2) =

+
+

+ ... +

2
1

1
n

> 2

Let us assume that P (k) =


P (k + 1) =

L.H.S. >
Since

1
1

k+

1
k +1

+ ... +

k +1

1
1

+ ... +

2
+

1
k +1

1
k

> k is true

> k + 1 has to be true.

k +1

( k 0)

k +1

>

k ( k + 1) + 1

k ( k + 1) > k

k ( k + 1) + 1

k +1

= k +1

Let P (n) = n ( n + 1) < n + 1


Statement 1 is correct.
P (2) = 2 3 < 3
If P (k) = k ( k + 1) < (k + 1) is true
Now P (k + 1) = ( k + 1) ( k + 2 ) < k + 2 has to be true
Since (k + 1) < k + 2
( k + 1) ( k + 2 ) < ( k + 2 )
Hence Statement 2 is not a correct explanation of Statement 1.
18.

Let A be a 2 2 matrix with real entries. Let I be the 2 2 identity matrix. Denote by tr (A), the sum of
diagonal entries of A. Assume that A2 = I.
Statement 1: If A I and A I, then det A = 1.
Statement 2: If A I and A I, then tr (A) 0.
(1) Statement 1 is false, Statement 2 is true
(2) Statement 1 is true, Statement 2 is true, Statement 2 is a correct explanation for Statement 1
(3) Statement 1 is true, Statement 2 is true; Statement 2 is not a correct explanation for
Statement 1.
(4) Statement 1 is true, Statement 2 is false.

Sol:

(4)
a b
2
Let A =
so that A =
c
d

a2 + bc ab + bd 1 0

2
ac + dc bc + d 0 1

a2 + bc = 1 = bc + d2 and (a + d)c = 0 = (a + d)b.


Since A I, A 1, a = d and hence detA =

1 bc

1 bc

= 1 + bc bc = 1

Statement 1 is true.
But tr. A = 0 and hence statement 2 is false.
n

19.

Statement 1:

(r + 1)

Cr = ( n + 2 ) 2n1 .

Cr xr = (1 + x ) + nx (1 + x )

r =0
n

Statement 2:

(r + 1)

n 1

r =0

(1) Statement 1 is false, Statement 2 is true


(2) Statement 1 is true, Statement 2 is true, Statement 2 is a correct explanation for Statement 1
(3) Statement 1 is true, Statement 2 is true; Statement 2 is not a correct explanation for
Statement 1.
(4) Statement 1 is true, Statement 2 is false.

FIITJEE Ltd., ICES House, 29 A, Kalu Sarai, Sarvapriya Vihar, New Delhi -110016, Ph 26515949, 26569493, Fax: 011-26513942.

FIITJEE Solutions to AIEEE - 2008


Sol:

(2)
n

(r + 1)

r =0

Cr =

Cr + n Cr

r =0

r =0

n n1
n
Cr 1 +
Cr = n 2n1 + 2n
r
r =0

= 2n1 (n + 2)
Statement 1 is true
( r + 1) n Cr xr =
r n Cr xr +

= n

n 1

Cr 1 xr +

r =0

Cr xr

Cr xr = nx (1 + x)n1 + (1 + x)n

r =0

Substituting x = 1
( r + 1) n Cr = n 2n 1 + 2n

Hence Statement 2 is also true and is a correct explanation of Statement 1.


20.

Let p be the statement x is an irrational number, q be the statement y is a transcendental number,


and r be the statement x is a rational number iff y is a transcendental number.
Statement 1: r is equivalent to either q or p
Statement 2: r is equivalent to (p q).
(1) Statement 1 is false, Statement 2 is true
(2) Statement 1 is true, Statement 2 is true, Statement 2 is a correct explanation for Statement 1
(3) Statement 1 is true, Statement 2 is true; Statement 2 is not a correct explanation for
Statement 1.
(4) Statement 1 is true, Statement 2 is false.

Sol:

(4)
Given statement r = p q
Statement 1 : r1 = (p q) ( p q)
Statement 2 : r2 = (p q) = (p q) ( q p)
From the truth table of r, r1 and r2,
r = r1.
Hence Statement 1 is true and Statement 2 is false.

21.

In a shop there are five types of ice-creams available. A child buys six ice-creams.
Statement -1: The number of different ways the child can buy the six ice-creams is 10C5.
Statement -2: The number of different ways the child can buy the six ice-creams is equal to the
number of different ways of arranging 6 As and 4 Bs in a row.
(1) Statement 1 is false, Statement 2 is true
(2) Statement 1 is true, Statement 2 is true, Statement 2 is a correct explanation for Statement 1
(3) Statement 1 is true, Statement 2 is true; Statement 2 is not a correct explanation for
Statement 1.
(4) Statement 1 is true, Statement 2 is false.

Sol:

(1)
x1 + x2 + x3 + x4 + x5 = 6
5+61
C5 1 = 10C4.

22.

Sol:

1
if x 1
( x 1) sin
,
. Then which one of the following is true?
Let f(x) =
x 1
0,
if x = 1

(1) f is neither differentiable at x = 0 nor at x = 1 (2) f is differentiable at x = 0 and at x = 1


(3) f is differentiable at x = 0 but not at x = 1
(4) f is differentiable at x = 1 but not at x = 0

(1)

f (1 + h ) f (1)
h0
h

f(1) = lim

FIITJEE Ltd., ICES House, 29 A, Kalu Sarai, Sarvapriya Vihar, New Delhi -110016, Ph 26515949, 26569493, Fax: 011-26513942.

FIITJEE Solutions to AIEEE - 2008


(1 + h 1) sin 1 0
h
1
1 + h 1
= lim sin
f(1) = lim
h 0
h 0 h
h
h
1
f(1) = lim sin
h0
h
f is not differentiable at x = 1.
f (h) f ( 0 )
Similarly, f(0) = lim
h 0
h
( h 1) sin 1 sin (1)
h 1
f(0) = lim
h0
h
f is also not differentiable at x = 0.
23.

The first two terms of a geometric progression add up to 12. The sum of the third and the fourth terms
is 48. If the terms of the geometric progression are alternately positive and negative, then the first
term is
(1) 4
(2) 12
(3) 12
(4) 4

Sol:

(2)
Let a, ar, ar2,
a + ar = 12
ar2 + ar3= 48
dividing (2) by (1), we have
ar 2 (1 + r )
=4
a ( r + 1)
r2 = 4 if r 1
r=2
also, a = 12 (using (1)).

24.

Suppose the cube x3 px + q has three distinct real roots where p > 0 and q > 0. Then which one of
the following holds?
p
p
and maxima at
(1) The cubic has minima at
3
3
(2) The cubic has minima at

Sol:

p
and maxima at
3

(3) The cubic has minima at both

p
and
3

p
3

(4) The cubic has maxima at both

p
and
3

p
3

(1)
Let f(x) = x3 px + q
Now for maxima/minima
f(x) = 0
3x2 p = 0
p
x2 =
3

x=
25.

(1)
(2)

p
3

(p/3)
(p/3)

p
.
3

How many real solutions does the equation x7 + 14x5 + 16x3 + 30x 560 = 0 have?
(1) 7
(2) 1
(3) 3
(4) 5

FIITJEE Ltd., ICES House, 29 A, Kalu Sarai, Sarvapriya Vihar, New Delhi -110016, Ph 26515949, 26569493, Fax: 011-26513942.

FIITJEE Solutions to AIEEE - 2008


Sol:

(2)
x7 + 14x5 + 16x3 + 30x 560 = 0
Let f(x) = x7 + 14x5 + 16x3 + 30x
f(x) = 7x6 + 70x4 + 48x2 + 30 > 0 x.
f (x) is an increasing function x.

26.

The statement p (q p) is equivalent to


(1) p (p q)
(3) p (p q)

Sol:

(2)
p (q p) = ~ p (q p)
= ~ p (~ q p) since p ~ p is always true
= ~ p p q = p (p q).

27.

Sol:

5
2

The value of cot cos ec 1 + tan1 is


3
3

6
(1)
17
4
(3)
17

(2) p (p q)
(4) p (p q)

3
17
5
(4)
17

(2)

(1)
5
2

Let E = cot cos ec 1 + tan1


3
3

3
2

E = cot tan1 + tan1


4
3

3 2
+

1
E = cot tan 4 3

1 3 2

4 3

17 6

.
E = cot tan1 =
6 17

28.

The differential equation of the family of circles with fixed radius 5 units and centre on the line y = 2 is
(1) (x 2)y2 = 25 (y 2)2
(2) (y 2)y2 = 25 (y 2)2
(3) (y 2)2y2 = 25 (y 2)2
(4) (x 2)2y2 = 25 (y 2)2

Sol:

(3)
(x h)2 + (y 2)2 = 25
dy
=0
2(x h) + 2(y 2)
dx
dy
(x h) = (y 2)
dx
substituting in (1), we have

(1)

( y 2 )2

dy
2
+ ( y 2 ) = 25
dx
(y 2)2y2 = 25 (y 2)2.
1

29.

Let I =

sin x
x

dx and J =

2
and J > 2
3
2
(3) I <
and J > 2
3

(1) I >

cos x
x

dx . Then which one of the following is true?


2
and J < 2
3
2
(4) I >
and J < 2
3

(2) I <

FIITJEE Ltd., ICES House, 29 A, Kalu Sarai, Sarvapriya Vihar, New Delhi -110016, Ph 26515949, 26569493, Fax: 011-26513942.

10

FIITJEE Solutions to AIEEE - 2008


Sol:

(2)
1

I=

sin x
x

dx <

dx =

xdx =

2 3/2
2
x
=
3
3
0

2
I<
3
1

J=

cos x
x

dx <

dx = 2 x 0 = 2

J 2.
30.

The area of the plane region bounded by the curves x + 2y2 = 0 and x + 3y2 = 1 is equal to
5
1
(2)
(1)
3
3
2
4
(4)
(3)
3
3

Sol:

(4)
Solving the equations we get the points of
intersection (2, 1) and (2, 1)
The bounded region is shown as shaded
region.

y
(2, 1)

The required area = 2

(1 3y ) ( 2y )
2

(1, 0)

y3
2 4
= 2 (1 y 2 ) dy = 2 y = 2 = .
3 0
3 3

x + 2y2 = 0

(2, 1)

x + 3y2 = 1

31.

The value of

sin xdx

sin x

Sol:

is

(1) x + log cos x + c

(2) x log sin x + c

(3) x + log sin x + c

(4) x log cos x + c

(3)

sin x + dx
sin xdx
4 4

2
= 2

sin x
sin x
4
4

2 cos + cot x sin dx


4

4
4

= dx + cot x dx

= x + ln sin x + c .

32.

How many different words can be formed by jumbling the letters in the word MISSISSIPPI in which no
two S are adjacent?
(2) 6 . 7 . 8C4
(1) 8 . 6C4 . 7C4
7
(4) 7 . 6C4 . 8C4
(3) 6 . 8 . C4
FIITJEE Ltd., ICES House, 29 A, Kalu Sarai, Sarvapriya Vihar, New Delhi -110016, Ph 26515949, 26569493, Fax: 011-26513942.

FIITJEE Solutions to AIEEE - 2008


Sol:

11

(4)

Other than S, seven letters M, I, I, I, P, P, I can be arranged in

7!
= 7 . 5 . 3.
2! 4!

Now four S can be placed in 8 spaces in 8C4 ways.


Desired number of ways = 7 . 5 . 3 . 8C4 = 7 . 6C4 . 8C4.
33.

Let a, b, c be any real numbers. Suppose that there are real numbers x, y, z not all zero such that x =
cy + bz, y = az + cx and z = bx + ay. Then a2 + b2 + c2 + 2abc is equal to
(1) 2
(2) 1
(3) 0
(4) 1

Sol:

(4)
The system of equations x cy bz = 0, cx y + az = 0 and bx + ay z = 0 have non-trivial solution if
1 c b
2
c 1 a = 0 1(1 a ) + c(c ab) b(ca + b) = 0
b a 1

a2 + b2 + c2 + 2abc = 1.
34.

Let A be a square matrix all of whose entries are integers. Then which one of the following is true?
(1) If detA = 1, then A1 exists but all its entries are not necessarily integers
(2) If detA 1, then A1 exists and all its entries are non-integers
(3) If detA = 1, then A1 exists and all its entries are integers
(4) If detA = 1, then A1 need not exist

Sol:

(3)
Each entry of A is integer, so the cofactor of every entry is an integer and hence each entry in the
adjoint of matrix A is integer.
1
Now detA = 1 and A1 =
(adj A)
det(A)
all entries in A1 are integers.

35.

The quadratic equations x2 6x + a = 0 and x2 cx + 6 = 0 have one root in common. The other roots
of the first and second equations are integers in the ratio 4 : 3. Then the common root is
(1) 1
(2) 4
(3) 3
(4) 2

Sol:

(4)
Let and 4 be roots of x2 6x + a = 0 and , 3 be the roots of x2 cx + 6 = 0, then
+ 4 = 6 and 4 = a
+ 3 = c and 3 = 6.
We get = 2 a = 8
So the first equation is x2 6x + 8 = 0 x = 2, 4
If = 2 and 4 = 4 then 3 = 3
If = 4 and 4 = 2, then 3 = 3/2 (non-integer)
common root is x = 2.

Chemistry
PART B
36.

The organic chloro compound, which shows complete stereochemical inversion during a SN2 reaction,
is
(2) (CH3)3CCl
(1) (C2H5)2CHCl
(4) CH3Cl
(3) (CH3)2CHCl

Sol.

(4)
For SN2 reaction, the C atom is least hindered towards the attack of nucleophile in the case of
(CH3Cl).
Hence, (4) is the correct answer.
FIITJEE Ltd., ICES House, 29 A, Kalu Sarai, Sarvapriya Vihar, New Delhi -110016, Ph 26515949, 26569493, Fax: 011-26513942.

18

FIITJEE Solutions to AIEEE - 2009

Mathematics
PART C
a

61.

a +1

Let a, b, c be such that b ( a + c ) 0 . If b b + 1 b 1 + a 1


c c 1 c + 1 ( 1)n + 2 a
value of n is
(1) zero
(3) any odd integer

Sol:

a +1 a 1

b +1

c 1

b 1

c + 1 = 0, then the

( 1)

n +1

( 1)

(2) any even integer


(4) any integer

(3)
a a +1 a 1
a +1 b +1 c 1
a a +1 a 1
a +1 a 1 a
n
n
b b + 1 b 1 + ( 1) a 1 b 1 c + 1 = b b + 1 b 1 + ( 1) b + 1 b 1 b
c c 1 c +1
a
b
c
c c 1 c +1
c 1 c +1 c
a a +1 a 1
a +1 a a 1
a a +1 a 1
a a +1 a 1
n +1
n+ 2
= b b + 1 b 1 + ( 1) b + 1 b b 1 = b b + 1 b 1 + ( 1) b b + 1 b 1
c c 1 c +1
c 1 c c +1
c c 1 c +1
c c 1 c +1

This is equal to zero only if n + 2 is odd i.e. n is odd integer.


62.

If the mean deviation of number 1, 1 + d, 1 + 2d, .. , 1 + 100d from their mean is 255, then the d is
equal to
(1) 10.0
(2) 20.0
(3) 10.1
(4) 20.2

Sol:

(3)
n
a + l)
1
sum of quantities 2 (
= [1 + 1 + 100d] = 1 + 50d
=
Mean ( x ) =
2
n
n

1
1
2 d 50 x 51
xi x 255 = 101[50d + 49d + 48d + .... + d + 0 + d + ...... + 50d] = 101 2
n

255 x 101
d=
= 10.1
50 x 51

M.D. =

*63.

If the roots of the equation bx 2 + cx + a = 0 be imaginary, then for all real values of x, the expression
3b2 x 2 + 6bcx + 2c 2 is
(1) greater than 4ab
(2) less than 4ab
(3) greater than 4ab
(4) less than 4ab

Sol:

(3)
bx 2 + cx + a = 0
Roots are imaginary c 2 4ab < 0 c 2 < 4ab c 2 > 4ab
3b2 x 2 + 6bcx + 2c 2
since 3b2 > 0
Given expression has minimum value
4 3b2 2c 2 36b2 c 2
12b2c 2
=
= c 2 > 4ab .
Minimum value =
2
12b2
4 3b

)(

FIITJEE Ltd., FIITJEE House, Kalu Sarai, Sarvapriya Vihar, New Delhi -110016, Ph 46106000, 26515949, 26569493, Fax: 011-26513942.

FIITJEE Solutions to AIEEE - 2009


*64.

Let A and B denote the statements


A: cos + cos + cos = 0
B: sin + sin + sin = 0
If cos ( ) + cos ( ) + cos ( ) =

3
, then
2

(1) A is true and B is false


(3) both A and B are true

Sol:

19

(2) A is false and B is true


(4) both A and B are false

(3)
3
2
2 cos ( ) + cos ( ) + cos ( ) + 3 = 0
cos ( ) + cos ( ) + cos ( ) =

2 cos ( ) + cos ( ) + cos ( ) + sin2 + cos2 + sin2 + cos2 + sin2 + cos2 = 0


( sin + sin + sin ) + ( cos + cos + cos ) = 0
2

*65.

The lines p p2 + 1 x y + q = 0 and p2 + 1 x + p2 + 1 y + 2q = 0 are perpendicular to a common line


for
(1) no value of p
(3) exactly two values of p

(2) exactly one value of p


(4) more than two values of p

Sol:

(2)
Lines must be parallel, therefore slopes are equal p p2 + 1 = p2 + 1 p = - 1

66.

If A, B and C are three sets such that A B = A C and A B = A C , then


(1) A = B
(2) A = C
(3) B = C
(4) A B =

Sol:

(3)

67.

Sol:

r r uur
If u, v, w are non-coplanar vectors and p, q are real numbers, then the equality
r
r
uur
r uur
r
uur
r
r
3u pv pw pv w qu 2w qv qu = 0 holds for

(1) exactly one value of (p, q)


(2) exactly two values of (p, q)
(3) more than two but not all values of (p , q)
(4) all values of (p, q)

(1)

r r
pq + 2q2 u v
r r uur
But u v w 0
3p2 pq + 2q2 = 0

( 3p

uur
w = 0

7q2
q
7
q

= 0 2p2 + p + q2 = 0
2p2 + p2 pq + +
4
2
4
2

q
p = 0, q = 0, p =
2
This possible only when p = 0, q = 0 exactly one value of (p, q)
x 2 y 1 z + 2
=
=
lies in the plane x + 3y z + = 0 . Then ( , ) equals
3
5
2
(1) (6, - 17)
(2) ( - 6, 7)
(3) (5, - 15)
(4) ( - 5, 15)

68.

Let the line

Sol:

(2)
Drs of line = ( 3, 5, 2 )

FIITJEE Ltd., FIITJEE House, Kalu Sarai, Sarvapriya Vihar, New Delhi -110016, Ph 46106000, 26515949, 26569493, Fax: 011-26513942.

FIITJEE Solutions to AIEEE - 2009

20

Drs of normal to the plane = (1, 3, )


Line is perpendicular to normal 3 (1) 5 ( 3 ) + 2 ( ) = 0 3 15 2 = 0 2 = 12 = 6
Also ( 2, 1, 2 ) lies on the plane
2 + 3 + 6 ( 2 ) + = 0 = 7

( , ) = ( 6, 7 )
*69.

From 6 different novels and 3 different dictionaries, 4 novels and 1 dictionary are to be selected and
arranged in a row on the shelf so that the dictionary is always in the middle. Then the number of such
arrangements is
(1) less than 500
(2) at least 500 but less than 750
(3) at least 750 but less than 1000
(4) at least 1000

Sol:

(4)
4 novels can be selected from 6 novels in 6 C4 ways. 1 dictionary can be selected from 3 dictionaries
in 3 C1 ways. As the dictionary selected is fixed in the middle, the remaining 4 novels can be arranged
in 4! ways.
The required number of ways of arrangement = 6 C4 x 3 C1 x 4! = 1080

70.

[cot x ] dx , [] denotes the greatest integer function, is equal to


0

(1)

(2) 1
(4)

(3) 1

Sol:

(4)

Let I = [cot x ] dx

(1)

= cot ( x ) dx =
0

[ cot x ] dx

(2)

Adding (1) and (2)

2I =

[cot x ] dx
0

[ cot x ] dx
0

= =

( 1) dx
0

Q [ x ] + [ x ] = 1 if x Z

= 0 if x Z

= [ x ]0 =

I =
71.

For real x, let f ( x ) = x 3 + 5x + 1 , then


(1) f is one-one but not onto R
(3) f is one-one and onto R

Sol:

(2) f is onto R but not one-one


(4) f is neither one-one nor onto R

(3)
Given f ( x ) = x 3 + 5x + 1
Now f ' ( x ) = 3x 2 + 5 > 0, x R
f(x) is strictly increasing function
It is one-one
Clearly, f(x) is a continuous function and also increasing on R,
f ( x ) = and Lt f ( x ) =
Lt
x
x
f(x) takes every value between and .
Thus, f(x) is onto function.

FIITJEE Ltd., FIITJEE House, Kalu Sarai, Sarvapriya Vihar, New Delhi -110016, Ph 46106000, 26515949, 26569493, Fax: 011-26513942.

21

FIITJEE Solutions to AIEEE - 2009


72.

In a binomial distribution B n, p = , if the probability of at least one success is greater than or


4

9
equal to
, then n is greater than
10
1
1
(1)
(2)
4
3
4
log10 log10
log10 + log103

(3)

Sol:

log10

9
log103

(4)

log10

4
log103

(1)
n

1 qn

*73.

9
1
1
3

n log 3 10 n
4
10
10
log

log103
4
10
4

If P and Q are the points of intersection of the circles

x 2 + y 2 + 3x + 7y + 2p 5 = 0

and

x + y + 2x + 2y p = 0 , then there is a circle passing through P, Q and (1, 1) for


(1) all values of p
(2) all except one value of p
(3) all except two values of p
(4) exactly one value of p
2

Sol:

(1)
Given circles S = x 2 + y 2 + 3x + 7y + 2p 5 = 0
S' = x 2 + y 2 + 2x + 2y p2 = 0

Equation of required circle is S + S' = 0


As it passes through (1, 1) the value of =

( 7 + 2p )

(6 p )
2

If 7 + 2p = 0, it becomes the second circle


it is true for all values of p
74.

The projections of a vector on the three coordinate axis are 6, - 3, 2 respectively. The direction
cosines of the vector are
6 3 2
(1) 6, 3, 2
(2) , ,
5 5 5
6 3 2
6 3 2
(3) , ,
(4) , ,
7 7 7
7 7 7

Sol:

(3)
Projection of a vector on coordinate axis are x 2 x1, y 2 y1, z2 z1
x 2 x1 = 6, y 2 y1 = 3, z 2 z1 = 2

( x 2 x1 )

+ ( y 2 y1 ) + ( z 2 z1 ) = 36 + 9 + 4 = 7
2

The D.Cs of the vector are

*75.

If Z
(1)

6 3 2
, ,
7 7 7

4
= 2 , then the maximum value of Z is equal to
z

3 +1

(3) 2

Sol:

(2)

5 +1

(4) 2 + 2

(2)
4 4
4 4

Z = Z +
Z = Z +
Z Z
Z Z

4
4
4
Z Z +
Z 2+
Z Z
Z

FIITJEE Ltd., FIITJEE House, Kalu Sarai, Sarvapriya Vihar, New Delhi -110016, Ph 46106000, 26515949, 26569493, Fax: 011-26513942.

22

FIITJEE Solutions to AIEEE - 2009


2

Z 2 Z 4 0

( Z (

) ) ( Z (1 5 )) 0 1

5 +1

5 Z 5 +1

*76.

Three distinct points A, B and C are given in the 2 dimensional coordinate plane such that the ratio
of the distance of any one of them from the point (1, 0) to the distance from the point ( - 1, 0) is equal
1
to . Then the circumcentre of the triangle ABC is at the point
3
5
(2) , 0
(1) ( 0, 0 )
4
5

5
(3) , 0
(4) , 0
2
3

Sol:

(2)
P = (1, 0 ) ; Q ( 1, 0 )

Let A = ( x, y )
AP BP CP 1
=
=
=
AQ BQ CQ 3

..(1)

3AP = AQ 9AP2 = AQ2 9 ( x 1) + 9y 2 = ( x + 1) + y 2


2

9x 2 18x + 9 + 9y 2 = x 2 + 2x + 1 + y 2 8x 2 20x + 8y 2 + 8 = 0
5
(2)
x +1= 0
2
A lies on the circle
Similarly B, C are also lies on the same circle
x2 + y2

5
Circumcentre of ABC = Centre of Circle (1) = , 0
4

*77.

The remainder left out when 82n ( 62 )

2n +1

is divided by 9 is

(1) 0
(3) 7
Sol:

(2) 2
(4) 8

(2)
82n ( 62 )

2n +1

= (1 + 63 ) ( 63 1)

= (1 + 63 ) + (1 63 )
n

2n +1

2n +1

= 1 + nc1 63 + nc 2 ( 63 ) + .... + ( 63 )
2

= 2 + 63 n c1 + nc 2 ( 63 ) + .... + ( 63 )

n 1

2n +1)

c1 + (

2n +1)

) + (1

( 2n +1)

c 2 ( 63 ) + .... ( 63 )

c1 63 + (

( 2n )

2n +1)

c 2 ( 63 ) + .... + ( 1)( 63 )
2

( 2n +1)

Reminder is 2
*78.

The ellipse x 2 + 4y 2 = 4 is inscribed in a rectangle aligned with the coordinate axes, which in turn in
inscribed in another ellipse that passes through the point (4, 0). Then the equation of the ellipse is
(2) x 2 + 12y 2 = 16
(1) x 2 + 16y 2 = 16
(3) 4x 2 + 48y 2 = 48

Sol:

(4) 4x 2 + 64y 2 = 48

(2)

FIITJEE Ltd., FIITJEE House, Kalu Sarai, Sarvapriya Vihar, New Delhi -110016, Ph 46106000, 26515949, 26569493, Fax: 011-26513942.

23

FIITJEE Solutions to AIEEE - 2009


x2 y2
+
= 1 a = 2, b = 1 P = ( 2, 1)
4
1
x2 y2
x2 y2
Required Ellipse is 2 + 2 = 1 2 + 2 = 1
a
b
4
b
(2, 1) lies on it
V
4
1
1
1 3
4

+
= 1 2 = 1 = b2 =
16 b2
4 4
3
b
2
2
2
2
x
y
x
3y

+
= 1
+
= 1 x 2 + 12y 2 = 16
16 4
16
4
3

2 6 10 14
The sum to the infinity of the series 1 + + 2 + 3 + 4 + ...... is
3 3
3
3
(1) 2
(2) 3
(3) 4
(4) 6
x 2 + 4y 2 = 4

*79.

Sol:

P (2, 1)
1
A

V
(4, 0)

(2)

2 6 10 14
+
+
+
+ ....
3 32 33 34
1
1 2
6 10
S = + 2 + 3 + 4 + ....
3
3 3
3
3
Dividing (1) & (2)
1
1 4
4
4

S 1 = 1 + + 2 + 3 + 4 + ....
3 3
3
3
3

Let S = 1 +

(1)
(2)

2
4 4
1 1
2
4 4 1 4 4 3 4 2 6
2
6

S = + 2 1 + + 2 + ...... S = + 2
= + 2 = + = S= S=3
1
3
3
3
3 3 3 3
3
3
3
2
3
3
2
3
3

1
3

80.

The differential equation which represents the family of curves y = c1ec 2 x , where c1 and c 2 are
arbitrary constants is
(2) y " = y ' y
(1) y ' = y 2
(4) yy " = ( y ' )

(3) yy " = y '


Sol:

(4)
y = c1ec 2 x

(1)

y ' = c 2 c1e

c2 x

y ' = c2 y
y " = c2 y '
From (2)
y'
c2 =
y
So, y " =

(2)

( y ')
y

yy " = ( y ' )

81.

One ticket is selected at random from 50 tickets numbered 00, 01, 02, ., 49. Then the probability
that the sum of the digits on the selected ticket is 8, given that the product of these digits is zero,
equals
1
1
(2)
(1)
14
7
1
5
(3)
(4)
50
14

Sol:

(1)

FIITJEE Ltd., FIITJEE House, Kalu Sarai, Sarvapriya Vihar, New Delhi -110016, Ph 46106000, 26515949, 26569493, Fax: 011-26513942.

24

FIITJEE Solutions to AIEEE - 2009


S = { 00, 01, 02, ., 49 }
Let A be the even that sum of the digits on the selected ticket is 8 then
A = { 08, 17, 26, 35, 44 }
Let B be the event that the product of the digits is zero
B = { 00, 01, 02, 03, ., 09, 10, 20, 30, 40 }
A B = {8}
1
1
= 50 =
Required probability = P ( A / B ) =
14 14
P (B )
50
2x
Let y be an implicit function of x defined by x 2x x cot y 1 = 0 . Then y ' (1) equals
P ( A B)

82.

(1) 1
(3) log 2
Sol:

(2) 1
(4) log 2

(1)
x 2x 2x x cot y 1 = 0
Now x = 1,

(1)

1 2 coty 1 = 0 coty = 0 y =

Now differentiating eq. (1) w.r.t. x


dy

2x 2x (1 + log x ) 2 x x ( c osec 2 y )
+ cot y x x (1 + log x ) = 0
dx


Now at 1,
2

dy
2 (1 + log1) 2 1( 1)
+ 0 = 0

dx 1,
2

dy
dy
2 + 2
=0
= 1
dx 1,
dx 1,

83.

The area of the region bounded by the parabola ( y 2 ) = x 1 , the tangent to the parabola at the
2

point (2, 3) and the x-axis is


(1) 3
(3) 9
Sol:

(2) 6
(4) 12

(3)

Equation

( y 2)

of

tangent

at

(2,

3)

to
2y = x + 4

= x 1 is S1 = 0

x 2y + 4 = 0
Required Area = Area of OCB + Area of
OAPD Area of PCD
3
1
1
= ( 4 x 2 ) + y 2 4y + 5 dy (1 x 2 )
2
2
0

y3

= 4 + 2y 2 + 5y 1 = 4 9 18 + 15 1
3

0
= 28 19 = 9 sq. units

D (0, 3)
P (2, 3)
C (0, 2)

B (-4, 0)

A (1, 2)
A (1, 2)
0

(or)
FIITJEE Ltd., FIITJEE House, Kalu Sarai, Sarvapriya Vihar, New Delhi -110016, Ph 46106000, 26515949, 26569493, Fax: 011-26513942.

25

FIITJEE Solutions to AIEEE - 2009


3

Area =

( 2y 4 y

84.

+ 4y 5 dy = y + 6y 5 dy = ( 3 y )
2

( y 3 )3
27
=
dy =
= 9 sq.units
3
3
0

Given P ( x ) = x 4 + ax 3 + bx 2 + cx + d such that x = 0 is the only real root of P' ( x ) = 0 . If P ( 1) < P (1) ,
then in the interval [ 1, 1]
(1) P ( 1) is the minimum and P (1) is the maximum of P
(2) P ( 1) is not minimum but P (1) is the maximum of P
(3) P ( 1) is the minimum and P (1) is not the maximum of P
(4) neither P ( 1) is the minimum nor P (1) is the maximum of P

Sol:

(2)
P ( x ) = x 4 + ax 3 + bx 2 + cx + d
P ' ( x ) = 4x 3 + 3ax 2 + 2bx + c

Q x = 0 is a solution for P' ( x ) = 0 , c = 0

P ( x ) = x 4 + ax 3 + bx 2 + d

(1)

Also, we have P ( 1) < P (1)

1 a + b + d < 1+ a + b + d a > 0
Q P ' ( x ) = 0 , only when x = 0 and P(x) is differentiable in ( - 1, 1), we should have the maximum and
minimum at the points x = - 1, 0 and 1 only
Also, we have P ( 1) < P (1)
Max. of P(x) = Max. { P(0), P(1) } & Min. of P(x) = Min. { P(-1), P(0) }
In the interval [ 0 , 1 ],
P' ( x ) = 4x 3 + 3ax 2 + 2bx = x 4x 2 + 3ax + 2b

Q P ' ( x ) has only one root x = 0, 4x + 3ax + 2b = 0 has no real roots.


2

( 3a ) 32b < 0
2

3a2
<b
32

b>0
Thus, we have a > 0 and b > 0
P' ( x ) = 4x 3 + 3ax 2 + 2bx > 0, x ( 0, 1)
Hence P(x) is increasing in [ 0, 1 ]
Max. of P(x) = P(1)
Similarly, P(x) is decreasing in [-1 , 0]
Therefore Min. P(x) does not occur at x = - 1
85.

Sol:

The shortest distance between the line y x = 1 and the curve x = y 2 is


(1)

3 2
8

(2)

2 3
8

(3)

3 2
5

(4)

3
4

(1)
x y +1= 0
x=y

(1)

1 = 2y

dy
dy
1
= Slope of given line (1)

=
dx
dx 2y
2

1
1
1
1
1
1 1
= 1 y = y = x = = ( x, y ) = ,
2
2
4
2y
2

4 2

FIITJEE Ltd., FIITJEE House, Kalu Sarai, Sarvapriya Vihar, New Delhi -110016, Ph 46106000, 26515949, 26569493, Fax: 011-26513942.

FIITJEE Solutions to AIEEE - 2009


The shortest distance is

1 1
+1
4 2
1+ 1

3
4 2

26

3 2
8

Directions: Question number 86 to 90 are Assertion Reason type questions. Each of these questions
contains two statements
Statement-1 (Assertion) and Statement-2 (Reason).
Each of these questions also have four alternative choices, only one of which is the correct answer. You have
to select the correct choice

86.

Let f ( x ) = ( x + 1) 1, x 1
2

Statement-1 : The set x : f ( x ) = f 1 ( x ) = {0, 1}


Statement-2 : f is a bijection.
(1) Statement-1 is true, Statement-2 is true; Statement-2 is a correct explanation for Statement-1
(2) Statement-1 is true, Statement-2 is true; Statement-2 is not a correct explanation for Statement-1
(3) Statement-1 is true, Statement-2 is false
(4) Statement-1 is false, Statement-2 is true
Sol:

(3)
There is no information about co-domain therefore f(x) is not necessarily onto.

87.

Let f ( x ) = x x and g ( x ) = sin x .


Statement-1 : gof is differentiable at x = 0 and its derivative is continuous at that point.
Statement-2 : gof is twice differentiable at x = 0.
(1) Statement-1 is true, Statement-2 is true; Statement-2 is a correct explanation for Statement-1
(2) Statement-1 is true, Statement-2 is true; Statement-2 is not a correct explanation for Statement-1
(3) Statement-1 is true, Statement-2 is false
(4) Statement-1 is false, Statement-2 is true

Sol:

(3)
f ( x ) = x x and g ( x ) = sin x
sin x 2 ,x < 0
gof ( x ) = sin ( x x ) =
2
,x 0
sin x
2x cos x 2 ,x < 0
( gof ) ' ( x ) =
2
,x 0
2x cos x
Clearly, L ( gof ) ' ( 0 ) = 0 = R ( gof ) ' ( 0 )

gof is differentiable at x = 0 and also its derivative is continuous at x = 0


2cos x 2 + 4x 2 sin x 2 ,x < 0
Now, ( gof ) " ( x ) =
2
2
2
,x 0
2cos x 4x sin x
L ( gof ) " ( 0 ) = 2 and R ( gof ) " ( 0 ) = 2
L ( gof ) " ( 0 ) R ( gof ) " ( 0 )

gof(x) is not twice differentiable at x = 0.


*88.

Statement-1 : The variance of first n even natural numbers is


Statement-2 : The sum of first n natural numbers is
numbers is

n ( n + 1)( 2n + 1)

n ( n + 1)
2

n2 1
4

and the sum of squares of first n natural

6
(1) Statement-1 is true, Statement-2 is true; Statement-2 is a correct explanation for Statement-1
(2) Statement-1 is true, Statement-2 is true; Statement-2 is not a correct explanation for Statement-1
(3) Statement-1 is true, Statement-2 is false

FIITJEE Ltd., FIITJEE House, Kalu Sarai, Sarvapriya Vihar, New Delhi -110016, Ph 46106000, 26515949, 26569493, Fax: 011-26513942.

27

FIITJEE Solutions to AIEEE - 2009


(4) Statement-1 is false, Statement-2 is true
Sol:

(4)
Statement-2 is true
Statement-1:
Sum of n even natural numbers = n (n + 1)
n ( n + 1)
Mean x =
= n +1
n
2
1
2
2
2
1
Variance = ( x i ) x = 22 + 42 + ..... + ( 2n ) ( n + 1)

n
n

()

()

1 2 2
4 n ( n + 1)( 2n + 1)
2
2
2 1 + 22 + ..... + n2 ( n + 1) =
( n + 1)
n
n
6
(n + 1) 2 ( 2n + 1) 3 (n + 1) (n + 1) [ 4n + 2 3n 3] (n + 1)(n 1) n2 1
=
=
=
=
3
3
3
3
Statement 1 is false.
=

89.

Statement-1 : ~ ( p ~ q ) is equivalent to p q .
Statement-2 : ~ ( p ~ q) is a tautology.
(1) Statement-1 is true, Statement-2 is true; Statement-2 is a correct explanation for Statement-1
(2) Statement-1 is true, Statement-2 is true; Statement-2 is not a correct explanation for Statement-1
(3) Statement-1 is true, Statement-2 is false
(4) Statement-1 is false, Statement-2 is true

Sol:

(3)

p
T
T
F
F

90.

pq

q
T
F
T
F

T
F
F
T

~q
F
T
F
T

p ~ q

~ ( p ~ q)

F
T
T
F

T
F
F
T

Let A be a 2 x 2 matrix
Statement-1 : adj ( adj A ) = A
Statement-2 : adj A = A
(1) Statement-1 is true, Statement-2 is true; Statement-2 is a correct explanation for Statement-1
(2) Statement-1 is true, Statement-2 is true; Statement-2 is not a correct explanation for Statement-1
(3) Statement-1 is true, Statement-2 is false
(4) Statement-1 is false, Statement-2 is true

Sol:

(2)

adj A = A

n 1

= A

adj ( adj A ) = A

n2

2 1

= A
0

A= A A=A

FIITJEE Ltd., FIITJEE House, Kalu Sarai, Sarvapriya Vihar, New Delhi -110016, Ph 46106000, 26515949, 26569493, Fax: 011-26513942.

AIEEE
2010
20

59.

59.
Sol.

A small particle of mass m is projected at an angle with the


x-axis with an initial velocity v0 in the x-y plane as shown in the
v sin
figure. At a time t < 0
, the angular momentum of the
g
particle is
(1) mgv 0 t 2 cos j
(2) mgv 0 t cos k

1
1
(3) mgv 0 t 2 cos k
(4) mgv 0 t 2 cos i
2
2
where i, j and k are unit vectors along x, y and zaxis respectively.
3
L = m(r v)

1
L = m v 0 cos t i + (v 0 sin t gt 2 )j v 0 cos i + (v 0 sin gt)j
2
1
= mv 0 cos t gt k
2
1
= mgv 0 t 2 cos k
2
60.

The equation of a wave on a string of linear mass density 0.04 kg m

t
x
y = 0.02(m)sin 2

0.04(s) 0.50(m)
60.

(1) 4.0 N
4

Sol.

T = v 2 =

61.

Let cos( + ) =
(1)

61.

is given by

. The tension in the string is

(2) 12.5 N

(3) 0.5 N

(4) 6.25 N

2
(2 / 0.004)2
=
= 6.25 N
0.04
k2
(2 / 0.50)2

56
33

4
5
and let sin( ) =
, where 0 , , then tan 2 =
5
13
4
19
20
25
(2)
(3)
(4)
12
7
16

3
4
5
tan( ) =
12
3 5
+
56
tan 2 = tan( + + ) = 4 12 =
3 5
33
1
4 12
4
5
5
sin( ) =
13
cos ( + ) =

62.

tan( + ) =

Let S be a non-empty subset of R. Consider the following statement:


P: There is a rational number x S such that x > 0.
Which of the following statements is the negation of the statement P ?
(1) There is no rational number x S such that x 0
(2) Every rational number x S satisfies x 0

(Hyderabad Classes) Limited. 5-9-14/B, Saifabad, (Opp. Secretariat) Hyderabad. 500 063. Phone: 040-66777000 03 Fax: 040-66777004

AIEEE
2010
21

62.

(3) x S and x 0
x is not rational
(4) There is a rational number x S such that x 0
2
P: there is a rational number x S such that x > 0
~P: Every rational number x S satisfies x 0

63.

Let a = j k and c = i j k . Then vector b satisfying a b + c = 0 and a b = 3 is


(1) 2i j + 2k
(2) i j 2k
(3) i + j 2k
(4) i + j 2k

63.

4
c = ba
bc = 0
b1i + b2 j + b3k i j k = 0

)(

b1 b2 b3 = 0
and a b = 3
b2 b3 = 3
b1 = b2 + b3 = 3 + 2b3
b = ( 3 + 2b3 ) i + ( 3 + b3 ) j + b3k .
64.

The equation of the tangent to the curve y = x +

64.

(1) y = 1
3
Parallel to x-axis

(2) y = 2

4
, that is parallel to the x-axis, is
x2
(3) y = 3
(4) y = 0

dy
=0
dx

x=2
y=3
Equation of tangent is y 3 = 0(x 2)
65.
65.

8
=0
x3

y3=0

is
2
(3) tan x = (sec x + c)y (4) sec x = (tan x + c)y

Solution of the differential equation cos x dy = y(sin x y) dx, 0 < x <


(1) y sec x = tan x + c (2) y tan x = sec x + c
4
cos x dy = y(sin x y) dx
dy
= y tan x y 2 sec x
dx
1 dy 1
tan x = sec x
y 2 dx y
Let

1
=t
y

1 dy dt
=
y 2 dx dx

dy
dt
t tan x = sec x
+ (tan x) t = sec x.
dx
dx
tan x dx
I.F. = e
= sec x
Solution is t(I.F) = (I.F) sec x dx

(Hyderabad Classes) Limited. 5-9-14/B, Saifabad, (Opp. Secretariat) Hyderabad. 500 063. Phone: 040-66777000 03 Fax: 040-66777004

AIEEE
2010
22

1
sec x = tan x + c
y
66.

The area bounded by the curves y = cos x and y = sin x between the ordinates x = 0 and x =

66.

(1) 4 2 + 2
4

(2) 4 2 1

(3) 4 2 + 1

5
4

3
2

5
4

( cos x sin x ) dx + ( sin x cos x ) dx + ( cos x sin x ) = 4

cos x

(4) 4 2 2

2 2

sin x

3
is
2

67.

If two tangents drawn from a point P to the parabola y = 4x are at right angles, then the locus of P is
(1) 2x + 1 = 0
(2) x = 1
(3) 2x 1 = 0
(4) x = 1
2
The locus of perpendicular tangents is directrix
i.e, x = a; x = 1

68.

b = 2i + 4j + k and c = i + j + k are mutually orthogonal, then (, ) =


If the vectors a = i j + 2k,

67.

68.

69.

(1) (2, 3)
(2) (2, 3)
4
a b = 0,
b c = 0,
c a = 0
2 + 4 + = 0
1 + 2 = 0
Solving we get: = 3, = 2

(4) (3, 2)

Consider the following relations:


R = {(x, y) | x, y are real numbers and x = wy for some rational number w};
S=

69.

(3) (3, 2)

m p
,
n q

m, n, p and q are integers such that n, q 0 and qm = pn . Then

(1) neither R nor S is an equivalence relation


(2) S is an equivalence relation but R is not an equivalence relation
(3) R and S both are equivalence relations
(4) R is an equivalence relation but S is not an equivalence relation
2
xRy need not implies yRx
m p
S:
s qm = pn
n q
m m
s
reflexive
n n

(Hyderabad Classes) Limited. 5-9-14/B, Saifabad, (Opp. Secretariat) Hyderabad. 500 063. Phone: 040-66777000 03 Fax: 040-66777004

AIEEE201023

m p
s
n q
m p p r
s , s
n q q s

p m
s
symmetric
q n
qm = pn, ps = rq

ms = rn transitive.

S is an equivalence relation.
70.

Let f: R R be defined by f(x) =

k 2x, if x 1
. If f has a local minimum at x = 1, then a
2x + 3, if x > 1

possible value of k is
(2)

(1) 0
70.

3
f(x) = k 2x
= 2x + 3

1
2

(3) 1

(4) 1

if x 1
if x > 1
2x + 3

k 2x
1
1

lim f(x) 1

x 1

71.
71.

This is true
where k = 1

The number of 3 3 non-singular matrices, with four entries as 1 and all other entries as 0, is
(1) 5
(2) 6
(3) at least 7
(4) less than 4
3
First row with exactly one zero; total number of cases = 6
First row 2 zeros we get more cases
Total we get more than 7.

Directions: Questions Number 72 to 76 are Assertion Reason type questions. Each of these questions
contains two statements.
Statement-1: (Assertion) and Statement-2: (Reason)
Each of these questions also has four alternative choices, only one of which is the correct answer.
You have to select the correct choice.

72.

72.

Four numbers are chosen at random (without replacement) from the set {1, 2, 3, ....., 20}.
Statement-1: The probability that the chosen numbers when arranged in some order will form an AP
1
is
.
85
Statement-2: If the four chosen numbers from an AP, then the set of all possible values of common
difference is {1, 2, 3, 4, 5}.
(1) Statement-1 is true, Statement-2 is true; Statement-2 is not the correct explanation for Statement-1
(2) Statement-1 is true, Statement-2 is false
(3) Statement-1 is false, Statement-2 is true
(4) Statement-1 is true, Statement-2 is true; Statement-2 is the correct explanation for Statement-1
2

(Hyderabad Classes) Limited. 5-9-14/B, Saifabad, (Opp. Secretariat) Hyderabad. 500 063. Phone: 040-66777000 03 Fax: 040-66777004

AIEEE201024

20

N(S) = C4
Statement-1: common difference is 1; total number of cases = 17
common difference is 2; total number of cases = 14
common difference is 3; total number of cases = 11
common difference is 4; total number of cases = 8
common difference is 5; total number of cases = 5
common difference is 6; total number of cases = 2
17 + 14 + 11 + 8 + 5 + 2
1
Prob. =
.
=
20
85
C4
73.

73.

74.

Statement-1: The point A(3, 1, 6) is the mirror image of the point B(1, 3, 4) in the plane x y + z = 5.
Statement-2: The plane x y + z = 5 bisects the line segment joining A(3, 1, 6) and B(1, 3, 4).
(1) Statement-1 is true, Statement-2 is true; Statement-2 is not the correct explanation for Statement-1
(2) Statement-1 is true, Statement-2 is false
(3) Statement-1 is false, Statement-2 is true
(4) Statement-1 is true, Statement-2 is true; Statement-2 is the correct explanation for Statement-1
1
A(3, 1, 6); B = (1, 3, 4)
Mid-point of AB = (2, 2, 5) lies on the plane.
and d.rs of AB = (2, 2, 2)
d.rs Of normal to plane = (1, 1, 1).
AB is perpendicular bisector
A is image of B
Statement-2 is correct but it is not correct explanation.
10

Let S1 =

j =1

j ( j 1) 10 C j , S2 =

10

10

j =1

C j and S3 =

10

j2

j =1

10

Cj .

74.

Statement-1: S3 = 55 2
8
8
Statement-2: S1 = 90 2 and S2 = 10 2 .
(1) Statement-1 is true, Statement-2 is true; Statement-2 is not the correct explanation for Statement-1
(2) Statement-1 is true, Statement-2 is false
(3) Statement-1 is false, Statement-2 is true
(4) Statement-1 is true, Statement-2 is true; Statement-2 is the correct explanation for Statement-1
2
10
10
10!
8!
S1 =
j ( j 1)
= 90
= 90 28 .
j
j

1
j

2
!
10

j
!
j

2
!
8

2
!
( )( ) (
)
) ( ( ))
j =1
j= 2 (

S2 =

10
j =1

S3 =

10
j =1

10
10!
9!
= 10
= 10 29 .
j ( j 1)! ( 9 ( j 1) )!
j

1
!
9

1
!
) ( ( ))
j =1 (

j ( j 1) + j

10!
=
j! (10 j )!

10
j =1

j ( j 1) 10 C j =
8

10
j =1

j 10 C j = 90 . 2 + 10 . 2
8

= 90 . 2 + 20 . 2 = 110 . 2 = 55 . 2 .
75.

Let A be a 2 2 matrix with non-zero entries and let A = I, where I is 2 2 identity matrix. Define
Tr(A) = sum of diagonal elements of A and |A| = determinant of matrix A.
Statement-1: Tr(A) = 0
Statement-2: |A| = 1
(1) Statement-1 is true, Statement-2 is true; Statement-2 is not the correct explanation for Statement-1
(2) Statement-1 is true, Statement-2 is false

(Hyderabad Classes) Limited. 5-9-14/B, Saifabad, (Opp. Secretariat) Hyderabad. 500 063. Phone: 040-66777000 03 Fax: 040-66777004

AIEEE201025

75.

(3) Statement-1 is false, Statement-2 is true


(4) Statement-1 is true, Statement-2 is true; Statement-2 is the correct explanation for Statement-1
2
a b
Let A =
, abcd 0
c d
a b
a b

c d
c d

A =
2

A =

a2 + bc ab + bd
ac + cd bc + d2

a2 + bc = 1, bc + d2 = 1
ab + bd = ac + cd = 0
c 0 and b 0
a+d=0
Trace A = a + d = 0
|A| = ad bc = a2 bc = 1.
76.

Let f: R R be a continuous function defined by f(x) =

1
.
e + 2e x
x

1
, for some c R.
3
1
Statement-2: 0 < f(x)
, for all x R
2 2
(1) Statement-1 is true, Statement-2 is true; Statement-2 is not the correct explanation for Statement-1
(2) Statement-1 is true, Statement-2 is false
(3) Statement-1 is false, Statement-2 is true
(4) Statement-1 is true, Statement-2 is true; Statement-2 is the correct explanation for Statement-1
4
1
ex
f(x) = x
=
e + 2e x e2x + 2
Statement-1: f(c) =

76.

f(x) =

(e

2x

+ 2 e x 2e2x e x

(e )

2x + 2 2

2x

f(x) = 0
2x
e =2

e + 2 = 2e
x
e = 2

maximum f(x) =
0 < f(x)

2
1
=
4
2 2

2 2
1
1
Since 0 < <
3 2 2
1
f(c) =
3

77.

2x

xR
for some c R

For a regular polygon, let r and R be the radii of the inscribed and the circumscribed circles. A false
statement among the following is
r
1
r 2
(1) There is a regular polygon with
=
(2) There is a regular polygon with
=
R
R 3
2

(Hyderabad Classes) Limited. 5-9-14/B, Saifabad, (Opp. Secretariat) Hyderabad. 500 063. Phone: 040-66777000 03 Fax: 040-66777004

AIEEE201026

(3) There is a regular polygon with


77.

r
3
=
R
2

(4) There is a regular polygon with

r 1
=
R 2

cot
2
n
a is side of polygon.
a

R = cosec
2
n

cot
r
n = cos
=

R
n
cosec
n
2
for any n N.
cos
n 3
r=

78.
78.

2009

If and are the roots of the equation x x + 1 = 0, then


(1) 1
(2) 1
(3) 2
2
2

x x+1=0

1 3 i
2
1
3
= +i
,
2
2

= cos + isin ,
3
3

x=

2009

=
(4) 2

1 1 4
2

x=

1 i 3

2
2

= cos isin
3
3

2009
2009

+
= 2cos2009
3
2
2
= 2cos 668 + +
= 2cos +
3
3
2
1
= 2
=1
= 2cos
3
2
79.
79.

80.

80.

The number of complex numbers z such that |z 1| = |z + 1| = |z i| equals


(1) 1
(2) 2
(3)
(4) 0
1
Let z = x + iy
|z 1| = |z + 1|
Re z = 0
x=0
x=y
|z 1| = |z i|
|z + 1| = |z i|
y = x
Only (0, 0) will satisfy all conditions.
Number of complex number z = 1
A line AB in three-dimensional space makes angles 45 and 120 with the positive x-axis and the
positive y-axis respectively. If AB makes an acute angle with the positive z-axis, then equals
(1) 45
(2) 60
(3) 75
(4) 30
2

(Hyderabad Classes) Limited. 5-9-14/B, Saifabad, (Opp. Secretariat) Hyderabad. 500 063. Phone: 040-66777000 03 Fax: 040-66777004

AIEEE201027

= cos 45 =

1
2

m = cos 120 =

1
2

n = cos
where is the angle which line makes with positive z-axis.
2
2
2
Now + m + n = 1

1 1
2
+ + cos = 1
2 4
1
2
cos =
4
1
cos =
2

= .
3
81.

The line L given by


the equation
(1)

81.

17

( Being acute)

x y
+ = 1 passes through the point (13, 32). The line K is parallel to L and has
5 b

x y
+ = 1. Then the distance between L and K is
c 3
17
23
(2)
(3)
15
17

(4)

23
15

b
5
3
Slope of line K =
c
Line L is parallel to line k.
b 3
bc = 15
=
5 c
(13, 32) is a point on L.
13 32
32
8
+
=1
=
5
b
b
5
3
b = 20
c=
4
Equation of K: y 4x = 3
52 32 + 3
23
Distance between L and K =
=
17
17

Slope of line L =

82.

82.

th

A person is to count 4500 currency notes. Let an denote the number of notes he counts in the n
minute. If a1 = a2 = ...... = a10 = 150 and a10, a11, ...... are in A.P. with common difference 2, then the
time taken by him to count all notes is
(1) 34 minutes
(2) 125 minutes
(3) 135 minutes
(4) 24 minutes
1
th
Till 10 minute number of counted notes = 1500
n
3000 = [2 148 + (n 1)(2)] = n[148 n + 1]
2
(Hyderabad Classes) Limited. 5-9-14/B, Saifabad, (Opp. Secretariat) Hyderabad. 500 063. Phone: 040-66777000 03 Fax: 040-66777004

AIEEE201028

n 149n + 3000 = 0
n = 125, 24
n = 125 is not possible.
Total time = 24 + 10 = 34 minutes.
83.

Let f: R R be a positive increasing function with lim

(1)
83.

2
3

(2)

3
2

f(3x)
f(2x)
= 1. Then lim
=
x f(x)
f(x)

(3) 3

(4) 1

4
f(x) is a positive increasing function
0 < f(x) < f(2x) < f(3x)
f(2x) f(3x)
<
0<1<
f(x)
f(x)
f(2x)
f(3x)
lim 1 lim
lim
x
x f(x)
x f(x)

By sandwich theorem.
f(2x)
lim
=1
x f(x)
84.

Let p(x) be a function defined on R such that p(x) = p(1 x), for all x [0, 1], p(0) = 1 and p(1) = 41.
1

p(x) dx equals

Then
0

84.

(1) 21
(2) 41
1
p(x) = p(1 x)
p(x) = p(1 x) + c
at x = 0
p(0) = p(1) + c
42 = c
now p(x) = p(1 x) + 42
p(x) + p(1 x) = 42
1

I=

(3) 42

(4)

41

p(x) dx = p(1 x) dx

0
1

2I=

(42) dx

I = 21.

85.

85.

Let f: (1, 1) R be a differentiable function with f(0) = 1 and f(0) = 1. Let g(x) = [f(2f(x) + 2)] .
Then g(0) =
(1) 4
(2) 0
(3) 2
(4) 4
1
d
g(x) = 2(f(2f(x) + 2))
( f ( 2f(x) + 2)) = 2f(2f(x) + 2) f(2f(x) + 2) . (2f(x))
dx
g(0) = 2f(2f(0) + 2) . f(2f(0) + 2) . 2(f(0) = 4f(0) f(0)
= 4(1) (1) = 4

(Hyderabad Classes) Limited. 5-9-14/B, Saifabad, (Opp. Secretariat) Hyderabad. 500 063. Phone: 040-66777000 03 Fax: 040-66777004

AIEEE201029

86.

There are two urns. Urn A has 3 distinct red balls and urn B has 9 distinct blue balls. From each urn
two balls are taken out at random and then transferred to the other. The number of ways in which
this can be done is
(1) 36
(2) 66
(3) 108
(4) 3

86.

3
3
9
Total number of ways = C2 C2
98
=3
= 3 36 = 108
2

87.

Consider the system of linear equations:


x1 + 2x2 + x3 = 3
2x1 + 3x2 + x3 = 3
3x1 + 5x2 + 2x3 = 1
The system has
(1) exactly 3 solutions
(3) no solution
3
1 2 1

87.

(2) a unique solution


(4) infinite number of solutions

D= 2 3 1=0

3 5 2

3 2 1
D1 = 3 3 1 0
1 5 2
Given system, does not have any solution.
No solution.
88.

88.

89.

89.

An urn contains nine balls of which three are red, four are blue and two are green. Three balls are
drawn at random without replacement from the urn. The probability that the three balls have different
colour is
2
1
2
1
(1)
(2)
(3)
(4)
7
21
23
3
1
9
n(S) = C3
3
4
2
n(E) = C1 C1 C1
3 4 2 24 3!
24 6
2
Probability = 9
=
6! =
= .
9!
987 7
C3
For two data sets, each of size 5, the variances are given to be 4 and 5 and the corresponding
means are given to be 2 and 4, respectively. The variance of the combined data set is
11
13
5
(2) 6
(3)
(4)
(1)
2
2
2
1
2
x = 4
2
y = 5
x= 2
y= 4

(Hyderabad Classes) Limited. 5-9-14/B, Saifabad, (Opp. Secretariat) Hyderabad. 500 063. Phone: 040-66777000 03 Fax: 040-66777004

AIEEE201030

xi
=2
5
1
2
x =
2

xi = 10;

xi2 ( x ) =
2

yi = 20

1
( yi2 ) 16
5

xi = 40
2
yi = 105
2

z =
90.
90.

1
10

xi 2 +
2

y i2

x+y
2

1
145 90 55 11
=
=
( 40 + 105 ) 9 =
10
10
10 2

The circle x + y = 4x + 8y + 5 intersects the line 3x 4y = m at two distinct points if


(1) 35 < m < 15
(2) 15 < m < 65
(3) 35 < m < 85
(4) 85 < m < 35
1
2
2
Circle x + y 4x 8y 5 = 0
Centre = (2, 4), Radius = 4 + 16 + 5 = 5
If circle is intersecting line 3x 4y = m
at two distinct points.
length of perpendicular from centre < radius
6 16 m
<5
5
|10 + m| < 25
25 < m + 10 < 25
35 < m < 15.

***

(Hyderabad Classes) Limited. 5-9-14/B, Saifabad, (Opp. Secretariat) Hyderabad. 500 063. Phone: 040-66777000 03 Fax: 040-66777004

Potrebbero piacerti anche